历年高考英语阅读汇总120篇详细解析

申明敬告: 本站不保证该用户上传的文档完整性,不预览、不比对内容而直接下载产生的反悔问题本站不予受理。

文档介绍

历年高考英语阅读汇总120篇详细解析

· 历年高考英语阅读汇总120篇 ‎ (共120篇+详细解析)‎ 阅读下列短文,从每题所给的A、B、C和D项中,选出最佳选项。‎ ‎   (第1篇)‎ ‎  You know how wonderful you are, and you know that others know how wonderful you are, but what do you do when admiration crosses over the line into jealousy? For most teens there will come a day when you realize that one of your friends is jealous and that this jealousy is hurting your friendship. When this happens it can seem like there is nothing that you can do, but the good news is that there is. Don't let jealousy spoil your relationships. Tackle it headon and you might be back to normal much sooner than you think.‎ ‎  It can be hard to walk up to a friend and ask them what the problem is, but if you want to save your friendship you'll have to do just that. Don't approach them and ask why they are jealous of you (unless of course you want to appear totally conceited), just take some time alone with them and let them know that you've been feeling like there's been something coming between you. If they refuse to respond, then use the opportunity to explain how you have been feeling. Chances are that something you say will strike a nerve and your friend will open up as well.‎ ‎  When you figure out what is annoying your friend, ask him or her what (s)he thinks would make the situation better. If, for example, (s)he says that (s)he feels like (s)he doesn't get to spend any time with you because of your being off with your new friends from tee swim team then maybe you could invite her along the next time or block off one day a week for just the two of you. Remember, though, that whatever solution you decide on should be a compromise. Don't limit your own talents or opportunities simply because your friend is unhappy. Try instead to include him or her in your new life and see how that works out.‎ ‎  Even the best of friendships can be tinged by jealousy. This destructive emotion is rarely productive and can turn best friends into worst enemies. Before taking extreme action, chat with your jealous friend to see if the two of you can work out a compromise.If you can't, be prepared to know exactly how far you will go to keep your friendand how far you won't.‎ ‎  解读:本文是关于人际关系的话题。尤其是青少年朋友之间会因为嫉妒而使相互间的友谊被破坏,本文着重讲述如何处理被朋友嫉妒及处理过程中可能遇到的困难。‎ ‎  1. According to the author, the jealousy emotion is________.‎ ‎  A. normal B. productive ‎  C. destructive D. extreme ‎  [解析] 细节理解题,由第一段第二句...that this jealousy is hurting,your friendship.可知。‎ ‎  [答案] C ‎  2. Which of the following is NOT mentioned as a way to confront your friend when jealousy happens?‎ ‎  A. Walking up to him/her and asking him/her why he/she is jealous of you.‎ ‎  B. Walking up to him/her and asking him/her what the problem is.‎ ‎  C. Spending some time with him/her and letting him/her know how you feel.‎ ‎  D. Spending some time with him/her and letting him/her know you think there's something between you.‎ ‎  [解析] 细节理解题。由第二段第二句Don't approach them and ask why they are jealous of you可知。‎ ‎  [答案] A ‎  3. The underlined "it" in the first paragraph refers to________.‎ ‎  A. friendship B. relationship ‎  C. admiration D. jealousy ‎  [解析] 细节理解题。由文章第一段可知。‎ ‎  [答案] D ‎  4. What can be inferred from the last two paragraphs?‎ ‎  A. There's always a solution to solve the problem of jealousy.‎ ‎  B. Jealousy can turn best friends into worst enemies.‎ ‎  C. You may lose a friend to keep your own gifts, chances or self development.‎ ‎  D. You should go a long way with your friend to work out a solution ‎  [解析] 推理判断题。‎ ‎  [答案] C ‎  5. The purpose of the passage is________.‎ ‎  A. to explain what causes jealousy ‎  B. to offer some advice on making friends ‎  C. to introduce the way to cope with a jealous friend ‎  D. to explain how destructive the jealous emotion is ‎  [解析] 作者意图题。通读全文可知,本文酌写作目的是介绍处理有嫉妒心朋友的方法。‎ ‎  [答案] C ‎  ‎ ‎(第2篇)‎ ‎  Do American children still learn handwriting in school? In the age of the keyboard, some people seem to think handwriting lessons are on the way out. 90% of teachers say they are required to teach handwriting.But studies have yet to answer the question of how well they are teaching it. One study published this year found that about three out of every four teachers say they are not prepared to teach handwriting. Some teachers are teaching handwriting by ‎ providing instruction for 10 15 minutes a day, and then other teachers who basically teach it for 60 to 70 minutes a day - which really for handwriting is pretty much.‎ ‎  Many adults remember learning that way - by copying letters over and over again. Today' s thinking is that short periods of practice are better. Many experts also think handwriting should not be taught by itself.Instead, they say it should be used as a way to get students to express ideas. After all, that is why we write.‎ ‎  Handwriting involves two skills. One is legibility,_which means forming the letters so they can be read. The other is fluency - writing without having to think about it. Fluency continues to develop up until high school.‎ ‎  But not everyone masters these skills. Teachers commonly report that about onefourth of their kids have poor handwriting. Some people might think handwriting is not important any more because of computers and voice recognition programs.‎ ‎  But Steve Graham at Vanderbilt says word processing is rarely done in elementary school, especially in the early years. American children traditionally first learn to print, and then to write in cursive, which connects the letters. But guess what we learned from a spokeswoman for the College Board, which administers the SAT college admission test. More than 75 percent of students choose to print their essay on the test rather than write in cursive.‎ ‎  [语篇解读] 文章介绍了在键盘普及的年代美国的书法教学情况。‎ ‎  1.We can learn ________ from Paragraph 1.‎ ‎  A. teaching handwriting is a basic requirement in teaching job ‎  B. most teachers prefer to teach handwriting ‎  C. teachers spend little time in teaching handwriting ‎  D. a keyboard has taken the place of the handwriting entirely ‎  [解析] 推理判断题。根据第一段中"90% of teachers say they are required to teach handwriting"可知书法教学是基本的教学要求,A项为正确推论。根据"One study published this year found that about three out of every four teachers say they are not prepared to teach handwriting"可知,很多老师不愿意教书法课,B项错误;根据第一段末句可知C项错误;根据全段判断,只是说键盘普及使一些人认为书法渐渐过时了,D项错误。‎ ‎  [答案] A ‎  2.Which of the following is WRONG for traditional handwriting in the USA?‎ ‎  A. The students are taught by practicing a long period.‎ ‎  B. The letters are repeated many times.‎ ‎  C. Handwriting includes two skills.‎ ‎  D. To write in cursive is taught first.‎ ‎  [解析] 细节认定题。根据第二段可知,传统的书法教学重复很多遍并且持续时间长,因此A、B两项内容正确。根据第三段第一句可知C项内容正确;根据最后一段第二句可知D项内容错误。‎ ‎  [答案] D ‎  3.The underlined word "legibility" in Paragraph 3 means ________.‎ ‎  A. easy to read B. complex ‎  C. unexpected D. unreadable ‎  [解析] 推测词义题。根据下文"which means forming the letters so they can be read"可推测画线词的意思是"清楚,易读"。‎ ‎  [答案] A ‎  4.The best title for the passage is ________.‎ ‎  A. How to improve handwriting in school ‎  B. Right or wrong: the death of handwriting ‎  C. Handwriting involves two skills ‎  D. Handwriting lessons are on the way out ‎  [解析] 标题归纳题。作者对美国书法教学的现状作了叙述,没有自己的主观评论,故选择B。A项没有涉及,C项只是其中一部分内容,D项只是一部分人的观点。‎ ‎  [答案] B ‎  5.The author' s attitude towards whether still to learn handwriting in school is________.‎ ‎  A. negative B. objective ‎  C. critical D. optimistic ‎  [解析] 态度判断题。根据全文的叙述,作者只是将美国书法教学的现状和不同人的观点呈现出来,并没有发表自己的见解,因此作者持客观的态度。[答案] B ‎  [长难句解读] One study published this year found that about three out of every four teachers say they are not prepared to teach handwriting.今年公布的一项研究发现,大约每四个教师中就有三个说他们不准备教书法了。published this year为过去分词短语作后置定语,found后面为宾语从句,其中又含有一个宾语从句they are not prepared to teach handwriting。‎ ‎(第3篇)‎ ‎  A Southampton University team found that people who were vegetarians by 30 had recorded five IQ points higher on average at the age of 10. Researchers said it could explain why people with a higher IQ were healthier as a vegetarian diet was linked to lower heart disease and obesity rates. The study of 8,179 people was reported in the British Medical Journal.‎ ‎  Twenty years after the IQ tests were carried out in 1970, 366 of the participants said they were vegetarians - although more than 100 reported eating either fish or chicken.‎ ‎  Men who were vegetarians had an IQ score of 106, compared with 101 for non-vegetarians; while female vegetarians averaged 104, compared with 99 for non-vegetarians. There was no difference in the IQ scores, between strict vegetarians and those who said they were vegetarians but reported eating fish or chicken.‎ ‎  Researchers said the findings were partly related to better education and higher class, but it remained statistically significant after adjusting for these factors.‎ ‎  Vegetarians were more likely to be female, to be of higher social class and to have higher academic or vocational qualifications than non-vegetarians. However, these differences were not reflected in their annual income, which was similar to that of non-vegetarians.‎ ‎  Lead researcher Catharine Gale said, "The findings that children with greater intelligence are more likely to report being vegetarians as adults, together with the evidence on the potential benefits of a vegetarian diet on heart health, may help to explain why a higher IQ in childhood or adolescence is linked with a reduced risk of coronary heart disease in adult life.‎ ‎  But Dr Frankie Phillips of the British Dietetic Association said,"It_ is_ like _the _chicken _and _egg. Do people become vegetarians because they have a very high IQ or is it just that they are clever enough to be more aware of health issues?"‎ ‎  [语篇解读] 研究表明,素食和高智商有一定的关系。‎ ‎  1.What' s the result of the research mentioned in the text?‎ ‎  A. Intelligent children are more likely to become vegetarians later in life.‎ ‎  B. Children with a higher IQ are less likely to have heart disease later in life.‎ ‎  C. Intelligent children tend to belong to higher social class later in life.‎ ‎  D. Children with a healthier heart tend to have a higher IQ later in life.‎ ‎  [解析] 细节理解题。根据第一段中的"people who were vegetarians by 30 had recorded five IQ points higher on average at the age of 10"可知A项正确。根据"it could explain why..."可知这一结果可帮助解释B项中的现象,但B项并非这次研究的结果。故本题选A。‎ ‎  [答案] A ‎  2.It was found in the research that________.‎ ‎  A. most of the participants became vegetarians 20 years after the IQ tests were carried out ‎  B. vegetarians who ate fish or chicken were of similar intelligence with strict vegetarians ‎  C. female vegetarians were more likely to have higher annual income than non-vegetarians ‎  D. vegetarians were more likely to have higher annual income than non-vegetarians ‎  [解析] 细节理解题。根据第三段中的"There was no difference in the IQ scores between strict vegetarians and those who said they were vegetarians but reported eating fish or chicken"可知答案。‎ ‎  [答案] B ‎  3.Catharine Gale talked about "being vegetarians" in a(n)________way.‎ ‎  A. doubtful B. favorable ‎  C. negative D. objective ‎  [解析] 推理判断题。根据第六段可知Catharine Gale只是客观地讲述研究结果,并没有表示自己的立场。故选D项。‎ ‎  [答案] D ‎  4.What does the underlined sentence in the last paragraph mean?‎ ‎  A. Intelligence is linked to not just being a vegetarian but to many factors.‎ ‎  B. The rate of getting heart disease is linked to your lifestyle.‎ ‎  C. The link between a high IQ and being a vegetarian is still uncertain.‎ ‎  D. The link between a healthy heart and diet remains to be proved.‎ ‎  [解析] 推理判断题。文章最后一句的意思是:人们成为素食者是因为他们智商高还是因为他们足够聪明而更注意健康问题?由此可见Dr Frankie Phillips认为素食和高智商的因果关系尚未明确,就像弄不清楚是因为有蛋才有鸡,还是因为有鸡才有蛋一样。故C项正确。‎ ‎  [答案] C ‎  5.What' s the best title for the text?‎ ‎  A. Get more IQ points!‎ ‎  B. Be a vegetarian, please!‎ ‎  C. Vegetarian diet cuts heart risk ‎  D. A high IQ is linked to being a vegetarian ‎  [解析] 标题概括题。本文介绍了一项研究结果:素食与高智商有关,故D项正确。作者并未提出建议,所以A、B两项错误;文章中虽然提到了心脏病,但不是文章的主旨,故C项错误。‎ ‎  [答案] D ‎  [长难句解读] (1) A Southampton University team found that people who were vegetarians by 30 had recorded five IQ points higher on average at the age of 10.‎ ‎  本句是含有定语从句的复合句。who were vegetarians by 30是定语从句,修饰先行词people。‎ ‎  (2)There was no difference in the IQ scores between strict vegetarians and those who said they were vegetarians but reported eating fish or chicken.‎ ‎  There was no difference in the IQ scores between strict vegetarians and those是主句,who said they were vegetarians but reported eating fish or chicken是定语从句,修饰先行词those。‎ ‎   (第4篇)‎ ‎  The hole in the Earth's ozone layer (臭氧层) has until now protected Antarctica from the worst effects of global warming. But scientists have warned that as the hole closes up in the next few decades, temperatures on the continent could rise by around 3 ℃ on average, with melting ice contributing to a global sea level increase of up to 1.4 metres.‎ ‎  In the past decades the western Antarctic has seen rapid ice loss as the world has warmed, but the other parts of the continent have, paradoxically, been cooling, resulting in a 10% increase in ice in the seas around the region. This is because the hole in the ozone layer has increased cold winds in Antarctica, making much of the continent surface colder than usual.‎ ‎  But now that the gasses that cause the ozone hole have been banned, scientists expect the hole to repair itself within the next 50 to 60 years. By then the cooling effect will have faded ‎ out and the Antarctic will face the full impact of global warming. This means an increase in average air temperatures of around 3 ℃ and a reduction in sea ice by around a third.‎ ‎  The biggest threat to the continent comes from warming seas. Robert Johnson, a scientist who monitors Antarctica ice sheets, said, "The ice sheets in Antarctica are hundreds of metres thick. But once warm ocean waters start flowing underneath, the ice will begin thinning and could break up very quickly." Thinning ice sheets cause ice to break away from the continent and to melt even faster. Escaping ice from western Antarctica has already resulted in a 10% rise in global sea level in recent decades.‎ ‎  Johnson believes that international action to reduce global warming is required immediately or it may be too late. "Everything is connected - Antarctica may be a long way away but it is an important part of the Earth's system," said Johnson. "It contains 90% of the world's ice, 70% of the world's fresh water and that is enough, if it melts completely, to raise sea levels by 63 metres."‎ ‎  Even in a worse-case situation scientists don't expect the ice to entirely disappear, but predict that, because of the melting ice sheets, average sea level rise will be around 1.4 metres higher by the end of the century.‎ ‎  1. The underlined word "paradoxically" (in Paragraph 2) most probably means"________."‎ ‎  A. rapidly B. approximately ‎  C. contradictorily D. apparently ‎  [答案] C ‎  2. What is the effect of the hole in the ozone layer on Antarctica?‎ ‎  A. It is causing the ice to melt faster.‎ ‎  B. It is making much of the continent colder.‎ ‎  C. It is making the effects of global warming in the region worse.‎ ‎  D. It is reducing the amount of water in Antarctica.‎ ‎  [答案] B ‎  3. What do scientists think is the biggest danger facing Antarctica?‎ ‎  A. Rising sea levels.‎ ‎  B. Warming sea water temperature.‎ ‎  C. Water pollution.‎ ‎  D. Growing ice sheets.‎ ‎  [答案] B ‎  4. Which of the following is true according to the passage?‎ ‎  A. Antarctica is currently experiencing the full effects of global warming.‎ ‎  B. The average temperature has increased by 3 ℃ in recent decades.‎ ‎  C. Antarctica contains most of the world's fresh water.‎ ‎  D. Ten percent of Antarctica's ice has already been lost.‎ ‎  [答案] C ‎  5. The best title for the passage is________.‎ ‎  A. Our planet in danger ‎  B. Antarctica melting away ‎  C. Action plan to save Antarctica ‎  D. Let's save the ozone layer ‎  [答案] B ‎  ‎ ‎(第5篇)‎ ‎  We bet that on cold wintry days, many of you love to stay in your warm home and, every now and then,come out into the kitchen for a snack. Unfortunately, plenty of creepy-crawly critters (爬行生物) like to do the same thing!‎ ‎  Winter is the time when bugs (虫子) invade your house without an invitation. The season can be tough for such creatures. In winter the air is cold, the ground is hard and many trees have no leaves. So bugs do what they have to do to survive.‎ ‎  Monarch butterflies head south to warmer climates. Ants crowd in deep underground colonies and eat food they have been storing all year. Many insects go into a deep sleep called diapauses. There' re different kinds of diapauses, but all are similar to hibernation, a time when bigger animals become inactive in the cold. Insects go into an inactive period, too, but it often isn' t when the temperature drops.‎ ‎  They rely on more dependable signals in the environment. For example, many insects can tell how much sunlight there' s each day. They use that to tell themselves when to shut down. Bugs are cold-blooded, meaning that their inside temperature is the same as the outside. They can' t move much when it gets below 40 degrees Fahrenheit. So they search for any warm place.‎ ‎  They' re looking for protection. These guys have been doing this for 300 million years, so they don' t really know they' re coming into your house. The home is a recent event in terms of their evolutionary behavior. They enter through tiny cracks or come in unnoticed on your clothes or shoes. Remember that they may be invading your homes for warmth and food, but they don't care about humans.‎ ‎  [语篇解读] 本文主要介绍的是:为什么虫子在冬天会入侵人类住所。‎ ‎  1. What is the purpose of the first paragraph?‎ ‎  A. To point out that humans like to stay at home in winter.‎ ‎  B. To mean that humans and bugs have the same living habits.‎ ‎  C. To mean that bugs will invade humans' houses for their winter.‎ ‎  D. To put forward the idea that bugs are not welcome in winter.‎ ‎  [解析] 推理判断题。根据文章第一段可知,虫子和人类一样,也会喜欢待在温暖的屋里,再结合文章的后面几段可知,本段暗示虫子在冬天会入侵人类住所。‎ ‎  [答案] C ‎  2.According to the text, what is diapauses?‎ ‎  A. It is the same as the animals' hibernation.‎ ‎  B. It often appears in warm areas all the year.‎ ‎  C. It is done to keep bugs active in winter.‎ ‎  D. It is a deep sleep similar to hibernation.‎ ‎  [解析] 细节理解题。根据文章第三段可知答案为D。‎ ‎  [答案] D ‎  3.What often decides bugs' diapauses?‎ ‎  A. The lower local temperature.‎ ‎  B. The amount of sunlight.‎ ‎  C. Kinds of environmental signals.‎ ‎  D. The insects' inside temperature.‎ ‎  [解析] 细节理解题。根据文章第四段的"They rely on more dependable signals in the environment"可知,虫子的"diapauses"是由多种环境可依赖性因素决定的,不一定全靠温度或者光照。‎ ‎  [答案] C ‎  4.According to the text, bugs invade humans' homes to________.‎ ‎  A. attack humans ‎  B. look for enough food ‎  C. seek for protection ‎  D. show their evolutionary results ‎  [解析] 细节理解题。根据文章最后一段可知,虫子进屋是为了寻求保护,把自己置身于安全形势之下,而不仅仅是为了寻求食物。‎ ‎  [答案] C ‎  5.What would be the best title for the text?‎ ‎  A. Bugs' life on cold wintry days ‎  B. Why do bugs invade your home in winter?‎ ‎  C. Good relations between humans and bugs ‎  D. What does diapauses mean?‎ ‎  [解析] 标题概括题。整篇文章主要告诉读者的是:在冬天里,为什么虫子会入侵人类的住所。‎ ‎  [答案] B ‎  [长难句解读] (1)There're different kinds of diapauses,but all are similar to hibernation,a time when bigger animals become inactive in the cold.‎ ‎  此句是but连接的并列复合句,but后的部分中a time是hibernation的同位语,而且后面接了when引导的定语从句。‎ ‎  (2)Bugs are cold-blooded,meaning that their inside temperature is the same as the outside.‎ ‎  句中的meaning相当于which means;meaning后接that引导的宾语从句,在此宾语从句中含有the same as结构。‎ ‎   (第6篇)‎ ‎  July 21st,2007 was a typical English summer's day - it rained for 24 hours! As usual, I rushed home from work at midday to check on the house. Nothing was amiss. By the time I left work at 5 pm, however,the road into our village was flooded. Our house had never been flooded but, as I opened the front door, a wave of water greeted me. Thank God the kids weren't with me, because the house was 5 feet deep in water. We lost everything downstairs. And the plaster had to be torn off the walls, ceilings pulled down.‎ ‎  At first we tried to push on through. We didn't want to move the children out of home, so we camped upstairs. We put a sheet of plastic across the floor to protect us from the damp. But after three months, we felt very sick, so we moved to a wooden house in a park. The house was small, but at first we were all just delighted to be in a new place. Unfortunately, things took longer than expected and we were there for 10 months. The life there was inconvenient. What surprised me most was how much I missed being part of a community (社区). We had lived in a friendly village with good neighbours,and I'd never thought how much I'd miss that.‎ ‎  Although our situation was very bad, it's difficult to feel too sorry for yourself when you look at what's happening elsewhere. I watched a news report about floods in Northern India and thought,"We didn't have a straw hut(茅草房) that was swept away, and our house is still standing. We're lucky."‎ ‎  We moved back home in August. With December coming,there's still reconstruction work to be done, so it's difficult to prepare for Christmas. But I can't wait-I'm going to throw a party for our friends in the village to say thanks for their support. This year, I won't need any gifts - living away from home for months has made me realize how little we actually need or miss all our possessions. Although we are replacing things, there's really no rush-we have our home back, and that's the main thing.‎ ‎  [语篇解读] 洪水给生活带来了困难,但在灾害面前,也能获得更多对人生的认识。‎ ‎  5.What does the underlined word "amiss" in the first paragraph mean?‎ ‎  A. Wrong.        B. Missing.‎ ‎  C. Right. D. Found.‎ ‎  [解析] 猜测词义题。从第一段中"As usual,I rushed home from work..."可知作者还是像往常那样赶回家,因此没有什么不正常的,故选择A。‎ ‎  [答案] A ‎  6.It can be inferred from the text that the author________.‎ ‎  A. was sick of staying upstairs ‎  B. cared much about her children ‎  C. could not stand living in a wooden house ‎  D. did not deal well with her family affairs during the flood ‎  [解析] 推理判断题。从第一段第五句Thank God the kids weren't with me,...可以看出作者最关心(在意)孩子。‎ ‎  [答案] B ‎  7.Why does the author say that they were lucky in the third paragraph?‎ ‎  A. Because her situation was not serious.‎ ‎  B. Because many other places were flooded.‎ ‎  C. Because she had been to Northern India.‎ ‎  D. Because some others suffered even more.‎ ‎  [解析] 事实细节题。当作者看到印度北部遭受的洪水灾难比自己的更惨痛的报道时,深感自己应该庆幸。‎ ‎  [答案] D ‎  8.What does the author mainly want to express by telling her story?‎ ‎  A. She valued human feelings more than before.‎ ‎  B. She realized she almost didn't need possessions.‎ ‎  C. She found Christmas gifts no longer badly needed.‎ ‎  D. She thought her own home was the most important.‎ ‎  [解析] 主旨大意题。从最后一段对即将来临的圣诞节的准备"...there's really no rush-we have our home back,and that's the main thing."可知作者通过这次洪水认识到了她比以前更加珍惜人们之间的情感,故选择A。‎ ‎  ‎ ‎(第7篇)‎ ‎  ‎ ‎  14 days from just £ 2,090 pp ‎  Fully inclusive from the UK ‎  Price covers: International airfares, departure taxes, fuel charges, local transportation, accommodation, all meals, entrance fees, guides, daily tours and visas for UK citizens.‎ ‎  ◆Days 1 3: UK Shanghai ‎  Fly to the great city of Shanghai and in the evening sample traditional Shanghai food. Visit the beautiful Yu Gardens, Old Town, Shanghai Museum, cross the Great Nanpu Bridge and tour the Pudong area. Also explore Xintiandi with its 1920' s style Shikumen buildings and end your stay in Shanghai with an amazing Huangpu River evening tour.‎ ‎  ◆Days 4 7: Shanghai Yangtze River Tour ‎  Fly to Yichang and change (approx: one hour)to board your Yangtze River ship for the next four nights. Enjoy a tour of the Three Gorges Dam (三峡大坝)before sailing on the grand ‎ Yangtze River, passing through the impressive Three Gorges. We take a side trip to the Lesser Three Gorges or travel up the Shennong Stream in a peapod boat and enjoy various shore trips along the way.‎ ‎  ◆Day 8: Chongqing Chengdu ‎  Get off in Chongqing and drive to Chengdu for an overnight stay.‎ ‎  ◆Days 9 10: Chengdu Xi'an ‎  Visit the famous Panda Reserve to see the lovely animals.We then fly to the historic city of Xi'an for two nights' stay and enjoy traditional Shui Jiao. Next day, explore one of the most important discoveries of the 20th Century - the Terracotta Warriors(兵马俑) ,followed by the ancient City Wall and a performance of Tang Dynasty dancing.‎ ‎  ◆Days 11 13:Xi'an Beijing ‎  Visit Little Wild Goose Pagoda and see the ancient objects at the wellknown Shaanxi Provincial Museum before walking through the lively Muslim Quarter to see the Great Mosque.Later fly to Beijing for three nights' stay and try Peking Duck.During our stay in Beijing, we stroll through Tiananmen Square to the Forbidden City, the largest and best preserved collection of ancient buildings in China, and visit the Summer Palace. Next day we take a walk on the Great Wall, tour the unique Temple of Heaven and enjoy an attractive Chinese Acrobatic Show.‎ ‎  ◆Day 14: Beijing UK ‎  Fly back to the UK, arriving home later the same day filled with happy memories.‎ ‎  [语篇解读] 本文是一则广告,介绍到中国旅游的一些日程安排。‎ ‎  1.The underlined word "sample" in the passage probably means "________".‎ ‎  A. buy         B. reserve ‎  C. taste D. make ‎  [解析] 词义猜测题。该句说的是飞抵上海,在晚上应该是"品尝"传统的上海食物。其他选项明显不符合句意。‎ ‎  [答案] C ‎  2.The first and last scenic spots to be visited in Xi'an are________.‎ ‎  A. the Terracotta Warriors and the Great Mosque ‎  B. the Terracotta Warriors and Shaanxi Provincial Museum ‎  C. Little Wild Goose Pagoda and the Great Mosque ‎  D. Little Wild Goose Pagoda and the Muslim Quarter ‎  [解析] 细节理解题。依据Days 910:Chengdu-Xi'an这个部分可知,在西安最先看的景点是兵马俑;再依据Days 1113:Xi'an-Beijing可知,离开西安前看的景点是the Great Mosque(清真寺)。‎ ‎  [答案] A ‎  3.Which of the following is TRUE according to the ad?‎ ‎  A. Tbe tourists will have to pay extra for fuel and meals.‎ ‎  B. The tourists will visit the 1920's Shikumen buildings in Beijing.‎ ‎  C. The tourists will take a side trip to the Three Gorges Dam during the tour.‎ ‎  D. The tourists will stay in Beijing for three nights before leaving for the UK.‎ ‎  [解析] 细节理解题。依据Days 1113:Xi'an-Beijing这个部分的内容可知,游客要在北京呆三个晚上,所以选D。‎ ‎  [答案] D ‎  4.The ad is mainly intended to________.‎ ‎  A. encourage the British to travel in China ‎  B. attract the British to traditional Chinese food ‎  C. offer services of booking air tickets to tourists ‎  D. provide the British with a better understanding of China ‎  [解析] 写作意图题。这是一则广告,把旅游的日程安排以及中国的文化古迹、人文地理及美食介绍给英国游客,自然目的是A。‎ ‎  [答案] A ‎(第8篇)‎ ‎  Next time a customer comes to your office, offer him a cup of coffee. And when you're doing your holiday shopping online, make sure you're holding a large glass of iced tea. The physical sensation (感觉) of warmth encourages emotional warmth, while a cold drink in hand prevents you from making unwise decisions - those are the practical lesson being drawn from recent research by psychologist John A. Bargh.‎ ‎  Psychologists have known that one person' s perception (感知) of another' s "warmth" is a powerful determiner in social relationships. Judging someone to be either "warm" or "cold" is a primary consideration, even trumping evidence that a "cold" person may be more capable. Much of this is rooted in very early childhood experiences, Bargh argues, when babies' conceptual sense of the world around them is shaped by physical sensations, particularly warmth and coldness.Classic studics by Harry Harlow, published in 1958, showed monkeys preferred to stay close to a cloth "mother" rather than one made of wire, even when the wire "mother" carried a food bottle. Harlow's work and later studies have led psychologists to stress the need for warm physical contact from caregivers to help young children grow into healthy adults with normal social skills.‎ ‎  Feelings of "warmth" and "coldness" in social judgments appear to be universal.Although no worldwide study has been done, Bargh says that describing people as "warm", or "cold" is common to many cultures, and studies have found those perceptions influence judgment in dozens of countries.‎ ‎  To test the relationship between physical and psychological warmth, Bargh conducted an experiment which involved 41 college students. A research assistant who was unaware of the study's hypotheses (假设) , handed the students either a hot cup of coffee, or a cold drink, to hold ‎ while the researcher filled out a short information form. The drink was then handed back. After that, the students were asked to rate the personality of"Person A"based on a particular description. Those who had briefly held the warm drink regarded Person A as warmer than those who had held the iced drink.‎ ‎  "We are grounded in our physical experiences even when we think abstractly," says Bargh.‎ ‎  [语篇解读] 最近的心理学研究发现,人们对"温暖"或者"寒冷"的感受可以改变人们的情感。‎ ‎  5.According to Paragraph 1, a person' s emotion may be affected by________.‎ ‎  A. the visitors to his office ‎  B. the psychology lessons he has ‎  C. his physical feeling of coldness ‎  D. the things he has bought online ‎  [解析] 细节理解题。由文章第一段的第三句话可知,温暖的感觉激发温暖的情感,而冷饮在手则会阻止你做出不明智的决定,因此可以说一个人的情感可能会受到冷的感觉的影响。‎ ‎  [答案] B ‎  6.The author mentions Harlow' s experiment to show that________.‎ ‎  A. adults should develop social skills ‎  B. babies need warm physical contact ‎  C. caregivers should be healthy adults ‎  D. monkeys have social relationships ‎  [解析] 细节理解题。在试验中,猴子喜欢去靠近布制的"妈妈"而不喜欢"金属线"做的"妈妈"。这使心理学家们强调养育者应该让孩子们多接触一些令人感觉温暖的东西,以帮助孩子们成长为有正常社交技能的健康的成年人,故B项正确。‎ ‎  [答案] B ‎  7.In Bargh' s experiment, the students were asked to________.‎ ‎  A. evaluate someone' s personality ‎  B. write down their hypotheses ‎  C. fill out a personal information form ‎  D. hold coffee and cold drink alternatively ‎  [解析] 细节理解题。由文中倒数第二段中的"After that,the students were asked to rate the personality of'Person A'based on a particular description."可知选A。在试验中,学生被要求评价某人的性格。‎ ‎  [答案] A ‎  8.We can infer from the passage that________.‎ ‎  A. abstract thinking does not come from physical experiences ‎  B. feelings of warmth and coldness are studied worldwide ‎  C. physical temperature affects how we see others ‎  D. capable persons are often cold to others ‎  [解析] 推理判断题。全文都在论述人们对"温暖"或"寒冷"的感觉可以影响人们对事物的评价。因此,物理温度会影响我们看待他人的方式。因此选C。‎ ‎  [答案] C ‎  9.What would be the best title for the passage?‎ ‎  A. Drinking for Better Social Relationships ‎  B. Experiments of Personality Evaluation ‎  C. Developing Better Drinking Habits ‎  D. Physical Sensations and Emotions ‎  [解析] 主旨大意题。本文主要介绍的是人们对物理温度的感觉对人们情绪的影响,D项概括性比较强。‎ ‎  [答案] D ‎(第9篇)‎ ‎  In ancient Egypt ,the pharaoh(法老) treated the poor message runner like a prince when he arrived at the palace, if he brought good news. However, if the exhausted runner had the misfortune to bring the pharaoh unhappy news, his head was cut off.‎ ‎  Shades of that spirit spread over today' s conversations.Once a friend and I packed up some peanut butter and sandwiches for an outing. As we walked lightheartedly out the door,picnic basket in hand, a smiling neighbor looked up at the sky and said, "Oh boy, bad day for a picnic. The weatherman says it' s going to rain." I wanted to strike him on the face with the peanut butter and sandwiches. Not for his stupid weather report, for his smile.‎ ‎  Several months ago I was racing to catch a bus.As I breathlessly put my handful of cash across the Greyhound counter, the sales agent said with a broad smile, "Oh that bus left five minutes ago." Dreams of headcutting!‎ ‎  It' s not the news that makes someone angry. It' s the unsympathetic attitude with which it' s delivered. Everyone must give bad news from time to time, and winning professionals do it with the proper attitude. A doctor advising a patient that she needs an operation does it in a caring way. A boss informing an employee he didn' t get the job takes on a sympathetic tone.Big winners know, when delivering any bad news, they should share the feeling of the receiver.‎ ‎  Unfortunately, many people are not aware of this. When you are tired from a long flight, has a hotel clerk cheerfully said that your room isn' t ready yet? When you had your heart set on the toast beef, has your waiter merrily told you that he just served the last piece? It makes you as traveler or diner want to land your fist right on their unsympathetic faces.‎ ‎  Had my neighbor told me of the upcoming rainstorm with sympathy, I would have appreciated his warning.Had the Greyhound salesclerk sympathetically informed me that my bus had already left, I probably would have said, "Oh, that ' s all right. I'll catch the next one.‎ ‎ "Big winners, when they bear bad news, deliver bombs with the emotion the bombarded (被轰炸的) person is sure to have.‎ ‎  [语篇解读] 当我们告诉别人不好的消息时,我们应该以同情的表情和语气来表达。‎ ‎  1.In Paragraph 1 ,the writer tells the story of the pharaoh to________.‎ ‎  A. make a comparison     B. introduce a topic ‎  C. describe a scene D. offer an argument ‎  [解析] 逻辑推理题。从第一段可知,作者用"法老的故事"来引出文章的话题,所以答案选B。‎ ‎  [答案] B ‎  2.In the writer' s opinion, his neighbor was________.‎ ‎  A. friendly B. warmhearted ‎  C. not considerate D. not helpful ‎  [解析] 事实细节题。文章中第二段,作者的邻居高兴地向作者传达一个坏消息,他没有考虑作者的感受,由此可知作者的邻居很不会体贴人,考虑事情不周全。‎ ‎  [答案] C ‎  3.From "Dreams of headcutting!" ( Paragraph 3 ), we learn that the writer________.‎ ‎  A. was mad at the sales agent ‎  B. was reminded of the cruel pharaoh ‎  C. wished that the sales agent would have bad dreams ‎  D. dreamed of cutting the sales agent' s head that night ‎  [解析] 逻辑推理题。从文章第三段推断可知,作者对售票员的这种行为非常愤怒,所以答案选A。‎ ‎  [答案] A ‎  4.What is the main idea of the text?‎ ‎  A. Delivering bad news properly is important in communication.‎ ‎  B. Helping others sincerely is the key to business success.‎ ‎  C. Receiving bad news requires great courage.‎ ‎  D. Learning ancient traditions can be useful.‎ ‎  [解析] 主旨大意题。本文告诉我们"在日常交流中,当向别人传达坏消息时应该委婉地表达出来,以照顾到对方的感受,这在日常交流中是非常重要的"。由此可知答案为A项。‎ ‎  [答案] A ‎(第10篇)‎ ‎  A rainforest is an area covered by tall trees with the total high rainfall spreading quite equally through the year and the temperature rarely dipping below 16℃.Rainforests have a great effect on the world environment because they can take in heat from the sun and adjust the climate. Without the forest cover, these areas would reflect more heat into the atmosphere,warming the rest of the world.Losing the rainforests may also influence wind and rainfall patterns, potentially causing certain natural disasters all over the world.‎ ‎  In the past hundred years, humans have begun destroying rainforests in search of three major resources (资源): land for crops, wood for paper and other products, land for raising farm animals. This action affects the environment as a whole. For example,a lot of carbon dioxide(二氧化碳)in the air comes from burning the rainforests. People obviously have a need for the resources we gain from cutting trees but we will suffer much more than we will benefit.‎ ‎  There are two main reasons for this. Firstly, when people cut down trees, generally they can only use the land for a year or two. Secondly, cutting large sections of rainforests may provide a good supply of wood right now, but in the long run it actually reduces the world' s wood supply.‎ ‎  Rainforests are often called the world's drug store. More than 25% of the medicines we use today come from plants in rainforests. However, fewer than 1% of rainforest plants have been examined for their medical value. It is extremely likely that our best chance to cure diseases lies somewhere in the world's shrinking rainforests.‎ ‎  [语篇解读] 文章论述了热带雨林的重要作用以及对环境的影响。‎ ‎  5.Rainforests can help to adjust the climate because they________.‎ ‎  A. reflect more heat into the atmosphere ‎  B. bring about high rainfall throughout the world ‎  C. rarely cause the temperature to drop lower than 16℃‎ ‎  D. reduce the effect of heat from the sun on the earth ‎  [解析] 因果判断题。由文章第一段内容可以得出答案。‎ ‎  [答案] D ‎  6.What does the word "this" underlined in the third paragraph refer to?‎ ‎  A. We will lose much more than we can gain.‎ ‎  B. Humans have begun destroying rainforests.‎ ‎  C. People have a strong desire for resources.‎ ‎  D. Much carbon dioxide comes from burning rainforests.‎ ‎  [解析] 由上下文内容可知,A项正确。‎ ‎  [答案] A ‎  7.It can be inferred from the text that________.‎ ‎  A. we can get enough resources without rainforests ‎  B. there is great medicine potential in rainforests ‎  C. we will grow fewer kinds of crops in the gained land ‎  D. the level of annual rainfall affects wind patterns ‎  [解析] 推测判断题。由文章最后一段内容即可得出答案。‎ ‎  [答案] B ‎  8.What might be the best title for the text?‎ ‎  A. How to Save Rainforests ‎  B. How to Protect Nature ‎  C. Rainforests and the Environment ‎  D. Rainforests and Medical Development ‎  [解析] 主旨大意题。整篇文章主要论述了热带雨林的重要作用以及砍伐热带雨林对环境所造成的影响。‎ ‎  [答案] C ‎(第11篇)‎ ‎  What is eBay? The simple answer is that it is a global trading platform where nearly anyone can trade practically anything. People can sell and buy all kinds of products and goods,including cars, movies and DVDs, sporting goods, travel tickets, musical instruments, clothes and shoes - the list goes on and on.‎ ‎  The idea came from Pierre Omidyar, who was born in Paris and moved to Washington when he was still a child. At high school, he became very interested in computer programming and after graduating from Tufts University in 1988, he worked for the next few years as a computer engineer. In his free time he started eBay as a kind of hobby, at first offering the service free by word of mouth. By 1996 there was so much traffic on the website that he had to upgrade(升级) and he began charging a fee to members. Joined by a friend, Peter Skoll, and in 1998 by his capable CEO, Meg Whitman, he_has_never_looked_back. Even in the great.com crashes of the late 1990s, eBay has gone from strength to strength. It is now one of the ten most visited online shopping websites on the Internet.‎ ‎  eBay sells connections, not goods, putting buyer and seller into contact with each other. All you have to do is take an ephoto, write a description, fill out a sales form and you are in business; the world is your market place. Of course for each item (商品)sold eBay gets a percentage and that is a great deal of money. Every day there are more than sixteen million items listed on eBay and eighty percent of the items are sold.‎ ‎  [语篇解读] 文章向我们介绍了eBay网的一些情况。‎ ‎  1.We learn from the text that eBay provides people with ________.‎ ‎  A. a way of buying and selling goods ‎  B. a website for them to upgrade ‎  C. a place to exhibit their own photos ‎  D. a chance to buy things at low prices ‎  [解析] 根据第一段内容可知。‎ ‎  [答案] A ‎  2.Why did Pierre create eBay after graduating from university?‎ ‎  A. For fun.‎ ‎  B. To make money.‎ ‎  C. For gathering the engineers.‎ ‎  D. To fulfill a task of his company.‎ ‎  [解析] 由第二段第三句话可知。‎ ‎  [答案] A ‎  3.From "he has never looked back "in Paragraph 2 we learn that Pierre________.‎ ‎  A. did not feel lonely ‎  B. was always hopeful ‎  C. did not think about the past ‎  D. became more and more successful ‎  [解析] Peter由原来的业余爱好开始,逐步走向成功,创建了世界知名网站。‎ ‎  [答案] D ‎  4.How does eBay make money from its website?‎ ‎  A. By bringing sellers together.‎ ‎  B. By charging for each sale.‎ ‎  C. By listing items online.‎ ‎  D. By making ephotos.‎ ‎  [解析] 由最后一段第一句话可知,这个网站通过提供交易平台,然后收取适当的服务费。因此B项正确。‎ ‎  [答案] B ‎(第12篇)‎ ‎  I was waiting for a phone call from my agent. He had left a message the night before, telling me that my show was to be cancelled. I called him several times, but each time his secretary told me that he was in a meeting and that he would call me later. So I waited and waited, but there was still no call. Three hours passing by, I became more and more impatient. I was certain that my agent didn't care about my work, and he didn't care about me. I was overcome with that thought. I started to shout at the phone," Let me wait, will you? Who do you think you are?"‎ ‎  At that time I didn't realize my wife was looking on. Without showing her surprise, she rushed in, seized the phone, tore off the wires,and shouted at the phone,"Yeah! Who do you think you are?Bad telephone ! Bad telephone!"And she swept it into the wastebasket.‎ ‎  I stood watching her, speechless. What on earth?‎ ‎  She stepped to the doorway and shouted at the rest of the house, "Now hear this ! All objects in this room - if you do anything to upset my husband, out you go!"‎ ‎  Then she turned to me, kissed me, and said calmly,"Honey,you just have to learn how to take control. "With that, she left the room.‎ ‎  After watching a crazy woman rushing in and out, shouting at everything in sight,I noticed that something in my mood(情绪) had changed. I was laughing. How could I have trouble ‎ with that phone?Her antics_helped me realize I had been driven crazy by small things. Twenty minutes later my agent did call. I was able to listen to him and talk to him calmly.‎ ‎  [语篇解读] "我"对自己的经纪人取消我的表演并且不及时接电话的行为大为恼火,冲电话大吼以泄愤,但接着妻子就此事的一系列异常及夸张的行为让我意识到自己对小事情反应过火,最后我的情绪得到好转,冷静了下来。‎ ‎  5.Why did the author shout at the telephone?‎ ‎  A. He was mad at the telephone.‎ ‎  B. He was angry with his agent.‎ ‎  C. He was anxious about his wife.‎ ‎  D. He was impatient with the secretary.‎ ‎  [解析] 推理判断题。从文中第一段"经纪人要取消我的表演、不接我的电话"等内容可得知我很生气。‎ ‎  [答案] B ‎  6.What did the author's wife do after she heard his shouting?‎ ‎  A. She said nothing.‎ ‎  B. She shouted at him.‎ ‎  C. She called the agent.‎ ‎  D. She threw the phone away.‎ ‎  [解析] 事实细节题。从文中第二段最后一句得知她把电话丢进了垃圾桶里。‎ ‎  [答案] D ‎  7.What made the author laugh?‎ ‎  A. His own behavior.‎ ‎  B. His wife's suggestion.‎ ‎  C. His changeable feelings.‎ ‎  D. His wife's sweet kiss.‎ ‎  [解析] 推理判断题。从文中最后一段中"How could I have trouble with that telephone?...realized I had been driven crazy by small things."得知"我"为自己刚才的行为感到好笑。‎ ‎  [答案] A ‎  8.What does the underlined word" antics" refer to?‎ ‎  A. Smart words.      B. Unusual actions.‎ ‎  C. Surprising looks. D. Anxious feelings.‎ ‎  [解析] 猜测词义题。由文章的最后一段第一句可知妻子行为异常。‎ ‎  [答案] B ‎  [长难句解读] After watching a crazy woman rushing in and out,shouting at everything in sight,I noticed that something in my mood had changed.看着一个疯狂的女人冲进冲出,并冲着她看到的每样东西大喊大叫后,我注意到自己的情绪已不知不觉地发生了变化。"After watching a...in sight"为介词短语作状语,后面的主句中含有一个that引导的宾语从句。‎ ‎(第13篇)‎ ‎  How to Be a Winner ‎  Sir Steven Redgrave ‎  Winner of 5 Olympic Gold Medals ‎  "In 1997 I was found to have developed diabetes(糖尿病).Believing my career(职业生涯) was over, I felt extremely low. Then one of the specialists said there was no reason why I should stop training and competing. That was it-the encouragement I needed. I could still be a winner if I believed in myself. I am not saying that it isn't difficult sometimes. But I wanted to prove to myself that I wasn't finished yet. Nothing is to stand in my way."‎ ‎  Karen Pickering ‎  Swimming World Champion ‎  "I swim 4 hours a day, 6 days a week. I manage that sort of workload by putting it on top of my diary. This is the key to suceess-you can't follow a career in any field without being wellorganized. List what you believe you can achieve. Trust yourself, write down your goals for the day, however small they are, and you'll be a step closer to achieving them."‎ ‎  Kirsten Best ‎  Poet & Writer ‎  "When things are getting hard, a voice inside my head tells me that I can't achieve something. Then,there are other distractions,_such as family or hobbies. The key is to concentrate. When I feel tense, it helps a lot to repeat words such as'calm','peace'or'focus', either out loud or silently in my mind. It makes me feel more in control and increases my confidence. This is a habit that can become second nature quite easily and is a powerful psychological(心理的) tool."‎ ‎  [语篇解读] 本文叙述了三位名人在成功的道路上所经历的不同的事情,一是战胜了病痛,一定克服了精力分散的困难,一是通过坚持不懈努力,最后都取得了胜利。‎ ‎  1.What does Sir Steven Redgrave mainly talk about?‎ ‎  A. Difficulties influenced his career.‎ ‎  B. Specialists offered him medical advice.‎ ‎  C. Training helped him defeat his disease.‎ ‎  D. He overcame the shadow of illness to win.‎ ‎  [解析] 推理判断题。第一部分写了Sir Steven得了糖尿病,但最终克服了病痛拿到了金牌,故D项符合题意。‎ ‎  [答案] D ‎  2.What does Karen Pickering put on top of her diary?‎ ‎  A. Her training schedule.‎ ‎  B. Her daily happenings.‎ ‎  C. Her achievements.‎ ‎  D. Her sports career.‎ ‎  [解析] 事实细节题。第二部分第二句中,it指代前面的句子内容,即她的训练计划。‎ ‎  [答案] A ‎  3.What does the underlined word"distractions" probably refer to?‎ ‎  A. Ways that help one to focus.‎ ‎  B. Words that help one to feel less tense.‎ ‎  C. Activities that turn one's attention away.‎ ‎  D. Habits that make it hard for one to relax.‎ ‎  [解析] 猜测词义题。从下文"The key is to concentrate."可以推出是指分散作家注意力的活动。‎ ‎  [答案] C ‎  4.According to the passage,what do the three people have in common?‎ ‎  A. Courage.         B. Devotion.‎ ‎  C. Hard work. D. Selfconfidence.‎ ‎  [解析] 事实细节题。第一部分中"I could still be a winner if I believed in myself.",第二部分中"Trust yourself..."及第三部分中"It makes me feel more in control and increases my confidence..."都表明自信是他们三个的共同之处。‎ ‎  [答案] D ‎  [长难句解读] Trust yourself,write down your goals for the day,however small they are,and you'll be a step closer to achieving them.相信你自己,不论目标多小,每天把它们写下来,这样你会向实现它们又迈进了一步。"祈使句+and/or+现在将来时句子"结构中,祈使句可转换成由if引导的条件句。本句含有两个祈使句,第二个祈使句又含有一个让步状语从句。‎ ‎(第14篇)‎ ‎  People diet to look more attractive. Fish diet to avoid being beaten up, thrown out of their social group, and getting eaten as a result. That is the fascinating conclusion of the latest research into fish behavior by a team of Australian scientists.‎ ‎  The research team have discovered that subordinate fish voluntarily diet to avoid challenging their larger competitors. "In studying gobies we noticed that only the largest two individuals, a male and female ,had breeding(繁殖) rights within the group, "explains Marian Wong. "All other group members are nonbreeding females, each being 5~10% smaller than its next largest competitor. We wanted to find out how they maintain this precise size separation."‎ ‎  The reason for the size difference was easy to see. Once a subordinate fish grows to within 5 10% of the size of its larger competitor,it causes a fight which usually ends in the smaller goby being driven away from the group. More often than not, the_evicted_fish_is then eaten up.‎ ‎  It appeared that the smaller fish were keeping themselves small in order to avoid challenging the boss fish. Whether they did so voluntarily,by restraining how much they ate, was not clear. The research team decided to do an experiment. They tried to fatten up some of the ‎ subordinate gobies to see what happened. To their surprise,the gobies simply refused the extra food they were offered,clearly preferring to remain small and avoid fights, over having a feast.‎ ‎  The discovery challenges the traditional scientific view of how boss individuals keep their position in a group. Previously it was thought that large individuals simply used their weight and size to threaten their subordinates and take more of the food for themselves,so keeping their competitors small.‎ ‎  While the habits of gobies may seem a little mysterious, Dr.Wong explains that understanding the relationships between boss and subordinate animals is important to understanding how hierarchical (等级的) societies remain stable.‎ ‎  The research has proved the fact that voluntary dieting is a habit far from exclusive to humans. "As yet, we lack a complete understanding of how widespread the voluntary reduction of food intake is in nature," the researchers comment. "Data on human dieting suggests that,while humans generally diet to improve health or increase attractiveness, rarely does it improve longterm health and males regularly prefer females that are fatter than the females' own ideal."‎ ‎  [语篇解读] 人们节食是为了更有魅力。令鱼类专家们惊奇的是有种鱼也要节食。实验发现它们节食可不是为了美,而是为了自我保护。如果身体大小吸引到竞争者的注意,会有被殴打、驱逐,最后被吃掉的危险。‎ ‎  5.When a goby grows to within 510% of the size of its larger competitor, it________.‎ ‎  A. faces danger B. has breeding rights ‎  C. eats its competitor D. leaves the group itself ‎  [解析] 推理判断题。从第三段可得知如果身体大小超过权威的规定范围,会有被殴打、驱逐最后被吃掉的危险。‎ ‎  [答案] A ‎  ‎ ‎  6.The underlined words "the evicted fish" in Paragraph 3 refer to ________.‎ ‎  A. the fish beaten up B. the fish found out ‎  C. the fish fattened up D. the fish driven away ‎  [解析] 猜测词义题。从本段前文可知,体积超标的鱼会被赶出鱼群。接着说到,不仅如此,这些鱼往往会被吃掉。这些鱼一定是指被驱逐的鱼。‎ ‎  [答案] D ‎  7.The experiment showed that the smaller fish________.‎ ‎  A. fought over a feast B. went on diet willingly ‎  C. preferred some extra food D. challenged the boss fish ‎  [解析] 事实细节题。从第四段中的试验可得知这些鱼是自愿节食。‎ ‎  [答案] B ‎  8.What is the text mainly about?‎ ‎  A. Fish dieting and human dieting.‎ ‎  B. Dieting and health.‎ ‎  C. Human dieting.‎ ‎  D. Fish dieting.‎ ‎  [解析] 主旨大意题。文章通篇都是围绕鱼的节食来叙述。故选D。‎ ‎  [答案] D ‎   (第15篇)‎ ‎  Find Which Direction Is South ‎  Do you have a good sense of direction? If not, please take with you a compass. But if you forget to take a compass, you can still find your way.‎ ‎  It' s never a good idea to imagine that the family member who was entrusted(委托) with the job of mapreading actually knows where the family is. You can tell by the slightly confused look on their faces that nothing on the ground seems to match the map. Never mind. The sun is shining and it' s still morning. If you don't know the exact time, you can still find out where south is, but you'll need to be patient.‎ ‎  ①Find a straight stick and put it in the ground in a place where you can mark its shadow.‎ ‎  ②Try to position the stick as vertically(垂直) as you can.You can check this by making a simple plumb line (铅锤线)with a piece of string and weight. You haven' t got any string?OK, use a thread from your clothes with a button tied at the end to act as a weight.‎ ‎  ③Mark the end of the shadow cast by the stick.‎ ‎  ④Wait approximately half an hour and mark the end of the shadow again.‎ ‎  ⑤Keep doing this until you have made several marks.‎ ‎  ⑥The mark nearest the stick will represent the shortest shadow, which is cast at midday, when the sun is highest in the sky and pointing to the exact south.‎ ‎  ⑦Pick a point in the distance along the line between the shortest shadow and the stick.‎ ‎  ⑧That point is south of where you are.‎ ‎  ⑨Now you can turn the map, like you did before, and find which way you should be travelling.‎ ‎  [语篇解读] 你有好的方向感吗?如果没有,那就要带上指南针。如果没带指南针呢?你仍然可以用其他方法找到方向,本文就是教你如何找到方向的。‎ ‎  1.To find the direction, we ought to be patient probably because________.‎ ‎  A. it is not easy to find a proper stick ‎  B. it is not easy to position the stick ‎  C. it takes hours to make the marks ‎  D. it takes about half an hour to make the marks ‎  [解析] 推理判断题。依据文章中的③⑤可以看出,做阴影的标记最费时间,所以要耐心。‎ ‎  [答案] C ‎  2.The passage would probably be most helpful to________.‎ ‎  A. those who draw maps ‎  B. those who get lost ‎  C. those who make compasses ‎  D. those who do experiments ‎  [解析] 推理判断题。依据文章第二段,"我们发现地图上的标记跟身边的不一致",可以知道这是迷路时的现象,所以答案为B。‎ ‎  [答案] B ‎  3.Which of the following pictures best shows the way of finding the direction of south?‎ ‎  ‎ ‎  [解析] 依据文章中⑥的叙述,可知:离树棍最近的那条线代表的阴影最短,证明当时太阳在天空最高,指向的就是最南方,对应的正好是A。‎ ‎  [答案] A ‎  4.The author presents this passage by________.‎ ‎  A. telling an interesting story ‎  B. describing an activity in a lively way ‎  C. testing an idea by reasoning ‎  D. introducing a practical method ‎  [解析] 细节理解题。本文介绍的这种方法切实可行,所以作者是通过讲述一个很实用的方法来阐述这篇文章的。‎ ‎  [答案] D ‎  ‎ ‎   (第16篇)‎ ‎  Plants can't communicate by moving or making sounds, as most animals do. Instead, plants produce volatile compounds,chemicals that easily change from a liquid to a gas. A flower' s sweet smell, for example, comes from volatile compounds that the plant produces to attract insects such as bugs and bees.‎ ‎  Plants can also detect volatile compounds produced by other plants.A tree under attack by hungry insects, for instance, may give off volatile compounds that let other trees know about the attack. In response, the other trees may send off chemicals to keep the bugs away - or even chemicals that attract the bugs' natural enemies.‎ ‎  Now scientists have created a quick way to understand what plants are saying: a chemical sensor(传感器) called an electronic nose. The "enose" can tell compounds that crop plants make when they're attacked.Scientists say the enose could help quickly detect whether plants are being eaten by insects. But today the only way to detect such insects is to visually inspect individual plants. This is a challenging task for managers of greenhouses ,enclosed gardens that can house thousands of plants.‎ ‎  The research team worked with an enose that recognizes volatile compounds. Inside the device, 13 sensors chemically react with volatile compounds. Based on these interactions, the enose gives off electronic signals that the scientists analyze using computer software.‎ ‎  To test the nose, the team presented it with healthy leaves from cucumber, pepper and tomato plants, all common greenhouse crops. Then the scientists collected samples of air around damaged leaves from each type of crop. These plants had been damaged by insects, or by scientists who made holes in the leaves with a hole punch (打孔器).‎ ‎  The enose, it turns out, could identify healthy cucumber,pepper and tomato plants based on the volatile compounds they produce. It could also identify tomato leaves that had been damaged. But even more impressive, the device could tell which type of damage - by insects or with a hole punch - had been done to the tomato leaves.‎ ‎  With some finetuning,a device like the enose could one day be used in greenhouses to quickly spot harmful bugs, the researchers say. A device like this could also be used to identify fruits that are perfectly ripe and ready to pick and eat, says Natalia Dudareva, a biochemist at Purdue University in West Lafayette, Ind. who studies smells of flowers and plants. Hopefully, scientists believe, the device could bring large benefits to greenhouse managers in the near future.‎ ‎  [语篇解读] 植物会说话吗?植物如何"交流"?本文中的科学家们正在尝试用enose来测试植物受到伤害时的反应。‎ ‎  5.We learn from the text that plants communicate with each other by________.‎ ‎  A. making some sounds ‎  B. waving their leaves ‎  C. producing some chemicals ‎  D. sending out electronic signals ‎  [解析] 事实细节题。从文章第一段中的"Instead,plants produce volatile compounds,chemicals that..."可知,植物通过它本身所产生的一种化学物质来进行交流,所以答案选C。‎ ‎  [答案] C ‎  6.What did the scientists do to find out if the enose worked?‎ ‎  A. They presented it with all common crops.‎ ‎  B. They fixed 13 sensors inside the device.‎ ‎  C. They collected different damaged leaves.‎ ‎  D. They made tests on damaged and healthy leaves.‎ ‎  [解析] 逻辑推理题。从文章第五段可知,为了证实enose的效果,科学家用受到伤害的叶子与健康的叶子来测试。所以选D。‎ ‎  [答案] D ‎  7.According to the writer, the most amazing thing about the enose is that it can________.‎ ‎  A. pick out ripe fruits ‎  B. spot the insects quickly ‎  C. distinguish different damages to the leaves ‎  D. recognize unhealthy tomato leaves ‎  [解析] 事实细节题。从倒数第二段中的"But even more impressive,the device could tell which type of damage..."可知,最令人惊奇的是enose能够分辨叶子受到的不同的"伤害"。‎ ‎  [答案] C ‎  8.We can infer from the last paragraph that the enose________.‎ ‎  A. is unable to tell the smell of flowers ‎  B. is not yet used in greenhouses ‎  C. is designed by scientists at Purdue ‎  D. is helpful in killing harmful insects ‎  [解析] 逻辑推理题。从文章最后一段中的"could one day be used in greenhouses to..."推断可知,enose还没有运用到greenhouses中,由此推断B是答案。‎ ‎  [答案] B ‎  ‎ ‎(第17篇)‎ ‎  The year 2009 is the Year of Ox. The ox is a representative of the farming culture of China. In the farming economy (经济), oxen are the major animals pulling plows (犁).‎ ‎  Of course, the good of oxen is not limited to plowing. In fact, they are seen as "boats_on_land" for their ability to carry loads. Besides, the whole body of an ox is full of treasures. Their meat and milk are food full of nutrition, and their skin can be used to make clothes and shoes. With all these qualities, oxen are regarded as generous creatures.‎ ‎  In the past, oxen played an important role in the spiritual life of the Chinese. Even today, oxen still play a special part in some folk activities. For example, some people who live in southwest China will cook cattle bone soup and share it among family members when holding the ceremony for children who reach 13. They believe that the cattle bone soup represents the blood relationship among family members. In order to express their love for oxen, people in some other areas will run to shake off diseases on the 16th day of the first month by the lunar calendar (农历), and during their run they will take their oxen along,which indicates they regard the creature as human.‎ ‎  Because of the contribution of oxen in their lives, the Chinese people are very grateful to the animal. In addition, the use of oxen in ceremonies and the thanks people owe to oxen help to develop various traditional customs, which becomes an important part of the folk culture of the Chinese nation.‎ ‎  [语篇解读] 牛年说牛,文章介绍了有关牛的一些民间文化。‎ ‎  1.The words "boats on land" underlined in Paragraph 2 refer to________.‎ ‎  A. animals for taking goods ‎  B. creatures for pulling plows ‎  C. treasures of the folk culture ‎  D. tools in the farming economy ‎  [解析] 猜测词义题。由第二段中的"they are seen as...ability to carry loads"内容可以得出答案,介词for是对前面作解释说明:为什么牛被看成陆地之舟。‎ ‎  [答案] A ‎  2.From the third paragraph, we know that________.‎ ‎  A. oxen are no more important today than in the past ‎  B. ceremonies are held when people cook cattle bone soup ‎  C. oxen are treated as human in some areas of China ‎  D. people run with oxen to shake off diseases every month ‎  [解析] 理解判断题。由文章第三段的首句以及最后一句内容可得出答案。‎ ‎  [答案] C ‎  3.Which of the following helps to develop traditional customs?‎ ‎  A. The special role of oxen in farming.‎ ‎  B. People' s respect and love for oxen.‎ ‎  C. The practical value of an ox's body.‎ ‎  D. The contribution of oxen to the economy.‎ ‎  [解析] 细节判断题。由文章最后一段最后一句话可得出答案。‎ ‎  [答案] B ‎  4.Why does the author write the text?‎ ‎  A. To stress the importance of oxen in farming.‎ ‎  B. To introduce the Chinese folk culture.‎ ‎  C. To describe how to celebrate the Year of Ox.‎ ‎  D. To explain how to develop agriculture with oxen.‎ ‎  [解析] 写作意图题。由文章内容可知,作者是为了介绍有关牛的一些民间文化。‎ ‎  [答案] B ‎(第18篇)‎ ‎  Have you ever noticed the colour of the water in a river or stream after a heavy rainfall? What do you think caused this change in colour? It is soil that has been washed into the river from the riverbank or from the nearby fields.‎ ‎  Components of Soil ‎  Soil is made up of a number of layers (层) , each having its own distinctive colour and texture. The upper layer is known as the litter. It acts like a blanket,limiting temperature changes and reducing water loss. The topsoil layer is made up of small particles of rock mixed with rotten plant and animal matter called humus (腐殖质), which is black and gives the topsoil its dark ‎ colour. This layer is usually rich in nutrients, oxygen, and water. Below the topsoil is the subsoil, a layer that contains more stones mixed with only small amounts of organic matter. This layer is lighter in colour because of the lack of humus. Beneath the soil lies a layer of bedrock.‎ ‎  Soil forms from the bottom up. Over time bedrock is attacked by rain, wind, frost, and snow. It is gradually broken down into smaller particles in a process called weathering. Plants begin to grow,and rotten materials enrich the topsoil. Most of the soil in Eastern Canada ,for example ,was formed from weathered rock that was exposed when the ice disappeared 12,000 years ago.‎ ‎  Water Beneath the Soil ‎  Surface water collects and flows above the ground in lakes ,ponds, and rivers. Once in the soil or rock, it is called groundwater. Gravity pulls groundwater through the soil in a process called percolation(渗透). Eventually the water reaches a layer called the water table. Under this is bedrock through which water cannot percolate.‎ ‎  As water percolates downward, it dissolves organic matter and minerals from the soil and carries them to deeper layers.This causes a serious problem because plants require these nutrients for growth.‎ ‎  Soil pH ‎  Soil can be acidic, neutral, or basic. The pH of the soil is determined by the nature of the rock from which it was formed,and by the nature of the plants that grow and rot in it.‎ ‎  The acidity of rain and snow can lower the pH of the groundwater that enters the soil. By burning fossil fuels such as coal ,oil and gasoline,humans have been contributing to higher levels of acidity in many soils. When fossil fuels are burned,gases are released into the air and then fall back to earth as acid rain. Acid soil increases the problem of carrying nutrients to lower soil levels. As nutrients are removed,soil is less fertile. Plants grow more slowly in acidic soil,and also become easily attacked by diseases.‎ ‎  [语篇解读] 本文是一篇科普文,介绍了土壤的构成与土壤的pH对于植物生长的影响。‎ ‎  5.The layer of soil that provides necessary nutrients for plant growth is called________.‎ ‎  A. litter         B. topsoil ‎  C. humus D. subsoil ‎  [解析] 考查细节理解。根据文章第二段中的"The topsoil layer is made up of small particles of rock mixed with rotten plant...which is black and gives the topsoil its dark colour.This layer is usually rich in nutrients"可知给植物提供必要营养的土层是topsoil layer。‎ ‎  [答案] B ‎  6.According to the text, which of the following is NOT true?‎ ‎  A. Soil forms from weathered rock on the earth surface.‎ ‎  B. The deeper layer of soil is darker in colour than the surface soil.‎ ‎  C. Air pollution is partially responsible for acid soil.‎ ‎  D. Groundwater tends to carry away nutrients for plant growth.‎ ‎  [解析] 考查细节判断。根据文章第二段中的"Below the topsoil is the subsoil,a layer that contains more stones mixed with only small amounts of organic matter.This layer is lighter in colour because of the lack of humus"可知,并不是土层深,它的颜色就深。‎ ‎  [答案] B ‎  7.We can infer from the passage that the water table lies________.‎ ‎  A. between the topsoil layer and the subsoil layer ‎  B. in the subsoil layer above bedrock ‎  C. between the subsoil layer and bedrock ‎  D. in the bedrock layer beneath the subsoil ‎  [解析] 考查细节理解。根据文章第四段中的"Eventually the water reaches a layer called the water table.Under this is bedrock through which water cannot percolate"可知water table是位于bedrock之上的一个土层,它是独立存在的不属于其他层,因此选项C正确。‎ ‎  [答案] C ‎  8.The underlined word "dissolve" is used to express the idea that organic matter and minerals from soil are________.‎ ‎  A. rushed away into the river ‎  B. cleaned and purified by water ‎  C. destroyed and carried away by water ‎  D. mixed with water and become part of it ‎  [解析] 考查词义猜测。根据本句中的"...and carries them to deeper layers"可知营养物质是在被水溶解之后才被带到更深的土层中去的。‎ ‎  [答案] D ‎(第19篇)‎ ‎  The latest research suggests that the key factor separating geniuses from the merely accomplished is not IQ, a generally bad predictor of success.Instead, it' s purposeful practice.Top performers spend more hours practising their craft. If you wanted to picture how a typical genius might develop, you' d take a girl who possessed a slightly above average language ability. It wouldn' t have to be a big talent, just enough so that she might gain some sense of distinction. Then you would want her to meet, say, a novelist, who coincidentally shared some similar qualities. Maybe the writer was from the same town, had the same family background, or, shared the same birthday.‎ ‎  This contact would give the girl a vision of her future self.It would give her some idea of a fascinating circle she might someday join.It would also help if one of her parents died when she was 12, giving her a strong sense of insecurity and fuelling a desperate need for success. Armed with this ambition, she would read novels and life stories of writers without end. This ‎ would give her a primary knowledge of her field.She'd be able to see new writing in deeper ways and quickly understand its inner workings.‎ ‎  Then she would practise writing. Her practice would be slow, painstaking and errorfocused. By practising in this way,she delays the automatizing process. Her mind wants to turn conscious, newly learned skills into unconscious, automatically performed skills. By practising slowly, by breaking skills down into tiny parts and repeating, she forces the brain to internalize a better pattern of performance. Then she would find an adviser who would provide a constant stream of feedback, viewing her performance from the outside, correcting the smallest errors,pushing her to take on tougher challenges. By now she is redoing problems - how do I get characters into a room - dozens and dozens of times. She is establishing habits of thought she can call upon in order to understand or solve future problems.‎ ‎  The primary quality our young writer possesses is not some mysterious genius. It' s the ability to develop a purposeful, laborious and boring practice routine. The latest research takes some of the magic out of great achievement. But it underlines a fact that is often neglected. Public discussion is affected by genetics and what we're "hardwired" to do. And it's true that genes play a role in our capabilities. But the brain is also very plastic. We construct ourselves through behavior.‎ ‎  [语篇解读] 你依然认为天才由基因决定,认为自己不可能成为天才吗?一项新的研究表明:成功并不取决于人的IQ,而是通过后天有目的的训练得来的。‎ ‎  1.The passage mainly deals with________.‎ ‎  A. the function of IQ in cultivating a writer ‎  B. the relationship between genius and success ‎  C. the decisive factor in making a genius ‎  D. the way of gaining some sense of distinction ‎  [解析] 主旨大意题。这篇文章主要讲了决定成功的主要因素并不是IQ,而是些后天的、有目的的训练和努力。所以C项正确。A、B两项不符合文章意思;D项不具体。‎ ‎  [答案] C ‎  2.By reading novels and writers' stories, the girl could________.‎ ‎  A. come to understand the inner structure of writing ‎  B. join a fascinating circle of writers someday ‎  C. share with a novelist her likes and dislikes ‎  D. learn from the living examples to establish a sense of security ‎  [解析] 细节判断题。依据文章第二段的"This would give her a primary knowledge of her field.She'd be able to see new writing in deeper ways and quickly understand its inner workings."可知A项正确。‎ ‎  [答案] A ‎  3.In the girl' s long painstaking training process,________.‎ ‎  A. her adviser forms a primary challenging force to her success ‎  B. her writing turns into an automatic pattern of performance ‎  C. she acquires the magic of some great achievements ‎  D. she comes to realize she is "hardwired" to write ‎  [解析] 细节理解题。根据文章第三段的"By practising in this way,she delays the automatizing process.Her mind wants to turn conscious,newly learned skills into unconscious,automatically perfomed skills."可知B项正确。‎ ‎  [答案] B ‎  4.What can be concluded from the passage?‎ ‎  A. A fuelling ambition plays a leading role in one' s success.‎ ‎  B. A responsible adviser is more important than the knowledge of writing.‎ ‎  C. As to the growth of a genius, IQ doesn' t matter, but just his/her efforts.‎ ‎  D. What really matters is what you do rather than who you are.‎ ‎  [解析] 推理判断题。文章主要讲述:一个人的成功并不是由IQ决定的而是通过后天的努力训练得来的。D项"重要的不是你是谁而是你做了什么"符合文章主旨大意。‎ ‎  [答案] D ‎(第20篇)‎ ‎  ‎ ‎  (2009年江西,C)‎ ‎  237 West Palmdale Boulevard ‎  Fresno, California 93706 ‎ ‎  AmToy Corporation ‎  Suite 15‎ ‎  TransAm Building ‎  San Francisco, California 94115‎ ‎  November 20,2008 ‎ Dear Sirs,‎ ‎  As a concerned parent, I am writing to protest your recent advertisement for ElectroRobo seen in local media in ‎  California. Specifically, I am referring to newspaper and magazine ads (attached to this letter)published the week of November 15.‎ ‎  Children respond to your type of advertising in an immature way;that is, they are unable to understand how expensive some toys are for middleclass parents. Further, your product is violent in nature. Your advertisement gives children the impression that it's fine to have"two guns and laser eyes. "You also suggest that children need your toy to protect them"when you go outside. "This is not a healthy attitude for children to have.‎ ‎  I hope you will stop advertising your product in such a way that may harm our children.‎ ‎  Sincerely yours,          ‎ ‎  (Mrs.) Alma Hernandez       ‎ ‎  President, Parents for Nonviolent Toys ‎  Enc.‎ ‎  It's Here ! Ready for You, Now ! It's ElectroRobo !‎ ‎  Every boy dreams of being in control of a robot, and AmToy can make your dream come true! ElectroRobo is the world's first fully automatic robot with radio control. Standing 80 centimeters tall, ElectroRobo is like a friend at home. He can walk, talk, and even shake your hand! He has two guns and laser eyes to help you defend yourself when you go outside with him. Every boy needs ElectroRobo !‎ ‎  Ask your Mom and Dad to buy ElectroRobo for your birthday or for Christmas, which is coming up soon. Imagine that you are in control of your friend for life, ElectroRobo!‎ Available at all toy stores and department stores NOW!‎ ‎   [语篇解读] 本文是家长写给一家玩具公司的一封抗议信,反对该公司有误导倾向的广告及有暴力倾向的产品。‎ ‎  5.What is the purpose of the letter?‎ ‎  A. To complain about a broken toy.‎ ‎  B. To oppose the advertising.‎ ‎  C. To order a gift for Christmas.‎ ‎  D. To apply for a job in a toy company.‎ ‎  [解析] 推理判断题。通读文章可知,写这封信的意图是反对这家公司所做的具有暴力倾向和有误导之嫌的广告,故选B。‎ ‎  [答案] B ‎  6.Why does the writer of the letter feel that ElectroRobo is violent?‎ ‎  A. It is controlled by radio waves.‎ ‎  B. It is expensive to buy.‎ ‎  C. It is 80 centimeters tall.‎ ‎  D. It bears arms.‎ ‎  [解析] 事实细节题。由第二部分第一段倒数第二句He has two guns and laser eyes...可知这玩具持有武器。‎ ‎  [答案] D ‎  7.What does"Enc. "at the end of the letter mean?‎ ‎  A. Something attached to the letter.‎ ‎  B. A complaint to the toy company.‎ ‎  C. A hidden message.‎ ‎  D. An encouraging response.‎ ‎  [解析] 猜测词义题。由下面的附加广告可以推出该词的含义为"附文"。‎ ‎  [答案] A ‎  8.ElectroRobo can do all the following EXCEPT________.‎ ‎  A. sitting down       B. shaking hands ‎  C. talking D. walking ‎  [解析] 事实细节题。由第二部分第一段倒数第三句描述该电动机器人的He can walk,talk,and even shake your hand!可知四个选项中只有A没有提及,故选A。‎ ‎  [答案] A ‎(第21篇)‎ ‎  It was the first snow of winter - an exciting day for every child but not for most teachers. Up until now, I had been able to dress myself for recess (课间休息), but today I would need some help.Miss Finlayson, my kindergarten teacher at Princess Elizabeth School near Hamilton,Ontario,had been through first snow days many times in her long career, but l think she may still remember this one.‎ ‎  I managed to get into my wool snow pants. But I struggled with my jacket because it didn' t fit well. It was a handmedown from my brother, and it made me wonder why I had to wear his ugly clothes. At least my hat and matching scarf were mine, and they were quite pretty. Finally it was time to have Miss Finlayson help me with my boots. In her calm, motherly voice she said, "By the end of winter, you will be able to put on your own boots. "I didn't realize at the time that this was more a statement of hope than of confidence.‎ ‎  I handed her my boots and stuck out my feet. Like most children, I expected the adult to do all the work. After much wiggling and pushing, she managed to get the first one into place and then, with a sigh, worked the second one on too.‎ ‎  I announced, "They' re on the wrong feet. "With the grace that only experience can bring, she struggled to get the boots off and went through the joyless task of putting them on again.Then I said," These aren't my boots, you know." As she pulled the offending boots from my feet, she still managed to look both helpful and interested. Once they were off, I said,"They are my brother' s boots. My mother makes me wear them, and I hate them !" Somehow, from long years of practice,she managed to act as though I wasn't an annoying little girl.She pushed and shoved, less gently this time, and the boots were returned to their proper place on my feet. With a great sigh of relief, seeing the end of her struggle with me, she asked, "Now,where are your gloves?"‎ ‎  I looked into her eyes and said," I didn' t want to lose them, so I put them into the toes of my boots."‎ ‎  [语篇解读] 本文通过为小作者穿靴子的故事,描写了老师的善解人意和小女孩的顽皮。‎ ‎  1.According to the passage, the little girl got________from her brother.‎ ‎  A. the wool snow pants and the jacket ‎  B. the jacket and the boots ‎  C. the jacket and the hat ‎  D. the boots and the gloves ‎  [解析] 考查细节理解。根据文章第二段中的"...my jacket because it didn't fit well.It was a handmedown from my brother..."与第四段中的"These aren't my boots...They are my brother's boots.My mother makes me wear them"可知作者穿的是她哥哥的夹克与靴子。‎ ‎  [答案] B ‎  2.What made it so hard for the teacher to help the little girl put her boots on?‎ ‎  A. The gloves in the toes of the boots.‎ ‎  B. The slowness of the teacher.‎ ‎  C. The wrong size of the boots.‎ ‎  D. The unwillingness of the girl.‎ ‎  [解析] 考查细节理解。根据文章的最后一段的"I didn't want to lose them,so I put them into the toes of my boots"可知让帮助穿鞋的老师费了那么多耐心与力气的是藏在鞋子里的手套。‎ ‎  [答案] A ‎  3.It can be inferred that before the little girl finally went out to enjoy the first snow of winter, the teacher had to help her put on her boots________.‎ ‎  A. once           B. twice ‎  C. three times D. four times ‎  [解析] 考查推理判断。从文章第三段与第四段可知老师要帮小作者穿四次鞋:第一次是穿错了脚,第二次是脱下来重新穿上,第三次是作者说了"The aren't my boots"之后,她又把鞋脱下来,当听说是作者的妈妈非让作者穿哥哥的鞋后,又把它们给作者穿上,第四次是听说手套被放在鞋子里了,就又脱下来重新穿了一次(文中没有点明,但可推出)。‎ ‎  [答案] D ‎  4.Which of the following sentences from the text BEST indicates that the teacher is very considerate?‎ ‎  A. In her calm, motherly voice she said," By the end of winter...."( Paragraph 2)‎ ‎  B. With the grace that only experience can bring, she struggled to... (Paragraph 4)‎ ‎  C. ...she still managed to look both helpful and interested. ( Paragraph 4 )‎ ‎  D. ...she managed to act as though I wasn't an annoying little girl. ( Paragraph 4 )‎ ‎  [解析] 考查细节理解。经过多年的磨砺,老师每次给她穿鞋都是那么有耐心,好像一点也不把小女孩的顽皮放在心上,这足以说明老师很体贴人。‎ ‎  [答案] D ‎(第22篇)‎ ‎  An increase in students applying to study economics at university is being attributed to (归因于) the global economic crisis awakening a public thirst for knowledge about how the financial system works.‎ ‎  Applications for degree courses beginning this autumn were up by 15% this January, according to UCAS, the Universities and Colleges Admissions Service. A spokesman for the Royal Economic Society said applications to do economics at Alevel were also up.‎ ‎  Professor John Beath, the president of the society and a leading lecturer at St Andrews University, said his firstyear lectures - which are open to students from all departments - were drawing crowds of 400, rather than the usual 250.‎ ‎  "There are a large number of students who are not economics majors, who would like to learn something about it.One of the things I have done this year is to relate my teaching to contemporary events in a way that one hash' t traditionally done," he added.‎ ‎  University applications rose 7% last year, but there were rises above average in several subjects. Nursing saw a 15% jump, with people's renewed interest in careers in the public sector(部门), which are seen as more secure in economic crisis.‎ ‎  A recent study showed almost two thirds of parents believed schools should do more to teach pupils about financial matters, and almost half said their children had asked them what was going on, although a minority of parents felt they did not understand it themselves well enough to explain.‎ ‎  Zack Hocking, the head of Child Trust Funds, said: "It's possible that one good thing to arise from the downturn will be a generation that' s financially wiser and better equipped to manage their money through times of economic uncertainty. "‎ ‎  [语篇解读] 本文主要讲述了金融危机下学生们开始更多地选择与经济有关的专业这一现象。‎ ‎  5.Professor John Beath' s lectures are________.‎ ‎  A. given in a traditional way ‎  B. connected with the present situation ‎  C. open to both students and their parents ‎  D. warmly received by economists ‎  [解析] 事实细节题。从第四段第二句话可知他把教学与当前的一些重大事件紧密相连,即贴近现实生活,故B(与当前的形势相联系)为正确答案。‎ ‎  [答案] B ‎  6.Careers in the public sector are more attractive because of their________.‎ ‎  A. greater stability B. higher pay ‎  C. fewer applications D. better reputation ‎  [解析] 事实细节题。从第五段最后可知在当前金融危机下这些公共部门的工作以其安全性和稳定性吸引了求职者,故A(极大的稳定性)为正确答案。‎ ‎  [答案] A ‎  7.In the opinion of most parents,________.‎ ‎  A. economics should be the focus of school teaching ‎  B. more students should be admitted to universities ‎  C. the teaching of financial matters should be strengthened ‎  D. children should solve financial problems themselves ‎  [解析] 事实细节题。由倒数第二段的内容可知应该"加强关于经济问题方面的教学",答案为C。‎ ‎  [答案] C ‎  8.According to Hocking, the global economic crisis might make the youngsters ________.‎ ‎  A. wiser in money management ‎  B. have access to better equipment ‎  C. confident about their future careers ‎  D. get jobs in Child Trust Funds ‎  [解析] 推理判断题。Hocking认为经济下滑带来的一个好处就是让年轻人学会了更明智地处理金钱方面的问题,在经济不稳定时期更好地理财。所以答案为A。‎ ‎  [答案] A ‎  9.What' s the main idea of the text?‎ ‎  A. Universities have received more applications.‎ ‎  B. Economics is attracting an increasing number of students.‎ ‎  C. College students benefit a lot from economic uncertainty.‎ ‎  D. Parents are concerned with children' s subject selecttion.‎ ‎  [解析] 主旨大意题。第一段提到全球性的金融危机使公众想更多地了解有关经济体系方面的一些问题,所以更多的学生申请经济学专业,本文正是对此展开的论述,故答案为B。‎ ‎  [答案] B ‎(第23篇)‎ ‎  A volcanic eruption in Iceland has sent ash across northern Europe. Airlines have stopped or changed the flights across the Atlantic Ocean, leaving hundreds of passengers stuck in airports.‎ ‎  Grimsvotn is one of the largest and most active volcanoes in Europe. What makes Grimsvotn different is that it lies under a huge glacier(冰川) of ice up to 12 meters thick. The hot volcano heats up the ice above it, which then forms a layer (层) of water between the glacier and the volcano. This layer of water puts pressure on the volcano, keeping it stable. As the water flows ont from under the glacier, the pressure lifts. The lava(岩浆) from the volcano then comes up to the surface.This is exactly what happened today.‎ ‎  Now, airlines have to make changes to their flights so as not to fly through the clouds of volcanic ash.According to KLM, one of Europe' s biggest airlines, airplanes cannot go under the cloud or over it. Going through the cloud can result in ash getting stuck in the airplane' s engines, causing damage to the plane.‎ ‎  The eruption has also caused problems for animals in Iceland. The volcano left ash and sharp, glasslike rocks all over the countryside.Farmers are keeping their animals inside to stop them from eating ashcovered grass or the sharp objects?‎ ‎  [语篇解读] 本文向我们介绍了一座叫做Crimsvotn的火山。这座火山与其他火山最大的不同在于:它位于厚达12米的冰川下面。‎ ‎  1.What makes Grimsvotn different from other volcanoes?‎ ‎  A. It is below ice.‎ ‎  B. It lies under the sea.‎ ‎  C. It is the largest volcano.‎ ‎  D. Its lava affects the airlines.‎ ‎  [解析] 由第二段第二句话可知。‎ ‎  [答案] A ‎  2.What keeps Grimsvotn still?‎ ‎  A. The slow flow of water.‎ ‎  B. The low water temperature.‎ ‎  C. The thick glacier.‎ ‎  D. The water pressure.‎ ‎  [解析] 根据第二段第四句话可知。‎ ‎  [答案] D ‎  3.Which of the following is the result of the volcanic eruption?‎ ‎  A. People stop traveling in Europe.‎ ‎  B. Airlines suffer from the loss of planes.‎ ‎  C. It becomes dangerous for animals to eat outside.‎ ‎  D. Farmers have lost many of their animals.‎ ‎  [解析] 根据最后一段内容可得出答案。‎ ‎  [答案] C ‎  4.This text is most probably taken from________.‎ ‎  A. a research paper B. a newspaper report ‎  C. a class presentation D. a geography textbook ‎  [解析] 根据文章内容可知,这应该是一篇新闻报道。‎ ‎  [答案] B ‎(第24篇)‎ ‎  Tens of thousands of theatre tickets will be given away to young people next year as part of a government campaign to inspire a lifelong love for theatre.‎ ‎  The plan to offer free seats to people aged between 18 to 26 - funded with £ 2.5 million of taxpayers' money - was announced yesterday by Andy Burnham, the Culture Secretary.It ‎ received a cautious welcome from some in the arts world,who expressed concern that the tickets may not reach the most underprivileged.‎ ‎  The plan comes as West End theatres are enjoying record audiences, thanks largely to musicals teaming up with television talent shows. Attendances reached 13.6 million in 2007,up 10 percent on 2006, itself a record year. Total sales were up 18 percent on 2006 to almost £ 470 million.‎ ‎  One theatre source criticised the Government' s priorities(优先考虑的事) in funding free tickets when pensioners were struggling to buy food and fuel, saying: "I don' t know why the Government' s wasting money on this. The Young Vic, as The Times reported today,offers excellent performances at cheap prices."‎ ‎  There was praise for the Government' s plan from Dominic Cooke of the Royal Court Theatre, who said: "I support any move to get young people into theatre, and especially one that aims to do it all over England, not just in London."‎ ‎  Ninetyfive publicly funded theatres could apply for funding under the twoyear plan. In return, they will offer free tickets on at least one day each week to 18 to 26yearolds,firstcome, firstserved. It is likely to be on Mondays, traditionally a quiet night for the theatre.‎ ‎  Mr. Burnham said: "A young person attending the theatre can find it an exciting experience, and be inspired to explore a new world. But sometimes people miss out on it because they fear it' s'not for them'. It' s time to change this perception."‎ ‎  Jeremy Hunt, the Shadow Culture Secretary, said: "The real issue is not getting enthusiastic children into the theatre,but improving arts education so that more young people want to go in the first place. For too many children theatres are a nogo area."‎ ‎  [语篇解读] 为了激发年轻一代对戏剧的喜爱,政府开展了一项运动:将于明年送出成千上万张戏票。这一计划在社会各界引起了不同的反应。‎ ‎  5.Critics of the plan argued that________.‎ ‎  A. the theatres would be overcrowded ‎  B. it would be a waste of money ‎  C. pensioners wouldn' t get free tickets ‎  D. the government wouldn' t be able to afford it ‎  [解析] 细节理解题。由文中第四段开头的criticised和"I don't know why the Government's wasting money on this."一句可知,批评者认为这一计划是在浪费金钱,故选B。‎ ‎  [答案] B ‎  6.According to the supporters, the plan should________.‎ ‎  A. benefit the television industry ‎  B. focus on producing better plays ‎  C. help increase the sales of tickets ‎  D. involve all the young people in England ‎  [解析] 细节理解题。由文中第五段最后一句话可知,支持该计划的人支持让年轻人到剧院去的任何运动、措施或手段,尤其是那种能够让全英国人而不是仅伦敦人能参与的措施。故选D。‎ ‎  [答案] D ‎  7.Which of the following is TRUE about the plan?‎ ‎  A. Ninetyfive theatres have received funding.‎ ‎  B. Everyone will get at least one free ticket.‎ ‎  C. It may not benefit all the young people.‎ ‎  D. Free tickets are offered once every day.‎ ‎  [解析] 细节判断题。A项错,95%的剧院只是可以申请该基金,而非获得;B项错在everyone,过于绝对;D项错是因为免费票只是每周发放一次,而非每天都有。‎ ‎  [答案] C ‎  8.We can infer from the passage that in England________.‎ ‎  A. many plays are not for young people ‎  B. many young people don' t like theatre ‎  C. people know little about the plan ‎  D. children used to receive good arts education ‎  [解析] 推理判断题。由文章最后一段可知真正的问题不是让孩子们进入到剧院里,而是要提高他们的艺术教 ‎  育水平,让更多的年轻人愿意进入剧院,由此可见许多年轻人是不喜欢戏剧的。‎ ‎  [答案] B ‎  9.According to the passage, the issue to offer free tickets to young people seems ________.‎ ‎  A. controversial B. inspiring ‎  C. exciting D. unreasonable ‎  [解析] 推理判断题。由第二、四、五三段可知该计划得到了艺术界一些人士的谨慎的欢迎,另一些人则持批评态度,还有一些人则赞扬该计划。由此可见这项计划是有争议的(controversial)。‎ ‎  [答案] A ‎(第25篇)‎ ‎  Foreign drivers will have to pay onthespot fines of up to £900 for breaking the traffic law to be carried out next month.‎ ‎  If they do not have enough cash or a working credit crad, their vehicles will be clamped (扣留)until they pay-and they will face an additional fee of £ 80 for getting back their vehicles.‎ ‎  The law will also apply to British citizens.The fines will be described officially as "deposits" when the traffic law takes effect, because the money would be returned if the driver went to court and was found not guilty. In practice, very few foreign drivers are likely to return to Britain to deal with their cases.‎ ‎  Foreign drivers are rarely charged because police cannot take action against them if they fail to appear in court. Instead, officers often merely give warnings.‎ ‎  Three million foreiguregistered vehicles enter Britain each year. Polish vehicles make up 36 percent, French vehicles 10 percent and German vehicles 9 percent.‎ ‎  Foreign vehicles are 30 percent more likely to be in a crash than Britishregistered vehicles. The number of crashes caused by foreign vehicles rose by 47 percent between 2003 and 2008. There were almost 400 deaths and serious injuries and 3,000 slight injuries from accidents caused by foreign vehicles in 2008.‎ ‎  The new law is partly intended to settle the problem of foreign lorry drivers ignoring limits on weight and hours at the wheel.Foreign lorries are three times more likely to be in a crash than British lorries. Recent spot checks found that three quarters of lorries that failed safety tests were registered overseas.‎ ‎  The standard deposit for a careless driving offence - such as driving too close to the vehicle in front or reading a map at the wheel - will be £ 300. Deposits for speeding offences and using mobile phones will be £ 60. Foreign drivers will not get points as punishment added to their licenses, while British drivers will.‎ ‎  [语篇解读] 这是一篇新闻报道:英国为了解决国外车辆导致的交通事故高发的问题,从下月起要实施一项措施:对违反交通规则的外国司机处以高达900英镑的现场罚款,如果当场不交,还要额外收取80英镑的(车辆)保管费。‎ ‎  1.The first paragraph serves as a(n)________.‎ ‎  A. explanation        B. introduction ‎  C. comment D. background ‎  [解析] 推理判断题。通读全文可知,第一段起的作用是引出这段新闻的具体内容,所以这段是引子,所以选B。‎ ‎  [答案] B ‎  2.The foreign drivers who break the traffic law and do not pay on the spot are likely to be fined up to________.‎ ‎  A. £ 60 B. £ 300‎ ‎  C. £ 900 D.£ 980‎ ‎  [解析] 数字计算题。依据文章第一段可知,这种罚款可高达900英镑,再依据第二段:如果司机不能够立刻缴纳罚款,还要额外缴纳80英镑的(车辆)保管费用,共计980英镑。‎ ‎  [答案] D ‎  3.We can learn from the passage that________.‎ ‎  A. many foreign drivers have been fined by British police ‎  B. 300,000 German vehicles enter Britain every year ‎  C. 25 percent of foreign vehicles entering Britain have failed safety tests ‎  D. British drivers will be punished with points and fines for breaking the traffic law ‎  [解析] 推理判断题。依据文章最后一段可知,英国本国的司机除了要缴纳罚款之外,还要扣驾照的点数,而国外的车辆只罚款,由此得出D项正确。‎ ‎  [答案] D ‎  4.The new traffic law is mainly intended to________.‎ ‎  A. limit the number of foreign vehicles entering Britain ‎  B. increase the British government's additional income ‎  C. lower the rate of traffic accidents and injuries ‎  D. get foreign drivers to appear in court ‎  [解析] 推理判断题。依据文章倒数第二段第一句话"The new law is partly intended to settle the problem of foreign lorry drivers ignoring limits on weight and hours at the wheel"及文章第五、六两段,可知选C。‎ ‎  [答案] C ‎   (第26篇)‎ ‎  Below are some classified ads from an English newspaper.‎ ‎  classified ads ‎  FOR DIRECT CLASSIFIED SERVICE CALL 800 0557 10A. M. 4P. M. MONDAY FRIDAY ‎  ●FOR SALE ‎  COME to our moving sale - Plants, pottery, books,clothes, etc. Sat, Dec. 14th - 9a. m. 5p. m. 1612 Ferndale, Apt. 1,800 4696.‎ ‎  USED FUR COATS and JACKETS. Good condition. $ 30 $ 50. Call 800 0436 after 12 noon.‎ ‎  MOVING: Must sell. TV21, $ 50; AM/FM radio A/C or battery, $ 15; cassette tape recorder, $ 10. Call Jon or Pat ,800 0739 after 5p. m. or weekends.‎ ‎  SHEEPSKIN COAT:man' s, size 42, 1 year old. $ 85. After 6 p. m. ,800 5224.‎ ‎  ●LOST AND FOUND ‎  FOUND: Cat, 6 months old, black and white markings. Found near Linden and South U. Steve. 800 4661.‎ ‎  LOST: Gold wire rim glasses in brown case. Campus area. Reward. Call Gregg 8002896.‎ ‎  FOUND: Set of keys on Tappan near Hill intersection. Identify key chain. Call 800 9662.‎ ‎  FOUND: Nov. 8th - A black and white puppy in PackardJewett area. 800 5770.‎ ‎  ●PERSONAL ‎  OVERSEAS JOBS-Australia, Europe, S. America, Africa. Students all professions and occupations, $ 700 to $ 3000 monthly. Expenses paid, overtime. Sightseeing. Free information at STUDENTS'UNION.‎ ‎  THE INTERNATIONAL CENTER plans to publish a booklet of student travel adventures. If you' d like to write about your foreign experiences, unusual or just plain interesting, call us (800 9310) and ask for Mike or Janet.‎ ‎  UNSURE WHAT TO DO?‎ ‎  LifePlanning Workshop, Dec. 13th 15th. Bob and Margaret Atwood, 800 0046.‎ ‎  ●ROOMMATES ‎  FEMALE ROOMMATE ‎  WANTED: Own room near campus. Available December 1st. Rent $ 300 per month until March 1st. $ 450 thereafter. Call Jill for details, 800 7839.‎ ‎  NEED PERSON to assume lease for own bedroom in apt. near campus, $ 380/mo. starting Jan. 1st. Call 800 6157 after 5p. m.‎ ‎  ●DOMESTIC SERVICE ‎  EARLY HOUR WAKEUP ‎  SERVICE: For prompt, courteous wakeup service, call 800 0760.‎ ‎  ●HELP WANTED ‎  BABYSITTER - MY HOME ‎  If you are available a few hours during the day, and some evenings to care for 2 schoolage children, please call Gayle Moore, days 800 1111, evenings and weekends 800 4964.‎ ‎  PERSONS WANTED for delivery work. Own transportation. Good pay. Apply 2311 E. Stadium. Office 101, after 9 a. m.‎ ‎  TELEPHONE RECEPTIONIST WANTED. NO experience necessary. Good pay. Apply 2311 E. Stadium. Office 101 ,after 9 a. m.‎ ‎  WAITRESS WANTED: 10a. m. 2 p. m. or 10∶30 a. m. 5 p.m. Apply in person. 207 S. Main, Curtis Restaurant.‎ ‎  HELP WANTED for house cleaning 1/2 day on weekends. When-to be discussed for mutual convenience. Good wages. Sylvan Street. Call 800 2817.‎ ‎  [语篇解读] 这是刊登在英语报纸上的一些广告。‎ ‎  5.Where will you post a notice if you need someone to look after your children?‎ ‎  A. PERSONAL.‎ ‎  B. HELP WANTED.‎ ‎  C. DOMESTIC SERVICE.‎ ‎  D. ROOMMATES.‎ ‎  [解析] 事实细节题。从第六则HELP WANTED中的BABYSITTER可知答案选B。‎ ‎  [答案] B ‎  6.A secondhand jacket will probably cost you________.‎ ‎  A. $ 60 B. $ 40‎ ‎  C. $ 20 D. $ 10‎ ‎  [解析] 事实细节题。从第一则FOR SALE中,发现jackets的价格在30美元50美元,所以答案选B。‎ ‎  [答案] B ‎  7.To have your travel notes published ,you may contact________.‎ ‎  A. Students' Union ‎  B. Gayle Moore ‎  C. The International Center ‎  D. LifePlanning Workshop ‎  [解析] 事实细节题。从文章第三则广告PERSONAL中的THE INTERNATIONAL CENTER中可以知道答案选C。‎ ‎  [答案] C ‎  8.If you want to have someone wake you up in the morning,you may call________.‎ ‎  A. 800 5224 B. 800 5770‎ ‎  C. 800 7839 D. 800 0760‎ ‎  [解析] 事实细节题。从文章第五则广告DOMESTIC SERVICE中的EARLY HOUR WAKEUP可以知道答案选D。‎ ‎  [答案] D ‎  ‎ ‎(第27篇)‎ ‎  ‎ ‎  Welcome to Banff, Canada' s first, most famous and arguably most fascinating national park. If you've come to ski or snowboard, we'll see you on the slopes. Skiing is a locals' favourite too.‎ ‎  While you' re here, try other recreational activities available in our mountains. Popular choices include a Banff Gondola ride up Sulphur Mountain, bathe in the natural mineral waters at the Upper Hot Spring, horsedrawn sleigh ride, driveyourownteam dog sled excursion, and snowmobile tour to the highland (but not in the national park).‎ ‎  We also recommend you make time to enjoy simple pleasures. After looking around Banff Ave shops, walk a couple of blocks west or south to the scenic Bow River.‎ ‎  Try ice skating on frozen Lake Louise where Ice Magic International Ice Sculpture Competition works are displayed after Jan 25. You can rent skates in Banff or at the sport shop in the Fairmont Chateau Lake Louise hotel.‎ ‎  Banff' s backcountry paths access a wilderness world of silence and matchless beauty - cross country skis and snowshoes provide the means. Banff sport shops rent equipment and clothes, or join an organized tour. Although we've been many times, we still find the cliffs and icefalls of our frozen canyons worth visiting.‎ ‎  Wildlife watching also creates satisfying memories. We have seen hundreds of the elk and bighorn sheep that attract visitors, yet they still arouse a sense of wonder. And the rare spotting of a cougar, wolf or woodland caribou takes our breath away.‎ ‎  See if simple pleasures work for you. Fight in the snow with your kids, walk bdside a stream or climb to a high place and admire the view.‎ ‎  -Banff Resort Guide Editors ‎  [语篇解读] 本文介绍了加拿大最有名的国家公园--Banff,在那儿你可以尽情享受冬季旅游的乐趣!‎ ‎  1.According to the passage, Banff's backcountry is accessible by________.‎ ‎  A. cross country skiing ‎  B. horsedrawn sleigh riding ‎  C. snowmobiling ‎  D. dogsledding ‎  [解析] 细节理解题。根据文章第五段第一句话中的"cross country skis and snowshoes provide the means"可知A项正确。‎ ‎  [答案] A ‎  2.Which of the following is true according to the passage?‎ ‎  A. Dogsledding is the most popular sport among local people.‎ ‎  B. Watching wildlife is a memorable experience.‎ ‎  C. Travellers should bring their own sports equipment.‎ ‎  D. Shopping is too simple a pleasure to enjoy.‎ ‎  [解析] 细节判断题。根据文章第六段第一句话"Wildlife watching also creates satisfying memories."可知B项正确。根据文章第一段的最后一句话可知A项错误;由文章第五段中的"Banff sport shops rent equipment"可知C项错误;D项在文中没有被提及。‎ ‎  [答案] B ‎  3.The purpose of the writing is to________.‎ ‎  A. promote scenic spots in Canada ‎  B. advertise for the sports in Banff ‎  C. introduce tourist activities in Banff ‎  D. describe breathtaking views in Banff ‎  [解析] 写作意图题。由文章的主要内容以及文章最后的落款"Banff Resort Guide Editors"可知这篇文章的主要目的就是介绍在Banff的旅游活动。A项中的in Canada太宽泛,不具体;B项没有体现出"旅游"的意思;D项中的views也不具体,所以答案选C。‎ ‎  [答案] C ‎(第28篇)‎ ‎  Sometimes,the simplest ideas are the best. For example,to absorb heat from the sun to heat water, you need large, flat,black surfaces. One way to do that is to build those surfaces specially, on the roofs of buildings. But why go to all that trouble when cities are full of black surfaces already, in the form of asphalt (柏油) roads?‎ ‎  Ten years ago, this thought came into the mind of Arian de Bondt, a Dutch engineer. He finally persuaded his boss to follow it up. The result is that their building is now heated in winter and cooled in summer by a system that relies on the surface of the road outside.‎ ‎  The heatcollector is a system of connected water pipes.Most of them run from one side of the street to the other, just under the asphalt road. Some, however, dive deep into the ground.‎ ‎  When the street surface gets hot in summer, water pumped through the pipes picks up this heat and takes it underground through one of the diving pipes. At a depth of 100 metres lies a natural aquifer (蓄水层) into which several heat exchangers (交换器) have been built. The hot water from the street runs through these exchangers, warming the groundwater, before returning to the surface through another pipe. The aquifer is thus used as a heat store.‎ ‎  In winter, the working system is changed slightly. Water is pumped through the heat exchangers to pick up the heat stored during summer. This water goes into the building and is used to warm the place up. After performing that task, it is pumped under the asphalt and its remaining heat keeps the road free of snow and ice.‎ ‎  [语篇解读] 文章介绍了怎样依靠外面的柏油马路表面在冬天给楼房供暖,在夏天给楼房降温。‎ ‎  4.Which of the following is true according to the first two paragraphs ?‎ ‎  A. Arian de Bondt got his idea from his boss.‎ ‎  B. Large, flat, black surfaces need to be built in cities.‎ ‎  C. The Dutch engineer' s system has been widely used.‎ ‎  D. Heat can also be collected from asphalt roads.‎ ‎  [解析] 细节判断题。由文章第二段最后一句话可以得出答案。‎ ‎  [答案] D ‎  5.For what purpose are the diving pipes used?‎ ‎  A. To absorb heat from the sun.‎ ‎  B. To store heat for future use.‎ ‎  C. To turn solar energy into heat energy.‎ ‎  D. To carry heat down below the surface.‎ ‎  [解析] 细节判断题。由文章第四段第一句话的内容可以得出答案。‎ ‎  [答案] D ‎  6.From the last paragraph we can learn that________.‎ ‎  A. some pipes have to be rearranged in winter ‎  B. the system can do more than warming up the building ‎  C. the exchangers will pick up heat from the street surface ‎  D. less heat may be collected in winter than in summer ‎  [解析] 推测判断题。由文章最后一段最后一句的内容可知,这一系统不仅能给楼房供暖,还可以使马路上的冰雪消融。‎ ‎  [答案] B ‎  7.What is most likely to be discussed in the paragraph that follows?‎ ‎  A. What we shall do if the system goes wrong.‎ ‎  B. What we shall do if there are no asphalt roads.‎ ‎  C. How the system cools the building in summer.‎ ‎  D. How the system collects heat in spring and autumn.‎ ‎  [解析] 文章结构题。前面谈到这一系统的工作原理以及冬天怎么给楼房供暖,下文应该讲的是夏天怎么给楼房降温。‎ ‎  [答案] C ‎(第29篇)‎ ‎  It's not easy being a teenager(13至19岁青少年)-nor is it easy being the parent of a teenager. You can make your child feel angry, hurt, or misunderstood by what you say without realizing it yourself. It is important to give your child the space he needs to grow while gently letting him know that you'll still be there for him when he needs you.‎ ‎  Expect a lot from your child, just not everything. Except for health and safety problems, such as drug use or careless driving, consider everything else open to discussion. If your child is unwilling to discuss something, don' t insist he tell you what' s on his mind. The more you insist, the more likely that he' ll clam_up. Instead, let him attempt to solve (解决) things by himself. At the same time, remind him that you're always there for him should he seek advice or help. Show respect for your teenager' s privacy (隐私). Never read his mail or listen in on personal conversations.‎ ‎  Teach your teenager that the family phone is for the whole family. If your child talks on the family ' s telephone for too long, tell him he can talk for 15 minutes, but then he must stay off the phone for at least an equal period of time. This not only frees up the line so that other family members can make and receive calls, but teaches your teenager moderation (节制). Or if you are open to the idea, allow your teenager his own phone that he pays for with his own pocket money or a parttime job.‎ ‎  [语篇解读] 本文向父母们介绍了一些教育青少年的可行的做法。‎ ‎  1.The main purpose of the text is to tell parents ________.‎ ‎  A. how to get along with a teenager ‎  B. how to respect a teenager ‎  C. how to understand a teenager ‎  D. how to help a teenager grow up ‎  [解析] 写作意图题。全文都是作者向父母提出的一些建议:第一段说要给孩子一定的空间;第二段说对孩子要有期望但是不能期望一切,要让孩子自己去解决问题,教孩子在遇到问题时,向父母求助;第三段说要教育孩子和家人分享电话。由此可见,文章的写作目的是给家长提建议,如何教育青少年子女。‎ ‎  [答案] D ‎  2.What does the phrase "clam up" in Paragraph 2 probably mean ?‎ ‎  A. become excited       B. show respect ‎  C. refuse to talk D. seek help ‎  [解析] 猜测词义题。由第二段内容可知,如果孩子不愿意讨论一些事情,不要去勉强他。你越是催促他,他就越不愿意说。短语"clam up(拒不开口)"与"refuse to talk"同义。‎ ‎  [答案] C ‎  3.What should parents do in raising a teenager according to the text?‎ ‎  A. Not allow him to learn driving or take drugs.‎ ‎  B. Give him advice only when necessary.‎ ‎  C. Let him have his own telephone.‎ ‎  D. Not talk about personal things with him.‎ ‎  [解析] 推理判断题。第二段中的"At the same time,remind him that you're always there for him should he seek advice or help"一句表明B项的陈述符合文章的内容。‎ ‎  [答案] B ‎(第30篇)‎ ‎  A few years ago,Paul Gerner began to gather a group of architects in Las Vegas to ask them what it would take to design a public school that used 50 percent less energy, cost much less to build and obviously improved student learning. "I think half of them fell off their chairs, "Gerner says.‎ ‎  Gerner manages school facilities (设施) for Clark County, Nevada,a district roughly the size of Massachusetts. By 2018,143,000 additional students will enter the already crowded publiceducation system. Gerner needs 73 new schools to house them. Four architecture teams have nearly finished designing primary school prototypes (样品) ;they plan to construct their schools starting in 2009. The district will then assess how well the schools perform, and three winners will copy those designs in 50 to 70 new buildings.‎ ‎  Green schools are appearing all over, but in Clark County,which stands out for its vastness, such aggressive targets are difficult because design requirements like more natural light for students go against the realities of a desert climate. "One of the biggest challenges is getting the right site orientation(朝向) ," Mark McGinty, a director at SH Architecture, says. His firm recently completed a high school in Las Vegas. "You have the same building, same set of windows, but if its orientation is incorrect and it faces the sun, it will be really expensive to cool."‎ ‎  Surprisingly, the man responsible for one of the most progressive greendesign competitions has doubts about ideas of ecofriendly buildings. "I don't believe in the new green religion," Gerner says."Some of the building technologies that you get are impractical. I'm interested in those that work. "But he wouldn't mind if some green features inspire students. He says he hopes to set up green energy systems that allow them to learn about the process of harvesting wind and solar power. "You never know what's going to start the interest of a child to study math and science,"he says.‎ ‎  [语篇解读] 创造性思维是我们每个人都应该有的,但是一切应该从实际出发,不切实际的想法也就如同空中楼阁。本文所涉及的主人公Gerner想创办一所绿色学校的想法遭到了很多人的质疑,他的设计要求连他自己也开始怀疑。‎ ‎  4.How did the architects react to Gerner's design requirements?‎ ‎  A. They lost balance in excitement.‎ ‎  B. They showed strong disbelief.‎ ‎  C. They expressed little interest.‎ ‎  D. They burst into cheers.‎ ‎  [解析] 推理判断题。从第一段的最后一句:"I think half of them fell of their chairs,"(一半人惊得从椅子上掉下来)推知:建筑师们对他的设计高度怀疑。‎ ‎  [答案] B ‎  5.Which order of steps is followed in carrying out the project?‎ ‎  A. Assessment-Prototype-Design-Construction.‎ ‎  B. Assessment-Design-Prototype-Construction.‎ ‎  C. Design-Assessment-Prototype-Construction.‎ ‎  D. Design-Prototype-Assessment-Construction.‎ ‎  [解析] 事实细节题。本题也可称作排序题。做题的关键是抓住时间顺序或其他次序。本文从绿色环保学校的设计(第一段),完成样品(第二段),评估(第三段)到建设(第四段)说明了建筑这所学校的难度。从而选择答案D。‎ ‎  [答案] D ‎  6.What makes it difficult to build green schools in Clark County?‎ ‎  A. The large size.‎ ‎  B. Limited facilities.‎ ‎  C. The desert climate.‎ ‎  D. Poor natural resources.‎ ‎  [解析] 事实细节题。第三段...such aggressive targets are difficult because design requirements like more natural light for students go against the realities off a desert climate.说明:在克拉克建筑绿色学校最大的困难就是沙漠性气候问题。‎ ‎  [答案] C ‎  7.What does Gerner think of the ideas of green schools?‎ ‎  A, They are questionable. B. They are out of date.‎ ‎  C. They are advanced. D. They are practical.‎ ‎  [解析] 推理判断题。根据最后一段Surprisingly,the man responsible for one of the most progressive greendesign competitions has doubts about ideas of ecofriendly buildings."I don't believe in the new green religion,"Gerner says.可以推测,Gerner对建立此学校持有怀疑态度。‎ ‎  [答案] A ‎(第31篇)‎ ‎  GUATEMALA CITY(Reuters)-A fish that lives in mangrove swamps(红树沼泽) across the Americas can live out of water for months at a time, similar to how animals adapted (适应)to land millions of years ago, a new study shows.‎ ‎  The Mangrove Rivulus, a type of small killifish, lives in small pools of water in a certain type of empty nut or even old beer cans in the mangrove swamps of Belize, the United States and Brazil. When their living place dries up, they live on the land in logs (圆木), said Scott Taylor,a researcher at the Brevard Endangered Lands Program in Florida.‎ ‎  The fish ,whose scientific name is Rivulus marmoratus, can grow as large as three inches. They group together in logs and breathe air through their skin until they can find water again.‎ ‎  The new scientific discovery came after a trip to Belize.‎ ‎  "We kicked over a log and the fish just came crowding out,"Taylor told Reuters in neighboring Guatemala by telephone. He said he will make his study on the fish known to the public in an American magazine early next year.‎ ‎  In lab tests, Taylor said he found the fish can live up to 66 days out of water without eating.‎ ‎  Some other fish can live out of water for a short period of time.The walking catfish found in Southeast Asia can stay on land for hours at a time, while lungfish found in Australia, Africa and South America can live ont of water,but only in an inactive state. But no other known fish can be out of water as long as the Mangrove Rivulus and remain active, according to Patricia Wright, a biologist at Canada's University of Guelph.‎ ‎  Further studies of the fish may tell how animals changed over time.‎ ‎  "These animals live in conditions similar to those that existed millions of years ago ,when animals began making the transition (过渡) from water onto land, "Wright said.‎ ‎  [语篇解读] 最新发现:有一种鱼,即使离开水也能存活。‎ ‎  1.The Mangrove Rivulus is a type of fish that ________.‎ ‎  A. likes eating nuts ‎  B. prefers living in dry places ‎  C. is the longest living fish on earth ‎  D. cau stay alive for two months out of water ‎  [解析] 事实细节题。依据在第六段,Taylor说这种鱼离水后能活66天,也就是两个多月。‎ ‎  [答案] D ‎  2.Who will write up a report on mangrove Rivulus?‎ ‎  A. Patricia Wright.‎ ‎  B. Researchers in Guatemala.‎ ‎  C. Scientists from Belize.‎ ‎  D. Scott Taylor.‎ ‎  [解析] 推理判断题。第五段最后一句话告诉我们:明年Scott Taylor将要把对于这种鱼的研究在一份美国杂志上公布于众。‎ ‎  [答案] D ‎  3.According to the text, lungfish can ________.‎ ‎  A. breathe through its skin ‎  B. move freely on dry land ‎  C. remain alive out of water ‎  D. be as active on land as in water ‎  [解析] 事实细节题。第七段第二句话告诉我们:lungfish离开水也可以存活。‎ ‎  [答案] C ‎  4.What can we say about the discovery of Mangrove Rivulus?‎ ‎  A. It was made quite by accident.‎ ‎  B. It was based on a lab test of sea life.‎ ‎  C. It was supported by an American magazine.‎ ‎  D. It was helped by Patricia Wright.‎ ‎  [解析] 推理判断题。第五段Taylor的话告诉我们:发现这种鱼是偶然的。‎ ‎  [答案] A ‎(第32篇)‎ ‎   "Old wives' tales" are beliefs passed down from one generation to another. For example,most of us remember our parents' telling us to eat more of certain foods or not to do certain things. Is there any truth in these teachings? Some of them agree with present medical thinking,but others have not passed the test of time.‎ ‎  Did your mother ever tell you to eat your carrots because they are good for your eyes? Scientists now report that eating carrots can help prevent a serious eye disease called macular degeneration. Eating just one carrot a day can reduce the possibility of getting this disease by 40%. Garlic(蒜)is good for you,too. It can kill the type of virus that causes colds.‎ ‎  Unfortunately, not all of Mom's advice passed the test of medical studies. For example, generations of children have been told not to go swimming within an hour after eating. But research suggests that there is no danger in doing so. Do sweets cause tooth problems?Well, yes and no. Sticky sweets made with grains tend to cause more problems than sweets made with simple sugars.‎ ‎  Even though science can tell us that some of our traditional beliefs don't hold_water,_there is still a lot of truth in the old wives' tales. After all, much of this knowledge has been accumulated (积累) from thousands of years of experience in family health care. We should respect this body of knowledge even as we search for clear scientific support to prove it true or false.‎ ‎  [语篇解读] 我们大多数人都记得父母告诉我们要吃某些食物或者不要去做某些事情。这些告诫是对的吗?实际上,其中一些是与现在的医学思想相符合的,而另外一些却经不起考验。文章举了例子,并指出即便当我们寻找明确的科学证据来证明它的对错时,我们也应该尊重这类知识。‎ ‎  5.Which of the following is TRUE according to the text?‎ ‎  A. Eating garlic is good for our eyes.‎ ‎  B. Sticky sweets are damaging to our teeth.‎ ‎  C. Swimming after a meal is dangerous.‎ ‎  D. Carrots prevent people from catching colds.‎ ‎  [解析] 事实细节题。从第三段最后一句Sticky sweels made with grains tend to cause more problems than sweets made with simple sugars.可以看出,sticky sweets的确可以对牙齿造成危害。‎ ‎  [答案] B ‎  6.The author develops the third paragraph mainly________.‎ ‎  A. by cause and effect     B. by order in space ‎  C. by order in time D. by examples ‎  [解析] 推理判断题。第三段第一句是主题句。随后,举了饭后一小时内游泳和吃糖这两个例子来证明主题句。因此,这一段是通过举例来构成段落的。‎ ‎  [答案] D ‎  7.The phrase" hold water" in the last paragraph most probably means"________".‎ ‎  A. to be believable B. to be valuable ‎  C. to be admirable D. to be suitable ‎  [解析] 猜测词义题。从最后一段第一句可知,虽然科学告诉我们有一些传统观念不可信,但是这些老妈妈们的说法还是有一些正确的地方。hold water指"可信的"。‎ ‎  [答案] A ‎  8.What is the author's attitude toward"old wives' tales"in the text?‎ ‎  A. Subjective. B. Objective.‎ ‎  C. Dissatisfied. D. Curious.‎ ‎  [解析] 推理判断题。纵观全文,作者既指出了一些传统说法的正确性,又说明了一些传统观念没有科学依据。尤其是文中最后一段作者表明了对这些观念的尊重,因此可以看出作者的态度是客观的。‎ ‎  [答案] B ‎(第33篇)‎ ‎  Even at school there had been an unhealthy competition between George and Richard.‎ ‎  "I' ll be the first millionaire in Coleford!" Richard used to boast.‎ ‎  "And you'll be sorry you knew me," George would reply "because I'll be the best lawyer in town!"‎ ‎  George never did become a lawyer and Richard never made any money. Instead both men opened bookshops on opposite sides of Coleford High Street. It was hard to make money from books, which made the competition between them worse.‎ ‎  Then Richard married a mysterious girl. The couple spent their honeymoon on the coast - but Richard never came back.The police found his wallet on a deserted beach but the body was never found. He must have drowned.‎ ‎  Now with only one bookshop in town, business was better for George. But sometimes he sat in his narrow, old kitchen and gazed out of the dirty window, thinking about his former rival(竞争对手). Perhaps he missed him?‎ ‎  George was very interested in old dictionaries. He' d recently found a collector in Australia who was selling a rare first edition. When the parcel arrived, the book was in perfect condition and George was delighted. But while he was having lunch, George glanced at the photo in the newspaper that the book had been wrapped in. He was astonished - the smiling face was older than he remembered but unmistakable! Trembling, George started reading.‎ ‎  "Bookends have bought ten bookstores from their rivals Dylans. The company, owned by multimillionaire Richard Pike, is now the largest bookseller in Australia."‎ ‎  [语篇解读] 本文讲述了Richard和George竞争的故事。他们上学时就扬言要分别成为百万富翁和最好的律师,结果Richard实现了自己的梦想,George却没有。‎ ‎  1.George and Richard were ________at school.‎ ‎  A. roommates        B. good friends ‎  C. competitors D. booksellers ‎  [解析] 细节理解题。根据文章第一段"Even at school there had been an unhealthy competition between George and Richard"可知两人是竞争对手。‎ ‎  [答案] C ‎  2.How did George feel about Richard after his disappearance?‎ ‎  A. He envied Richard's marriage.‎ ‎  B. He thought of Richard from time to time.‎ ‎  C. He felt lucky with no rival in town.‎ ‎  D. He was guilty of Richard' s death.‎ ‎  [解析] 细节理解题。根据第六段中的"But sometimes he sat in his narrow,old kitchen and gazed out of the dirty window,thinking about his former rival."可知George有时会想起Richard,故答案为B。‎ ‎  [答案] B ‎  3.George got information about Richard from ________.‎ ‎  A. a dictionary collector in Australia ‎  B. the latter's rivals Dylans ‎  C. a rare first edition of a dictionary ‎  D. the wrapping paper of a book ‎  [解析] 细节理解题。根据倒数第二段中的"George glanced at the photo in the newspaper that the book had been wrapped in..."可知George是在包书的报纸上得到了Richard的消息。故答案为D。‎ ‎  [答案] D ‎  4.What happened to George and Richard in the end?‎ ‎  A. Both George and Richard became millionaires.‎ ‎  B. Both of them realized their original ambitions.‎ ‎  C. George established a successful business while Richard was missing.‎ ‎  D. Richard became a millionaire while George had no great success.‎ ‎  [解析] 根据文章最后一段可知Richard最后成功了,而George依然开着书店,没有成为百万富翁,也没有成为有名的律师,A、B、C三项的表述都错误,所以答案为D。‎ ‎  [答案] D ‎(第34篇)‎ ‎  Remembering names is an important social skill. Here are some ways to master it.‎ ‎  Recite and repeat in conversation.‎ ‎  When you hear a person' s name, repeat it. Immediately say it to yourself several times without moving your lips. You could also repeat the name in a way that does not sound forced or artificial.‎ ‎  Ask the other person to recite and repeat.‎ ‎  You can let other people help you remember their names.After you've been introduced to someone,ask that person to spell the name and pronounce it correctly for you. Most people will be pleased by the effort you're making to learn their names.‎ ‎  Admit you don' t know.‎ ‎  Admitting that you can' t remember someone' s name can actually make people relaxed. Most of them will feel sympathy if you say, "I' m working to remember names better. Yours is right on the tip of my tongue. What is it again?"‎ ‎  Use associations.‎ ‎  Link each person you meet with one thing you find interesting or unusual. For example, you could make a mental note:"Vicki Cheng - tall, black hair." To reinforce (加强) your associations, write them on a small card as soon as possible.‎ ‎  Limit the number of new names you learn at one time.‎ ‎  When meeting a group of people, concentrate on remembering just two or three names. Free yourself from remembering everyone. Few of the people in mass introductions expect you to remember their names. Another way is to limit yourself to learning just first names. Last names can come later.‎ ‎  Go early.‎ ‎  Consider going early to conferences, parties and classes.Sometimes just a few people show up on time. That's fewer names for you to remember. And as more people arrive, you can hear them being introduced to others - an automatic review for you.‎ ‎  [语篇解读] 文章讲述了记住别人的名字是一项重要的社交技能,并就此提供了一些方法技巧。‎ ‎  5.How will most people feel when you try hard to remember their names?‎ ‎  A. They will be moved.‎ ‎  B. They will be annoyed.‎ ‎  C. They will be delighted.‎ ‎  D. They will be discouraged.‎ ‎  [解析] 细节理解题。由文章提到的第二种方法中的"Most people will be pleased by the effort you're making to learn their names"内容就可得出答案,pleased=delighted。‎ ‎  [答案] C ‎  6.If you can't remember someone's name, you may ________.‎ ‎  A. tell him the truth B. tell him a white lie ‎  C. ask him for pity D. ask others to help you ‎  [解析] 细节判断题。由文章提到的第三种方法的内容,即可得出答案。‎ ‎  [答案] A ‎  7.When you meet a group of people, it is better to remember________.‎ ‎  A. all their names ‎  B. a couple of names first ‎  C. just their last names ‎  D. as many names as possible ‎  [解析] 细节判断题。由文章提到的第五种方法中的"When meeting a group of people,concentrate on remembering just two or three names."内容即可得出答案。‎ ‎  [答案] B ‎  8.What does the text mainly tell us?‎ ‎  A. Tips on an important social skill.‎ ‎  B. Importance of attending parties.‎ ‎  C. How to make use of associations.‎ ‎  D. How to recite and repeat names.‎ ‎  [解析] 主旨大意题。由文章首段即可得出答案。‎ ‎  [答案] A ‎(第35篇)‎ ‎  We have met the enemy, and he is ours. We bought him at a pet shop. When monkeypox, a disease usually found in the African rain forest, suddenly turns up in children in the American Midwest, it' s hard not to wonder if the disease that comes from foreign animals is homing in on human beings. "Most of the infections (感染) we think of as human infections ‎ started in other animals," says Stephen Morse, director of the Center for Public Health Preparedness at Columbia University.‎ ‎  It' s not just that we' re going to where the animals are;we' re also bringing them closer to us. Popular foreign pets have brought a whole new disease to this country. A strange illness killed Isaksen' s pets, and she now thinks that keeping foreign pets is a bad idea. "I don' t think it' s fair to have them as pets when we have such a limited knowledge of them," says Isaksen.‎ ‎  "Laws allowing these animals to be brought in from deep forest areas without stricter control need changing," says Peter Schantz. Monkeypox may be the_wakeup_call. Researchers believe infected animals may infect their owners. We know very little about these new diseases. A new bug (病毒) may be kind at first. But it may develop into something harmful (有害的). Monkeypox doesn' t look a major infectious disease. But it is not impossible to pass the disease from person to person.‎ ‎  [语篇解读] 饲养宠物可能被传染疾病。猴痘本是非洲雨林中的一种疾病,现在突然出现在美国西部的孩子们的身上。我们认为的大多数只有人类才会感染的疾病其实就是来自动物。国家要制定严格的法律,控制外来动物把疾病传播给人类。‎ ‎  1.We learn from Paragraph 1 that the pet sold at the shop may ________.‎ ‎  A. come from Columbia ‎  B. prevent us from being infected ‎  C. enjoy being with children ‎  D. suffer from monkeypox ‎  [解析] 细节理解题。根据第一段的第一、二两句话可知,我们从宠物商店购买的动物身上携带疾病,这种动物能将之传播给人类。下文以猴痘为例说明作者的观点。‎ ‎  [答案] D ‎  2.Why did Isaksen advise people not to have foreign pets?‎ ‎  A. They attack human beings.‎ ‎  B. We need to study native animals.‎ ‎  C. They can' t live out of the rain forest.‎ ‎  D. We do not know much about them yet.‎ ‎  [解析] 推理判断题。从第二段中的"A strange illness killed Isaksen's pets,and she now thinks that keeping foreign pets is a bad idea.'I don't think it's fair to have them as pets when we have such a limited knowledge of them"可知答案为D。‎ ‎  [答案] D ‎  3.What does the phrase "the wakeup call" in Paragraph 3 most probably mean?‎ ‎  A. a new disease      B. a clear warning ‎  C. a dangerous animal D. a morning call ‎  [解析] 词义猜测题。根据文章的内容可知,猴痘是一种从非洲的动物身上传播到人类身上的疾病,这种情况给人类带来了一种警示:不要随意把野外的动物带回家中饲养,以免使宠物主人感染疾病。"the wakeup call"表示"一种警示"。‎ ‎  [答案] B ‎  4.The text suggests that in the future we________.‎ ‎  A. may have to fight against more new diseases ‎  B. may easily get infected by diseases from dogs ‎  C. should not be allowed to have pets ‎  D. should stop buying pets from Africa ‎  [解析] 推理判断题。根据最后一段的最后三句可知,人类今后可能面临更多疾病的挑战。‎ ‎  [答案] A ‎  ‎ ‎   (第36篇)‎ ‎  Runners in a relay(接力)race pass a stick in one direction. However, merchants passed silk, gold, fruit, and glass along the Silk Road in more than one direction. They earned their living by traveling the famous Silk Road.‎ ‎  The Silk Road was not a simple trading network. It passed through thousands of cities and towns. It started from eastern China, across Central Asia and the Middle East, and ended in the Mediterranean Sea. It was used from about 200 B. C. to about A. D. 1300, when sea travel offered new routes (路线). It was sometimes called the world's longest highway. However,the Silk Road was made up of many routes, not one smooth path. They passed through what are now 18 countries. The routes crossed mountains and deserts and had many dangers of hot sun, deep snow and even battles. Only experienced traders could return safe.‎ ‎  The Silk Road got its name from its most prized product. Silk could be used like money to pay taxes or buy goods. But the traders carried more than just silk. Gold, silver, and glass from Europe were much found in the Middle East and Asia.Horses traded from other areas changed farming practices in China. Indian merchants traded salt and other valuable goods.Chinese merchants traded paper, which produced an immediate effect on the West. Apples traveled from central Asia to Rome.The Chinese had learned to graft(嫁接)different trees together to make new kinds of fruit. They passed this science on to others, including the Romans. The Romans used grafting to grow the apple. Trading along the Silk Road led to worldwide business 2000 years before the World Wide Web.‎ ‎  The people along the Silk Road did not share just goods. They also shared their beliefs. The Silk Road provided pathways for learning,diplomacy(外交),and religion (宗教).‎ ‎  5. It's probable that traders along the Silk Road needed________.‎ ‎  A. to remember the entire trade route ‎  B. to know the making of products ‎  C. to receive certain special training ‎  D. to deal with a lot of difficulties ‎  [解析] 细节理解题。根据"The routes crossed mountains and deserts and had many dangers of hot sun,deep snow and even battles.Only experienced traders could return safe."可知D项正确。‎ ‎  [答案] D ‎  6. The Silk Road became less important because________.‎ ‎  A. it was made up of different routes ‎  B. silk trading became less popular ‎  C. sea travel provided easier routes ‎  D. people needed fewer foreign goods ‎  [解析] 细节理解题。根据"It was used from about 200B.C.,to about AD1300,when sea travel offered new routes"能找到原因是"sea travel provided easier routes"。‎ ‎  [答案] C ‎  7. New technologies could travel along the Silk Road because people________.‎ ‎  A. learned from one another ‎  B. shared each other's beliefs ‎  C. traded goods along the route ‎  D. earned their living by traveling ‎  [解析] 细节理解题。根据"The people along the Silk Road did not share just goods.They also shared their beliefs.The Silk Road provided pathways for learning,diplomacy,and religion"能看出人们彼此之间能学习一些东西。‎ ‎  [答案] A ‎  8. What is the best title for the passage?‎ ‎  A. The Silk Road:Past and Present ‎  B. The Silk Road: East Meets West ‎  C. The Silk Road:Routes Full of Dangers ‎  D. The Silk Road:Pathways for Learning ‎  [解析] 主旨大意题。根据"The Silk Road was not a simple trading network.It passed through thousands of cities and towns.It started from eastern China,across Central Asia and the Middle East,and ended in the Mediterranean Sea."可知The Silk Road:East Meets West是最佳标题。‎ ‎  [答案] B ‎  ‎ ‎(第37篇)‎ ‎  Four people in England ,back in 1953, stared at Photo 51.It wasn' t much - a picture showing a black X. But three of these people won the Nobel Prize for figuring out what the ‎ photo really showed - the shape of DNA. The discovery brought fame and fortune to scientists James Watson, Francis Crick,and Maurice Wilkins. The fourth, the one who actually made the picture, was left out.‎ ‎  Her name was Rosalind Franklin. "She should have been up there," says historian Mary Bowden." If her photos hadn' t been there, the others couldn' t have come up with the structure." One reason Franklin was missing was that she had died of cancer four years before the Nobel decision.But now scholars doubt that Franklin was not only robbed of her life by disease but robbed of credit by her competitors.‎ ‎  At Cambridge University in the 1950s, Watson and Crick tried to make models by cutting up shapes of DNA' s parts and then putting them together. In the meantime, at King' s College in London, Franklin and Wilkins shone Xrays at the molecule (分子). The rays produced patterns reflecting the shape.‎ ‎  But Wilkins and Franklin's relationship was a lot rockier than the celebrated teamwork of Watson and Crick. Wilkins thought Franklin was hired to be his assistant. But the college actually employed her to take over the DNA project.‎ ‎  What she did was produce Xray pictures that told Watson and Crick that one of their early models was inside out. And she was not shy about saying so. That angered Watson, who attacked her in return," Mere inspection suggested that she would not easily bend. Clearly she had to go or be put in her place."‎ ‎  As Franklin's competitors, Wilkins, Watson and Crick had much to gain by cutting her out of the little group of researchers, says historian Pnina AbirAm. In 1962 at the Nobel Prize awarding ceremony, Wilkins thanked 13 colleagues by name before he mentioned Franklin. Watson wrote his book laughing at her. Crick wrote in 1974 that "Franklin was only two steps away from the solution."‎ ‎  No, Franklin was the solution." She contributed more than any other player to solving the structure of DNA. She must be considered a codiscoverer," AbirAm says. This was backed up by Aaron Klug, who worked with Franklin and later won a Nobel Prize himself. Once described as the "Dark Lady of DNA", Franklin is finally coming into the light.‎ ‎  [语篇解读] 谁发现了DNA,仅仅只有James Watson,Francis Crick和Maurice Wilkins吗?本文作者告诉我们,功劳也有Rosalind Franklin的一份。‎ ‎  1.What is the text mainly about?‎ ‎  A. The disagreements among DNA researchers.‎ ‎  B. The unfair treatment of Franklin.‎ ‎  C. The process of discovering DNA.‎ ‎  D. The race between two teams of scientists.‎ ‎  [解析] 主旨大意题。通读全文,可知Franklin在DNA的发现中也做出了重要贡献,但却由于某些原因而不为人知。由此答案选B项:她受到了不公正的对待。‎ ‎  [答案] B ‎  2.Watson was angry with Franklin because she________.‎ ‎  A. took the lead in the competition ‎  B. kept her results from him ‎  C. proved some of his findings wrong ‎  D. shared her data with other scientists ‎  [解析] 逻辑推理题。从第五段前两句可知,Franklin直言不讳地指出他们犯的错误,这引起了Watson的不满。答案选C。‎ ‎  [答案] C ‎  3.Why is Franklin described as "Dark Lady of DNA" ?‎ ‎  A. She developed pictures in dark labs.‎ ‎  B. She discovered the black X- the shape of DNA.‎ ‎  C. Her name was forgotten after her death.‎ ‎  D. Her contribution was unknown to the public.‎ ‎  [解析] 逻辑推理题。从全文可知,作者认为Franklin的贡献被忽略掉了,她的贡献理应得到人们的承认。所以答案选D。‎ ‎  [答案] D ‎  4.What is the writer's attitude toward Wilkins ,Watson and Crick?‎ ‎  A. Disapproving.      B. Respectful.‎ ‎  C. Admiring. D. Doubtful.‎ ‎  [解析] 态度推测题。从全文知,作者肯定了Franklin的贡献,对另外三个科学家的行为表示"不满,不赞成",所以答案选A。‎ ‎  [答案] A ‎(第38篇)‎ ‎  This was no ordinary class. The students who came together were all science or engineering professors at Cornell University. They had interrupted their research to accept an invitation to take part in an unusual experiment: "an interesting week of poetry." This class was part of a study to answer the questions: Why is science difficult for many nonscience students? What can teachers learn about teaching if they take a class that is not in their field?‎ ‎  The students in the poetry class listened to lectures and took notes. They had reading tasks and had to write three short papers. All students noticed one thing-the importance of spoken words. In science and engineering classes, the instructors put tables and drawings on the blackboard. But in this poetry class, the instructors just talked. They didn't write anything on the board.‎ ‎  The scientists and engineers noticed one similarity between science and poetry. In both subjects, students need to find layers(层次) of meaning. Some layers are simple, clear, and on the surface;other layers are deeper and more difficult. This search for different levels of meaning ‎ doesn't happen much in undergraduate (本科) science classes, but it is important later, in graduate school. And it is always important in humanities(人文学科).‎ ‎  Both the poetry instructors and their students learned something about teaching from this experience. One poetry instructor, for example, now sees the importance of using careful,clear definitions (定义)when he explains a poem. He also plans to be more informative as he teaches. Most of the scientists agreed on several points. First, humanities classes might help science students to see patterns and decide which information is important. Second, the poetry class was fun. One engineer decided, "We need to change the way we teach engineering to make it an enjoyable experience for students."‎ ‎  But perhaps the most important result of the experience was this:All of the professors began to think about how they teach and how they can teach better.‎ ‎  5. What do we know about this unusual class?‎ ‎  A. The teachers did lots of writing on the board.‎ ‎  B. The teachers were invited to attend several lectures.‎ ‎  C. The students were professors from a university.‎ ‎  D. The students were studying science and humanities.‎ ‎  [解析] 细节理解题。根据"This was no ordinary class.The students who came together were all science or engineering professors at Cornell University"可知C项正确。‎ ‎  [答案] C ‎  6. The experiment was designed to find out________.‎ ‎  A. how to teach the students in the science class ‎  B. whether poetry is difficult for science students ‎  C. what to be taught in the humanities class ‎  D. why many humanities students find science hard ‎  [解析] 细节理解题。根据"why is science difficult for many nonscience students?"找到答案。‎ ‎  [答案] D ‎  7. Finding levels of meaning is________.‎ ‎  A. important for graduate students in humanities ‎  B. difficult for graduate students in humanities ‎  C. common for undergraduate students in science ‎  D. easy for undergraduate students in science ‎  [解析] 细节理解题。根据第三段最后一句"And it is always important in humanities"可知选A项。‎ ‎  [答案] A ‎  8. What did the science professors learn after the experiment?‎ ‎  A. They should change the way they teach.‎ ‎  B. A poem could be explained in clear definitions.‎ ‎  C. A poetry class could be more informative.‎ ‎  D. Their teaching was an enjoyable experience.‎ ‎  [解析] 细节理解题。由"But perhaps the most important result of the experience was this:All of the professors began to think about how they teach and how they can teach better."我们可知道全文的中心思想。‎ ‎  [答案] A ‎(第39篇)‎ ‎  When I was going home to India last year, I called up my mother to ask if she wanted anything from China.‎ ‎  When India had not opened up its markets to the world, I carried suitcase loads of dark glasses and jeans. Thankfully,we can get all these anywhere in India now.‎ ‎  Still, her answer surprised me," Green tea."‎ ‎  As long as I can remember she didn' t even drink Indian tea.‎ ‎  I dutifully bought a big packet of Longjing and headed home to hear the story. My mother and her brother, both regular newspaper readers, believed that Chinese green tea was the wonder drug for all illnesses.‎ ‎  At the turn of the century, China was not really familiar to the average Indian. It was a strange country.‎ ‎  How things change! And how soon!‎ ‎  Now every town of any size seems to have a "China Market". And everyone is talking about China.‎ ‎  The government of India has planned to send a team to China to see how things are done. A minister once said that India must open the doors for more foreign investment(投资) and such a step would "work wonders as it did for China".‎ ‎  But it'_s_a_twoway_street. I just heard about a thousand Shenzhen office workers who have gone to Bangalore to train in software. Meanwhile, all the Indian IT majors are setting up a strong presence in China.‎ ‎  No wonder that trade, which was only in the millions just ten years ago, was expected to hit about US $15 billion for last year and US $20 billion by 2008, a goal set by both governments.‎ ‎  No wonder, my colleague wrote some weeks ago about this being the SinoIndian (中印)century as the two countries started on January 1 the SinoIndian Friendship Year.‎ ‎  But what is still a wonder to me is my mother drinking Chinese tea.‎ ‎  [语篇解读] 文章主要讲述了一个印度人对中国的印象:中国的发展令他惊讶不已。另外,中国和印度相互合作,实现了双赢。‎ ‎  1.Why did the mother ask for Chinese green tea?‎ ‎  A. She was tired of Indian tea.‎ ‎  B. She had a son working in China.‎ ‎  C. She believed it had a curing effect.‎ ‎  D. She was fond of Chinese products.‎ ‎  [解析] 根据第五段最后一句话可知答案。‎ ‎  [答案] C ‎  2.What does the author mean by "it' s a twoway street" in Paragraph 10?‎ ‎  A. China and India have different traffic rules.‎ ‎  B. Tea trade works wonders in both India and China.‎ ‎  C. Chinese products are popular in both China and India.‎ ‎  D. The exchanges between India and China benefit both.‎ ‎  [解析] 结合上下文可知,这句话的意思是"中国和印度互惠互利,共同发展"。‎ ‎  [答案] D ‎  3.What do we know about the Indian IT industry?‎ ‎  A. It will move its head office to Shenzhen.‎ ‎  B. It is seeking further development in China.‎ ‎  C. It has attracted an investment of US $15 billion.‎ ‎  D. It caught up with the US IT industry in 2008.‎ ‎  [解析] 由第十段可知,B项正确。‎ ‎  [答案] B ‎  4.In the text the author expresses________.‎ ‎  A. his concern for his mother' s health ‎  B. his support for drinking Chinese green tea ‎  C. his surprise at China' s recent development ‎  D. his wonder at the growth of India' s IT industry ‎  [解析] 综观全文可知,作者对中国的发展变化惊讶不已。因此C项正确。‎ ‎  [答案] C ‎(第40篇)‎ ‎  People believe that climbing can do good to health. Where can you learn the skill of climbing then? If you think that you have to go to the mountains to learn how to climb, you're wrong. Many Americans are learning to climb in city gyms(体育馆). Here,people are learning on special climbing walls. The climbing wall goes straight up and has small holding places for hands and feet.‎ ‎  How do people climb the wall? To climb,you need special shoes and a harness (保护带) around your chest to hold you.There are ropes(绳索)tied to your harness. The ropes hold you in place so that you don't fall.A beginner's wall is usually about 15 feet high, and you climb straight up. There are small pieces of metal that stick out for you to stand on and hold on to.‎ ‎ Sometimes it's easy to see the next piece of metal. Sometimes, it's not. The most difficult part is to control your fear.It's normal for humans to be afraid of falling, so it's difficult not to feel fear. But when you move away from the wall, the harness and the ropes hold you,and you begin to feel safe. You move slowly until you reach the top.‎ ‎  Climbing attracts people because it's good exercise for almost everyone. You use your whole body, especially your arms and legs.This sport gives your body a complete workout. When you climb,both your mind and your body can become stronger.‎ ‎  5. What can we infer from the passage?‎ ‎  A. People are fairly interested in climbing nowadays.‎ ‎  B. It is impossible to build up one's body by climbing.‎ ‎  C. People can only learn the skill of climbing outdoors.‎ ‎  D. It is always easy to see holding places in climbing.‎ ‎  [解析] 推理判断题。由本文第一段中"People believe that climbing can do good to health.Where can you learn the skill of climbing then"和第三段中"Climbing attracts people because it's good exercise for almost everyone."可推断出答案。‎ ‎  [答案] A ‎  6. The most difficult thing to do in wall climbing is________.‎ ‎  A. to tie ropes to your harness ‎  B. to control your fear ‎  C. to move away from the wall ‎  D. to climb straight up ‎  [解析] 细节理解题。根据第二段"The most difficult part is to control your fear."可以推断出应该控制一下恐惧心理。‎ ‎  [答案] B ‎  7. The word "workout" underlined in the last paragraph most probably means________.‎ ‎  A. settlement       B. exercise ‎  C. excitement D. tiredness ‎  [解析] 词义猜测题。由文中最后一段最后一句"When you climb,both your mind and your body can become stronger."可知,该词意为"锻炼"。‎ ‎  [答案] B ‎  8. Why does the author write this passage?‎ ‎  A. To tell people where to find gyms.‎ ‎  B. To prove the basic need for climbing.‎ ‎  C. To encourage people to climb mountains.‎ ‎  D. To introduce the sport of wall climbing.‎ ‎  [解析] 主旨大意题。通读全文可以判断出作者主要介绍了wall climbing这项运动。‎ ‎  [答案] D ‎(第41篇)‎ ‎  A year ago August, Dave Fuss lost his job driving a truck for a small company in west Michigan. His wife, Gerrie, was still working in the local school cafeteria, but work for Dave was scarce, and the price of everything was rising. The Fusses were at risk of joining the millions of Americans who have lost their homes in recent years. Then Dave and Gerrie received a timely gift - $ 7,000, a legacy (遗产) from their neighbors Ish and Arlene Hatch, who died in an accident. "It really made a difference when we were going under financially," says Dave.‎ ‎  But the Fusses weren't the only folks in Alto and the neighboring town of Lowell to receive unexpected legacy from the Hatches. Dozens of other families were touched by the Hatches' generosity. In some cases, it was a few thousand dollars; in others, it was more than $100,000.‎ ‎  It surprised nearly everyone that the Hatches had so much money, more than $3 million - they were an elderly couple who lived in an old house on what was left of the family farm.‎ ‎  Children of the Great Depression, Ish and Arlene were known for their habit of saving. They thrived on (喜欢) comparison shopping and would routinely go from store to store,checking prices before making a new purchase.‎ ‎  Through the years, the Hatches paid for local children to attend summer camp when their parents couldn' t afford it."Ish and Arlene never asked if you needed anything," says their friend Sandy Van Weelden, "They could see things they could do to make you happier, and they would do them."‎ ‎  Even more extraordinary was that the Hatches had their farmland distributed. It was the Hatches' wish that their legacy - a legacy of kindness as much as one of dollars and cents -should enrich the whole community (社区) and last for generations to come.‎ ‎  Neighbors helping neighbors - that was Ish and Arlene Hatch' s story.‎ ‎  [语篇解读] 本文主要介绍了一对关心邻里、无私奉献的夫妻。他们把自己的财产留给需要的人,强调人人互助,世界就会更美好。‎ ‎  1.According to the text, the Fusses ________.‎ ‎  A. were employed by a truck company ‎  B. were in financial difficulty ‎  C. worked in a school cafeteria ‎  D. lost their home ‎  [解析] 事实细节题。从第一段前三句话可以看出,Fuss夫妇现在正处于经济困境中,所以B为正确答案。A、C项与文中有关细节不符,D项在文中没有信息支持。‎ ‎  [答案] B ‎  2.Which of the following is true of the Hatches?‎ ‎  A. They had their children during the Great Depression.‎ ‎  B. They left the family farm to live in an old house.‎ ‎  C. They gave away their possessions to their neighbors.‎ ‎  D. They helped their neighbors to find jobs.‎ ‎  [解析] 事实细节题。第四段第一句话说Hatch夫妇是在大萧条时期出生的,而不是他们在大萧条时期有了孩子,排除A;第三段提到他们的老房子位于他们家的农场里,B错误;第二段提到他们帮助有困难的邻居们,但并未提及帮他们找工作,D错误。C项说他们把自己的财产捐赠给邻居,符合文意,故答案为C。‎ ‎  [答案] C ‎  3.Why would the Hatches routinely go from store to store?‎ ‎  A. They decided to open a store.‎ ‎  B. They wanted to save money.‎ ‎  C. They couldn' t afford expensive things.‎ ‎  D. They wanted to buy gifts for local kids.‎ ‎  [解析] 推理判断题。第四段提到他们有节约的习惯,买东西之前喜欢一个商店一个商店地比较价格,显然是为了省钱,故答案为B。A,他们决定开个商店;C,他们买不起贵的东西;D,他们想给当地的孩子买礼物,都明显与文意不符。‎ ‎  [答案] B ‎  4.According to Sandy Van Weelden, the Hatches were________.‎ ‎  A. understanding     B. optimistic ‎  C. childlike D. curious ‎  [解析] 推理判断题。从文章第五段Sandy Van Weelden的话中可知,the Hatches从来不问你需要什么,而是能够发现你需要的东西,然后就来帮助你,这说明他们非常善解人意,故答案为A。B表示乐观的;C表示孩子气的,天真烂漫的;D表示好奇的,明显不符合文意。‎ ‎  [答案] A ‎  5.What can we learn from the text?‎ ‎  A. The community of Alto was poor.‎ ‎  B. The summer camp was attractive to the parents.‎ ‎  C. Sandy Van Weelden got a legacy from the Hatches.‎ ‎  D. The Hatches would like the neighbors to follow their example.‎ ‎  [解析] 推理判断题。A项错在这个地区很贫困,文章只是说这里有经济困难的人,并不是说这里是一个贫困地区;B项错在parents上;C项说Sandy Van Weelden得到了Hatches夫妇的一份遗产,文章只是提到他对他们的评价,并未说他得到遗产之事;从文章倒数第二段第二句话以及最后一段可以看出,Hatches夫妇的愿望就是他们的遗产能够为社区以及子孙后代造福,邻里之间能够互相帮助,故答案为D。‎ ‎  [答案] D ‎(第42篇)‎ ‎  B ‎  Parttime Front Desk Position ‎  Kirchoff, Inc. , a book development company, is looking for a parttime front desk office worker. This position is perfect for a person who is cheerful, dependable,and pleasant to work with. Also, you should be able to welcome guests, redirect phone calls, and take messages. More importantly, you can stay cool under pressure. You are expected to work 5∶006∶00 pm weekdays. You need to fill in some forms if you are interested.‎ ‎  Forms can be collected at ‎  Kirchoff, Inc.‎ ‎  866 United Nations Plaza, # 525‎ ‎  New York,NY 10017‎ ‎  ‎ ‎  Important Points to Remember When Swimming ‎  · Wait at least an hour after meals.‎ ‎  · Follow the advice of lifeguards.‎ ‎  · Don't dive into unknown waters. Always swim in line with the shore.‎ ‎  · Find out at the seaside when and where it is safe to swim.‎ ‎  · Don't use floating toys on the water. Wind can easily sweep them out to sea.‎ ‎  · Get out of the water if you feel tired or cold. Cold can kill even strong swimmers.‎ ‎  Help Telephone: 2125435902‎ ‎  Atlantic City Beach Office ‎  ‎ ‎  Arrive on time. ‎ ‎  Introduce yourself in a polite manner.‎ ‎  Read company materials while you wait.‎ ‎  Have a firm handshake.‎ ‎  Listen.‎ ‎  Use body language to show interest.‎ ‎  Smile and nod to the interviewer.‎ ‎  Ask about the next thing you should do.‎ ‎  Thank the interviewer.‎ ‎  Write a thankyou letter to anyone you have spoken to.‎ ‎  For more information, please visit jobweb.com ‎  368 Cooper Square,New York. NY 10008‎ ‎  6. If you want to work in an office, where can you get the forms to fill in?‎ ‎  A. Website. jobweb. com ‎  B. Atlantic City Beach Office.‎ ‎  C. 368 Cooper Square, New York, NY 10008.‎ ‎  D. 866 United nations Plaza, # 525 ,New York,NY 10017.‎ ‎  [解析] 细节理解题。根据第一部分"is looking for a parttime front desk office worker."可知答案。‎ ‎  [答案] D ‎  7. What does a person need most to be fit for the onehour weekday job?‎ ‎  A. He should be cheerful,dependable,and easygoing.‎ ‎  B. He has to work from Monday to Friday.‎ ‎  C. He can remain calm in a difficult situation.‎ ‎  D. He can welcome guests and deal with phone calls.‎ ‎  [解析] 推理判断题。由第一部分中间"More importantly,you can stay cool under pressure."可以推断出在有压力的情况下,需要保持冷静。‎ ‎  [答案] C ‎  8. According to the above information,what is the right thing to do when you swim?‎ ‎  A. To keep close to the beach.‎ ‎  B. To dive into unknown waters.‎ ‎  C. To use floating toys on the water.‎ ‎  D. To swim soon after lunch.‎ ‎  [解析] 细节理解题。由文中第二部分"Find out at the seaside when and where it is safe to swim."可知道答案。‎ ‎  [答案] A ‎  9. The best title for the last piece of information would be________.‎ ‎  A. Tips on Showing Interest in a Job ‎  B. Steps to a Successful Interview ‎  C. Advice on Introducing Yourself Politely ‎  D. Rules of Body Language in an Interview ‎  [解析] 主旨大意题。根据文中最后一部分的"Smile and nod to the interviewer."以及"Thank the interviewer。"可以判断出这一部分是关于如何面试成功的建议。‎ ‎  [答案] B ‎(第43篇)‎ ‎  When women sit together to watch a movie on TV, they usually talk simultaneously (同时地) about a variety of subjects, including children, men, careers and what' s happening in their lives. When groups of men and women watch a movie together, the men usually end up telling the women to shut up.Men can either talk or watch the screen - they can't do both -and they don' t understand that women can. Besides, women consider that the point of all getting together is to have a good time and develop relationships - not just to sit there like couch potatoes staring at the screen.‎ ‎  During the ad breaks, a man often asks a woman to explain the plot and tell him where the relationship between the characters is going. He is unable, unlike women, to read the subtle ‎ body language signals that reveal how the characters are feeling emotionally. Since women originally spent their days with the other women and children in the group, they developed the ability to communicate successfully in order to maintain relationships. For a woman, speech continues to have such a clear purpose: to build relationships and make friends. For men, to talk is to relate the facts.‎ ‎  Men see the telephone as a communication tool for sending facts and information to other people, but a woman sees it as a means of bonding. A woman can spend two weeks on vacation with her girlfriend and, when she returns home, telephone the same girlfriend and talk for another two hours.‎ ‎  There is no convincing evidence that social conditioning, the fact that girls' mothers talked to them more, is the reason why girls talk more than boys. Psychiatrist Dr Michael Lewis, author of Social Behaviour and Language Acquisition, conducted experiments that found mothers talked to, and looked at, baby girls more often than baby boys. Scientific evidence shows parents respond to the brain bias of their children. Since a girl' s brain is better organized to send and receive speech, we therefore talk to them more. Consequently, mothers who try to talk to their sons are usually disappointed to receive only short grunts in reply.‎ ‎  [语篇解读] 女人在一起总是有说不完的话,甚至能一边看电影一边聊,男人却做不到,这是为什么?因为她们认为感情联络很重要,而且她们的大脑在这方面有着特殊功能。‎ ‎  1.While watching TV with others, women usually talk a lot because they________.‎ ‎  A. are afraid of awkward silence with their families and friends ‎  B. can both talk and watch the screen at the same time ‎  C. think they can have a good time and develop relationships ‎  D. have to explain the plot and body language to their husbands ‎  [解析] 考查细节理解。根据文章第一段的最后一句可知妇女们聚在一起总有说不完的话是因为她们认为在一起就要玩得愉快,同时加深关系。‎ ‎  [答案] C ‎  2.After a vacation with her girlfriend, a woman would talk to her again on the phone for hours in order to________.‎ ‎  A. experience the happy time again ‎  B. keep a close tie with her ‎  C. recommend her a new scenic spot ‎  D. remind her of something forgotten ‎  [解析] 考查细节理解。根据文章的第三段中的a woman sees it as a means of bonding可知,女人度假回来会给朋友打电话长聊是因为她们把电话看做维系关系的纽带。‎ ‎  [答案] B ‎  3.What does the author want to tell us most?‎ ‎  A. Women's brains are better organized for language and communication.‎ ‎  B. Women love to talk because they are more sociable than men.‎ ‎  C. Men do not like talking because they rely more on facts.‎ ‎  D. Social conditioning is not the reason why women love talking.‎ ‎  [解析] 考查写作意图。根据文章最后一段中的"Since a girl's brain is better organized to send and receive speech"一句可知,作者最想让读者知道的就是她们能滔滔不绝的原因是她们的大脑在交流当中更善于处理信息。‎ ‎  [答案] C ‎  4.Which of the following would be the best title for the passage?‎ ‎  A. Women Are Socially Trained to Talk ‎  B. Talking Maintains Relationships ‎  C. Women Love to Talk ‎  D. Men Talk Differently from Women ‎  [解析] 考查标题归纳。本文是围绕女人喜欢讲话这个话题来展开的,因此C项为最佳标题。‎ ‎  [答案] C ‎(第44篇)‎ ‎  When I was a child, I often dreamed of the time when I could leave home and escape to the city. We lived on a farm and,in the winter especially, we were quite cut off from the outside world. As soon as I left school, I packed my bags and moved to the capital. However, I soon discovered that city life has its problems too.‎ ‎  One big disadvantage is money-it costs so much to go out,not to mention basics like food and housing. Another disadvantage is pollution. I suffer from asthma (哮喘), and at times the air is so bad that I am afraid to go outside. Then there is the problem of traveling round. Although I have a car, I seldom use it because of the traffic jams. One choice is to go by bicycle,but that can be quite dangerous.‎ ‎  Of course there are advantages. First, there is so much to do in the city,whatever your tastes in culture or entertainment (娱乐活动). Besides, there are wonderful jobs and greater chances of moving to a more important job or position. Finally,if you like shopping, the variety of goods is very surprising-and, what is more, shops are often only a short walk away.‎ ‎  Is life better then,in the city? Perhaps it is,when you are in your teens(十几岁)or twenties. However, as you get older,and especially if you have small children, the peace of the countryside may seem preferable. I certainly hope to move back there soon.‎ ‎  5. What was the writer always thinking about when he was a child?‎ ‎  A. Staying on the farm.‎ ‎  B. Moving to the countryside.‎ ‎  C. Leaving home for the city.‎ ‎  D. Running away from the school.‎ ‎  [解析] 细节理解题。由文章第一段第一句"When I was a child,I often dreamed of the time when I could leave home and escape to the city."可知作者小时候的梦想是离开家去大城市。‎ ‎  [答案] C ‎  6. Which of the following is TRUE about the writer?‎ ‎  A. He is very old now. B. He is in good health.‎ ‎  C. He prefers driving a car. D. He lives in the city now.‎ ‎  [解析] 判断正误题。由文章最后一段"However,as you get older,and especially if you have small children,the peace of the countryside may seem preferable.I certainly hope to move back there soon."可以推出D选项正确。‎ ‎  [答案] D ‎  7. In the passage, the writer tries to________.‎ ‎  A. express his opinions about way of life ‎  B. describe his life in the countryside ‎  C. show an interest in the outside world ‎  D. persuade the reader to live in the city ‎  [解析] 推理判断题。通读全文不难判断,作者主要表达自己对生活方式的看法。‎ ‎  [答案] A ‎  8. How is the passage mainly developed?‎ ‎  A. By inferring. B. By comparing.‎ ‎  C. By listing examples. D. By giving explanations.‎ ‎  [解析] 推理判断题。从第二、三段的首句可以判断,文章主要通过对比展开。‎ ‎  [答案] B ‎(第45篇)‎ ‎  I am a writer. I spend a great deal of my time thinking about the power of language - the way it can evoke (唤起)an emotion, a visual image, a complex idea, or a simple truth.Language_is_the_tool_of_my_trade. And I use them all - all the Englishes I grew up with.‎ ‎  Born into a Chinese family that had recently arrived in California, I' ve been giving more thought to the kind of English my mother speaks. Like others, I have described it to people as "broken" English. But I feel embarrassed to say that. It has always bothered me that I can think of no way to describe it other than "broken" , as if it were damaged and needed to be fixed, as if it lacked a certain wholeness. I've heard other terms used, "limited English" , for example. But they seem just as bad, as if everything is limited, including people' s perceptions (认识) of the limited English speaker.‎ ‎  I know this for a fact, because when I was growing up,my mother' s "limited" English limited my perception of her. I was ashamed of her English. I believed that her English reflected the quality of what she had to say. That is, because she expressed them imperfectly her thoughts were imperfect. And I had plenty of evidence to support me: the fact that people in ‎ department stores, at banks, and at restaurants did not take her seriously, did not give her good service, pretended not to understand her, or even acted as if they did not hear her.‎ ‎  I started writing fiction in 1985. And for reasons I won't get into today, I began to write stories using all the Englishes I grew up with: the English she used with me, which for lack of a better term might be described as "broken"; and what I imagine to be her translation of her Chinese, her internal (内在的) language, and for that I sought to preserve the essence,but neither an English nor a Chinese structure. I wanted to catch what language ability tests can never show: her intention, her feelings, the rhythms of her speech and the nature of her thoughts.‎ ‎  [语篇解读] 本文主要记叙了作者在不同时期对母亲所使用的语言的不同看法。‎ ‎  1.By saying "Language is the tool of my trade", the author means that________.‎ ‎  A. she uses English in foreign trade ‎  B. she is fascinated by languages ‎  C. she works as a translator ‎  D. she is a writer by profession ‎  [解析] 推理判断题。此句中the tool of my trade中的trade指的是职业。由本文第一句可知"我"是一名作家;此句又说语言是"我"的职业工具,故此句指的自然是"我"的职业为作家。‎ ‎  [答案] D ‎  2.The author used to think of her mother' s English as ________.‎ ‎  A. impolite        B. amusing ‎  C. imperfect D. practical ‎  [解析] 推理判断题。由文中倒数第二段中"...because she expressed them imperfectly,her thoughts were imperfect"一句可知选C,作者一度以为自己妈妈所说的英语是不完美的。‎ ‎  [答案] C ‎  3.Which of the following is TRUE according to Paragraph 3?‎ ‎  A. Americans do not understand broken English.‎ ‎  B. The author' s mother was not respected sometimes.‎ ‎  C. The author' s mother had positive influence on her.‎ ‎  D. Broken English always reflects imperfect thoughts.‎ ‎  [解析] 细节理解题。由文章倒数第二段最后一句话可知选B。此题易误选D。其实,此句只是"我"小时候对妈妈的一种误解,由最后一段可知这种误解已经消除了,因此D项的说法是不对的。‎ ‎  [答案] B ‎  4.The author gradually realizes her mother's English is________.‎ ‎  A. well structured B. in the old style ‎  C. easy to translate D. rich in meaning ‎  [解析] 推理判断题。由最后一段可知,我写小说时开始运用她与我说话时所用的英语。我尽力地保留她语言的精髓,但未运用她所使用的英语或汉语的结构。我想领悟语言能力测试中所不能反映但我妈妈语言中所包含的意思。故选D。‎ ‎  [答案] D ‎  5.What is the passage mainly about?‎ ‎  A. The change of the author' s attitude to her mother' s English.‎ ‎  B. The limitation of the author' s perception of her mother.‎ ‎  C. The author's misunderstanding of "limited" English.‎ ‎  D. The author' s experiences of using broken English.‎ ‎  [解析] 主旨大意题。本文以时间为线索,讲述了作者对妈妈所使用的语言的不同态度,故选A。‎ ‎  [答案] A ‎  ‎ ‎   (第46篇)‎ ‎  Tom was one of the brightest boys in the year, with supportive parents. But when he was 15 he suddenly stopped trying. He left school at 16 with only two scores for secondary school subjects. One of the reasons that made it cool for him not to care was the power of his peer(同龄人) group.‎ ‎  The lack of right male (男性的) role models in many of their lives - at home and particularly in the school environment (环境) - means that their peers are the only people they have to judge themselves against.‎ ‎  They don't see men succeeding in society so it doesn't occur to them that they could make something of themselves.Without male teachers as a role model, the effect of peer actions and street culture(文化) is allpowerful. Boys want to be part of a club. However, schools can provide the environment for change, and provide the right role models for them. Teachers need to be trained to stop that but not in front of a child's peers. You have to do it one to one,because that is when you see the real child.‎ ‎  It's pointless sending a child home if he or she has done wrong. They see it as a welcome day off to watch television or play computer games. Instead, schools should have a special unit where a child who has done wrong goes for the day and gets advice about his problemssomewhere he can work away from his peers and go home after the other children.‎ ‎  6. Why did Tom give up studying?‎ ‎  A. He disliked his teachers.‎ ‎  B. His parents no longer supported him.‎ ‎  C. It's cool for boys of his age not to care about studies.‎ ‎  D. There were too many subjects in his secondary school.‎ ‎  [解析] 细节理解题。由第一段最后一句话"One of the reasons that made it cool for him not to care was the power of his peer group."可知受同龄人的影响,年轻人把不在乎学习看成是很酷的一件事情。‎ ‎  [答案] C ‎  7. What seems to have a bad effect on students like Tom?‎ ‎  A. Peer groups.‎ ‎  B. A special unit.‎ ‎  C. The student judges.‎ ‎  D. The home environment.‎ ‎  [解析] 推理判断题。由第一段最后一句话和第二段内容"their peers are the only people they have to judge themselves against..."可推论出对于学生来说对他们产生很大影响的就是他们的同龄人。‎ ‎  [答案] A ‎  8. What should schools do to help the problem schoolboys?‎ ‎  A. Wait for their change patiently.‎ ‎  B. Train leaders of their peer groups.‎ ‎  C. Stop the development of street culture.‎ ‎  D. Give them lessons in a separate area.‎ ‎  [解析] 推理判断题。由最后一段"Instead,schools should have a special unit where a child who has done wrong goes for the day and gets advice about his problems-somewhere he can work away from his peers and go home after the other children."可推论出学校应该为这样的学生设立独立的空间单元,来有针对性的采取措施解决他们的问题。‎ ‎  [答案] D ‎  9. A teacher's work is most effective with a schoolboy when he________.‎ ‎  A. is with the boy alone ‎  B. teaches the boy a lesson ‎  C. sends the boy home as punishment ‎  D. works together with another teacher ‎  [解析] 细节理解题。由第三段最后一句话"You have to do it one to one,because that is when you see the real child."可知在单独面对和帮助这样的问题学生时,老师会看到他们最真实的一面,这样的方式最有效果。‎ ‎  [答案] A ‎  ‎ ‎(第47篇)‎ ‎  Only three local students won Chinese Blog(博客) Competition. And 15 of the 18 awards went to students from China.‎ ‎  170 students' task: to get a fullydesigned blog up and running, complete with many postings based on a theme of choice - all written in Chinese.‎ ‎  Themes ranged from local opinions - such as the usage of Singlish, education and whether Singapore can be a cultural centre - to food blogs.‎ ‎  The entries were judged on language proficiency(熟练程度) and the quality of writing, as well as the design and level of exchanging ideas with readers.‎ ‎  Academics from the National University of Singapore and the SIM University, IT experts, and a journalist from Chinese newspaper Lianhe Zaobao in Singapore made up the judges.‎ ‎  In the end, only three Singaporean students made it to the award list - the rest of the awards were swept up by students from China.‎ ‎  "No surprise, "said Mr. Chow Yaw Long, 37, teacherincharge from Innova Junior College, which organized the event. "Although the topics were local subjects, the foreign students were generally better in terms of the content of the posts and their grasp of the Chinese language."‎ ‎  One of the three local students winning the first prize in the Best Language Award was blogger Christina Gao, 19, from the Saint Andrew's Junior College, who spared no effort in researching for and writing her blog. Each entry took her between five and seven days to produce, complete with pictures and even podcasts (播客).‎ ‎  Her advice for bloggers is: Be responsible.‎ ‎  "Some bloggers out there only seek to blame the authorities and other bloggers," said Miss Gao. "I think they lack responsibilities and there is no value to their posts."‎ ‎  [语篇解读] 新加坡举行一次国内外学生用汉语写博客的大赛,结果只有3个本土学生得奖,其余的被中国学生包揽。‎ ‎  1.The competition was organized by ________.‎ ‎  A. the National University of Singapore ‎  B. Chinese newspaper Lianhe Zaobao ‎  C. Innova Junior College ‎  D. the Saint Andrew' s Junior College ‎  [解析] 细节理解题。依据文章第七段中的"teacherincharge from Innova Junior College,which organized the event"可知,这里的which指代Innova Junior College,也就是这次活动的组织者。‎ ‎  [答案] C ‎  2.Which of the following is TRUE according to the passage?‎ ‎  A. Chinese students won most of the awards.‎ ‎  B. Not all the themes were about local subjects ‎  C. The blogs could be written in Chinese or Singlish.‎ ‎  D. The judges were from universities in Singapore and China.‎ ‎  [解析] 正误判断题。依据文章第一段可知,A项正确,这次大赛中国学生获得了绝大多数奖。‎ ‎  [答案] A ‎  3.What Miss Gao said suggests that ________.‎ ‎  A. she likes to blame the authorities ‎  B. she has a sense of responsibility ‎  C. she thinks highly of the others' blogs ‎  D. she loves to read valuable posts ‎  [解析] 推理判断题。依据文章最后三段可知她很有责任感。‎ ‎  [答案] B ‎  4.The passage is mainly about________.‎ ‎  A. how Chinese students won the awards in the competition ‎  B. why bloggers should take responsibility for their blogs ‎  C. how Miss Gao won the first prize in the competition ‎  D. what the result of the competition was ‎  [解析] 主旨大意题。本文主要介绍新加坡举行一次国内外学生用汉语写博客的大赛,叙述了比赛的结果,所以可以得出答案为D。‎ ‎  [答案] D ‎(第48篇)‎ ‎  Five million people visit Grand Canyon in the US every year. For the purpose of helping project Grand Canyon for your fellow visitors and future generations, please follow the guidelines below.‎ ‎  Camping ‎  To protect the park, camping is allowed only within permitted campgrounds. Permits are required for overnight camping at the North Rim. Advance booking can be received by mail. Please write: Information Center, P. O. Box 129, Grand Canyon, AZ 86023‎ ‎  Fires ‎  Because of the extreme fire danger, campfires are not allowed except at Mather and Desert View campgrounds. Collection of firewood is not allowed either.‎ ‎  Hiking(远足)‎ ‎  Please stay on permitted paths.Otherwise you may destroy desert plants. Pack_out_what_you_pack_in,_so you leave no signs of your visit. It is important to keep in mind that you are in a national park where wildlife exists.‎ ‎  Weather ‎  The weather at Grand Canyon can change very quickly.With so much rock,lightning(闪电) causes a particular danger during sudden summer storms. These storms also frequently bring floods inside valleys,a danger to hikers. Watch the skies and check daily weather reports.‎ ‎  Wildlife ‎  Do not feed park wildlife. There have been a few cases at Grand Canyon National Park where deer(鹿) were purposely shot because they are plastic bags that left them sick and weak. Hungry deer can be a danger and have kicked and bitten visitors at Grand Canyon. Some other animals will also beg and bite. For your own safety and the wellbeing of the animals,please do not feed wildlife, no matter how gentle they may appear.‎ ‎  5. What can you do first if you want to go camping?‎ ‎  A. Know the permitted paths.‎ ‎  B. Book campgrounds in advance.‎ ‎  C. Make sure not to make a fire.‎ ‎  D. Stop at Mather and Desert View.‎ ‎  [解析] 细节理解题。根据文章第二段"Advance booking can be received by mail"能看出要想出去夏令营首先做的就是book campgrounds in advance.‎ ‎  [答案] B ‎  6. What do the underlined words "pack out what you pack in" in Paragraph 4 probably mean?‎ ‎  A. keep everything out of campgrounds ‎  B. take away everything you bring in ‎  C. carry all the necessary food ‎  D. look after your personal belongings ‎  [解析] 推理判断题。根据"you leave no signs of your visit."来推断带走你所带来的东西,不要留下任何的东西。‎ ‎  [答案] B ‎  7. Why were some deer killed on purpose at Grand Canyon?‎ ‎  A. They ate wrong things and became very ill.‎ ‎  B. They were a danger to other gentle animals.‎ ‎  C. They begged food from visitors.‎ ‎  D. They kicked and bit visitors.‎ ‎  [解析] 细节理解题。根据"purposely shot because they are plastic bags that left them sick and weak"可知它们吃错了东西而变得有病。‎ ‎  [答案] A ‎  8. What is the main purpose of the text?‎ ‎  A. To provide travel information.‎ ‎  B. To report some recent news.‎ ‎  C. To teach tourists hiking skills.‎ ‎  D. To introduce the wild life.‎ ‎  [解析] 主旨大意题。根据第一段"For the purpose of helping project Grand Canyon for your fellow visitors and future generations,please follow the guidelines below."可知道整篇文章都是为旅游提供信息。‎ ‎  [答案] A ‎(第49篇)‎ ‎  Three years ago,five parrots were set free in a wild place of Arizona,thousands of miles from the Channel Islands in Jersey where they had been looked after by zookeepers. No evolutionary strategies informed them how to behave in this new landscape of mountainous pine forest unoccupied by their kind for 50 years. To the researchers' surprise, they failed to ‎ make contact with a group of wild parrots imported from Mexico and set free at the same time. Within 24 hours the reintroducing ended in failure, and the poor birds were back in cages, on their way to the safety of the Arizona reintroduction programme.‎ ‎  Ever since then, the programme has enjoyed great success,mainly because the birds now being set free are Mexican birds illegally caught in the wild, confiscated (没收)on arrival north of the border, and raised by their parents in the safety of the programme.The experience shows how little we know about the behaviour and psychology(心理) of parrots, as Peter Bennett, a bird researcher,points out: "Reintroducing species of high intelligence like parrots is a lot more difficult. People like parrots, always treating them as nothing more than pets or valuable 'collectables'."‎ ‎  Now that many species of parrot are in immediate danger of dying out, biologists are working together to study the natural history and the behaviour of this family of birds. Last year was an important turning point:conservationists founded the World Parrot Trust, based at Hayle in Cornwall, to support research into both wild and caged birds.‎ ‎  Research on parrots is vital for two reasons. First, as the Arizona programme showed, when reintroducing parrots to the wild, we need to be aware of what the birds must know if they are to survive in their natural home. We also need to learn more about the needs of parrots kept as pets, particularly as the Trust's campaign does not attempt to discourage the practice, but rather urges people who buy parrots as pets to choose birds raised by humans.‎ ‎  [语篇解读] 本文通过在亚利桑那州五只鹦鹉被释放的故事,说明了鹦鹉引入计划的重要性以及对鹦鹉生活习性等各方面进行研究的重要性和对鹦鹉进行研究的重要原因。‎ ‎  1.What do we know about the area where the five parrots were reintroduced?‎ ‎  A. Its landscape is new to parrots of their kind.‎ ‎  B. It used to be home to parrots of their kind.‎ ‎  C. It is close to where they had been kept.‎ ‎  D. Pine trees were planted to attract birds.‎ ‎  [解析] 事实细节题。根据第一段第二句:这五只鹦鹉未被教过如何在这个新地方生活;这片松林山区50年来未被这种鹦鹉占据过(50年前有过)。可知,五只鹦鹉被引入的地方曾经是鹦鹉的家园或繁衍之地。‎ ‎  [答案] B ‎  2.The reintroducing experience three years ago shows that manraised parrots________.‎ ‎  A. can find their way back home in Jersey ‎  B. are unable to recognize their parents ‎  C. are unable to adapt to the wild ‎  D. can produce a new species ‎  [解析] 推理判断题。根据第一、二段,特别是信息句Ever since then,the programme has enjoyed great success,mainly because the birds now being set free are Mexican birds illegally caught in the wild.可以推测出,人工喂养的鹦鹉不能适应野外的生存环境。‎ ‎  [答案] C ‎  3.Why are researches on parrots important according to the passage?‎ ‎  A. The Trust shows great concern for the programme.‎ ‎  B. We need to know more about how to preserve parrots.‎ ‎  C. Many people are interested in collecting parrots.‎ ‎  D. Parrots' intelligence may some day benefit people.‎ ‎  [解析] 推理判断题。最后一段就是信息提示。主要表达了对鹦鹉进行研究的重要性,主要表现在两个方面:我们必须了解鹦鹉在自然条件下如何生存;其次,我们需要更多地了解宠物鹦鹉的需要,督促那些买鹦鹉当宠物的人选择人工培育的鹦鹉。从而判断,本题主要表达我们如何保护鹦鹉的问题。‎ ‎  [答案] B ‎  4.According to the passage, people are advised________.‎ ‎  A. to treat wild and caged parrots equally ‎  B. to set up comfortable homes for parrots ‎  C. not to keep wild parrots as pets ‎  D. not to let more parrots go to the wild ‎  [解析] 推理判断题。由最后一段特别是最后一句We also need to learn more about the needs of parrots kept as pets,particularly as the Trust's campaign does not attempt to discourage the practice,but rather urges people who buy parrots as pets to choose birds raised by humans.可以推出,作者建议我们不要把野鹦鹉当作宠物。‎ ‎  [答案] C ‎  [长难句解读] We also need to learn more about the needs of parrots kept as pets,particularly as the Trust's campaign does not attempt to discourage the practice,but rather urges people who buy parrots as pets to choose birds raised by humans.我们也需要更多了解宠物鹦鹉的需要,尤其因为信托基金运动并不旨在劝阻这一做法,而是督促买鹦鹉作宠物的人选择人工培育的鹦鹉。‎ ‎  句中的not...but...表示"不是......而是......"。连接两个并列的句子attempt to discourage the practice和rather urges people who buy parrots...。其中urge sb.to do sth.表示"督促某人做某事"。‎ ‎(第50篇)‎ ‎  (2008年江西)‎ ‎  Intellectual property (IP) is a product of the mind that has commercial value. The concept dates back to 1623, when the first patent law to protect IP rights was passed. IP rights protect an artist from having his/her creative ideas copied by another. For example, if somebody generates an idea for a novel, that idea is protected by IP rights. If someone else wishes to represent the idea or develop it further,he/she must consult the original artists, who will normally be rewarded financially for its use. Back in the 17th century,IP rights were primarily ‎ carried out to protect newly developed manufacturing processes against stealing,but today, intellectual property rights are also enjoyed by those who create music, art and literature.‎ ‎  In recent years, IP rights have been the focus of a great deal of discussion because of a technology which looks set to weaken them altogether:the Internet. Many years ago, if you wanted a recording of a song, you would have to purchase it from a music store;if a novel,from a book store. In those days, IP rights were easily protected since it was very difficult to obtain intellectual property without paying for it. However, a lot of IP, including songs, films, books and artwork, can be downloaded today free of charge using the Internet. This practice has now taken the world by storm, dramatically affecting the way in which we view IP rights.‎ ‎  5. According to the writer, in the beginning, IP rights were mainly of use to________.‎ ‎  A. those creating music,art and literature ‎  B. novelists ‎  C. engineers and inventors ‎  D. those not receiving financial reward for their work ‎  [解析] 细节判断题。从第一段最后两句可以判断出知识产权主要对工程师和发明家的创作成果有保护作用。‎ ‎  [答案] C ‎  6. What do we know about the Internet according to the passage?‎ ‎  A. It makes IP rights harder to protect.‎ ‎  B. It sells songs and films.‎ ‎  C. It does not affect the way we understand IP rights.‎ ‎  D. It prevents the production of artwork.‎ ‎  [解析] 细节判断题。从第二段的"...because of a technology which looks set to weaken them altogether..."可以判断出答案。‎ ‎  [答案] A ‎  7. According to Paragraph 2, what has "taken the world by storm" ?‎ ‎  A. Intellectual property rights.‎ ‎  B. The Internet.‎ ‎  C. Free downloading.‎ ‎  D. The large number of songs, films and books.‎ ‎  [解析] 细节判断题。第二段最后一句中的"This practice"代指上句中提到的"can be downloaded today free of charge using the Internet.",故选C项。‎ ‎  [答案] C ‎  8. A possible title for this passage could be________.‎ ‎  A. A History of IP Rights ‎  B. Ways to Protect Your IP Rights ‎  C. The Present and the Future of IP ‎  D. IP Rights and Our Attitudes ‎  [解析] 主旨大意题。本文从头到尾都是围绕知识产权和我们的态度这两方面写的。‎ ‎  [答案] D ‎(第51篇)‎ ‎  Cities alarmed by deaths and injuries of pedestrians are taking efforts to make crosswalks safer for people on foot, especially seniors and children who need more time to cross streets.‎ ‎  A pedestrian is killed in a traffic accident in the USA every 110 minutes; one is injured every nine minutes, according to official data.Crosswalks can be especially dangerous for the elderly. Among people 70 and older,36% of pedestrian deaths in 2006 occurred in crosswalks, compared with 21% of those younger than 70, according to the Insurance Institute for Highway Safety.‎ ‎  The Federal Highway Administration(FHWA) advises that next year states increase by nearly 15% the amount of time traffic lights provide for pedestrians to cross the street after the flashing orange hand appears.‎ ‎  FHWA spokesman Doug Hecox says reasons for the change include an aging population that needs more time to cross, healthconscious Americans walking more ,children encouraged to walk to prevent getting overweight and high gas prices pushing people to walk instead of drive.‎ ‎  Pedestrian deaths went down by 12% from 5,449 in 1996 to 4,784 in 2006. But among those in 2006,471 were killed in crosswalks, down slightly from 488 ten years earlier, the National Highway Traffic Safety Administration(NHTSA) says.‎ ‎  [语篇解读] 横过马路时,行人的意外死亡率居高不下。很多城市正在努力采取措施保证行人过马路的安全,尤其是老人和孩子。‎ ‎  1.Which of the following is true according to the text?‎ ‎  A. Among 100 pedestrian deaths there were 21 people younger than 70.‎ ‎  B. Old people are more likely to meet with accidents in crosswalks.‎ ‎  C. Traffic accidents killed more old people than young people.‎ ‎  D. About seven traffic accidents happened per hour.‎ ‎  [解析] 事实细节题。第二段第二三句告诉我们:老年人在十字路口遭遇交通事故的可能性最大。‎ ‎  [答案] B ‎  2.What is FHWA's suggestion to states?‎ ‎  A. Fixing more traffic lights.‎ ‎  B. Providing more crosswalks.‎ ‎  C. Giving pedestrians more time to cross streets.‎ ‎  D. Increasing the time before the orange lights appear.‎ ‎  [解析] 事实细节题。第三段告诉我们FHWA建议给行人更多的时间通过路口。‎ ‎  [答案] C ‎  3.What's the cause of the crosswalk safety problem according to the text?‎ ‎  A. There're many cars and buses on the road.‎ ‎  B. Pedestrians are careless.‎ ‎  C. Crosswalks are crowded.‎ ‎  D. Drivers don't give way.‎ ‎  [解析] 推理判断题。第四段指出:老年人需要更多的时间通过马路,有健康意识的人也好步行,儿童被鼓励步行以防止发胖,油价较高也让人们更倾向于步行而不是驾车。由此可知十字路口的安全问题是过度拥挤。‎ ‎  [答案] C ‎  4.The report from NHTSA suggests that ________.‎ ‎  A. fewer people were injured in crosswalks ‎  B. crosswalk safety has been greatly improved ‎  C. much has been done to reduce traffic accidents ‎  D. pedestrian deaths in crosswalks remain a serious problem ‎  [解析] 推理判断题。最后一段NHTSA称:2006年行人死亡人数4,784,比1996年(5,449)降低了12%,但这一数字依旧很高,注意句中用的是down slightly稍微有点下降,所以十字路口的安全问题依然严峻。‎ ‎  [答案] D ‎(第52篇)‎ ‎  Topping the class academically was certainly an advantage. Studying was a breeze for Nigel. The reward was certainly incomparable to the little effort that he had to put in. It began when he was selected to help the teachers in the computer laboratories.‎ ‎  The peak of his school career came not when he topped the school but when he was selected for the nationwide competition. Unlike everyone else, Nigel wanted to join the contest because he liked playing with the Lego sets and making something out of them. Nigel spent the next two months rebuilding his robot. It was during this time that Nigel found out about the prizes for the competition as well as another competitor,Alicia,from a neighboring school. His early intentions were forgotten. Getting the thousanddollar prize was more important than anything else. Nigel decided to befriend Alicia.Unaware of his intentions,she told him all about the robot that she had been building for the competition. He even helped her to put the finishing touches to her robot. He was glad with the way things had progressed. His robot looked even better than Alicia's and it was able to bounce a ball with its arm, something Alicia had failed to do.‎ ‎  On the day of the competition, he saw Alicia. Everything dawned on her the minute she saw him among the competitors.She stared at him, puzzled at first, then angry and finally a look of helplessness came over her.‎ ‎  The flashbulbs of the camera exploded in Nigel's face. The robot had performed actions so unique and different that the specialists'judgments were the same. Nigel was so pleased with himself that he did not even notice the girl standing a few feet away from him. Without her, he would never have won the competition.‎ ‎  5. What reward did Nigel receive for doing well in his school work?‎ ‎  A. He was offered a parttime job.‎ ‎  B. He was honored with a scholarship.‎ ‎  C. He helped his teachers construct a robot.‎ ‎  D. He helped in the computer laboratories.‎ ‎  [解析] 细节理解题。由文中第一段"It began when he was selected to help the teachers in the computer laboratories."可以得知答案。‎ ‎  [答案] D ‎  6. Nigel's original intention of joining the contest was to________.‎ ‎  A. be the top student of the school ‎  B. bring great honor to his school ‎  C. construct a robot with the Lego sets ‎  D. win the thousanddollar prize ‎  [解析] 细节理解题。根据第二段"Unlike everyone else,Nigel wanted to join the contest because he liked playing with the Lego sets and making something out of them."可以看出答案。‎ ‎  [答案] C ‎  7. Why did Nigel help Alicia finish her robot?‎ ‎  A. He tried to make friends with her.‎ ‎  B. He was fond of building robots.‎ ‎  C. He intended to help her.‎ ‎  D. He didn't want her to suspect him.‎ ‎  [解析] 推理判断题。由文中第二段"...as well as another competitor,Alicia,from a neighboring school.His early intentions were forgotten.Getting the thousanddollar prize was more important than anything else."能够推断出答案。‎ ‎  [答案] D ‎  8. What is the author's attitude towards Nigel's actions?‎ ‎  A. He is mildly critical.  B. He is strongly critical.‎ ‎  C. He is in favor of them. D. His attitude is not clear.‎ ‎  [解析] 观点态度题。根据文中第一段第二、三句"Studying was a breeze for Nigel.The reward was certainly incomparable to the little effort that he had to put in."可以判断学习对于他是不费吹灰之力的事,他付出很少的努力就能的到奖赏当然就是举世无双的了,言语中带着讽刺。‎ ‎  [答案] A ‎(第53篇)‎ ‎  Dear Friend,‎ ‎  The recent success of children's books has made the general public aware that there's a huge market out there.‎ ‎  And there's a growing need for new writers trained to create the $3 billion worth of children's books bought each year... plus stories and articles needed by over 650 publishers of magazines for children and teenagers.‎ ‎  Who are these needed writers? They're ordinary folks like you and me.‎ ‎  But am I good enough?‎ ‎  I was once where you might be now. My thoughts of writing had been pushed down by selfdoubt, and I didn't know where to turn for help.‎ ‎  Then, I accepted a free offer from the Institute to test my writing aptitude(潜能), and it turned out to be the inspiration I needed.‎ ‎  The promise that paid off ‎  The Institute made the same promise to me that they will make to you, if you show basic writing ability:‎ ‎  You will complete at least one manuscript(手稿) suitable to hand in to a publisher by the time you finish our course.‎ ‎  I really didn't expect any publication before I finished the course,but that happened.I sold three stories. And I soon discovered that was not unusual at the Institute.‎ ‎  Since graduation, I have authored 34 nationally published children's books and over 300 stories and articles.‎ ‎  Free test and brochure ‎  We offer a free aptitude test and will send you a copy of our brochure describing our recognized homestudy courses on the basis of oneonone training.‎ ‎  Realize your writing dream today. There's nothing sadder than a dream delayed until it fades forever.‎ ‎  Sincerely,‎ ‎  Kristi Holl, Instructor ‎  Institute of Children's Literature ‎  [语篇解读] 儿童读物有着巨大的市场,市场需要更多的儿童读物作家。参加我们的培训课程,相信你就能成为儿童作家中的一员。‎ ‎  1.From the first three paragraphs, we learn that ________.‎ ‎  A. children's books are usually bestsellers ‎  B. publishers are making $3 billion each year ‎  C. magazines for teenagers have drawn public attention ‎  D. there is a growing need for writers of children's books ‎  [解析] 推理判断题。这篇文章在为培训课程做广告,因此前三段先讲到儿童作家的市场需求以吸引读者参加该课程。‎ ‎  [答案] D ‎  2.When finishing the course, you are promised to ________.‎ ‎  A. be a successful publisher ‎  B. become a confident editor ‎  C. finish one work for publication ‎  D. get one story or article published ‎  [解析] 事实细节题。由文章"The promise that paid off"部分:You will complete at least one manuscript(手稿)suitable to hand in to a publisher by the time you finish our course.可知。‎ ‎  [答案] C ‎  3.Kristi Holl mentions her experience mainly to ________.‎ ‎  A. prove she is a good instructor ‎  B. promote the writing program ‎  C. give her advice on course preparation ‎  D. show she sold more stories than articles ‎  [解析] 推理判断题。作者作为课程的主办方通过讲述自己的经历来推广该课程。‎ ‎  [答案] B ‎(第54篇)‎ ‎  Despite the fact that it has never been seen, almost everyone is familiar with the legendary unicorn (独角兽).Descriptions of unicorns have been found dating from ancient times. The great philosopher Aristotle theorized that there were two types of unicorn-the socalled Indian Ass and the Oryx,a kind of antelope. Unicorns are often used in the logo of a noble family, town council or university as their special sign. Even Scotland is represented by a unicorn.‎ ‎  According to the legend, anyone attempting to catch a unicorn had to be extremely cautious as it has a reputation for being very fierce. A clever trick suggested by unicorntrappers,in order to catch this magnificent beast without being hurt by its horn,was for the hunter to stand in front of a tree and then to move quickly behind it as the unicorn charged. Hopefully,the creature could then be captured when its horn was stuck in the tree.‎ ‎  When hollowed out and used as a drinkingcup, the unicorn's horn was said to have the power to offer protection against poison. It was believed that nobody could be harmed by drinking the contents of a unicorn's horn. Right up until the French Revolution in 1789, the French court was said to have used cups made of "unicorn" horn in order to protect the king.In addition, the horn was said to have medicinal value, so much so that it could be sold for more than ten times the price of the same weight of gold. What, then, was "unicorn" horn? We know ‎ at times the rhino (犀牛) was confused with this legendary creature. A drinkingcup supposedly made of "unicorn" horn was discovered to be made of the horn of a rhino.‎ ‎  4. Which of the following is TRUE of the unicorn?‎ ‎  A. It was not historically recorded.‎ ‎  B. Its horn was first used in France.‎ ‎  C. It was similar to the Indian Ass and the Oryx.‎ ‎  D. It could be the symbol of a university.‎ ‎  [解析] 判断正误题。由第一段"Unicorns are often used in the logo of a noble family,town council or university as their special sign."可知,独角兽是贵族家庭、城镇理事会或大学的一个特殊标志。‎ ‎  [答案] D ‎  5. To catch a unicorn, the unicorntrappers had to try all of the following EXCEPT________.‎ ‎  A. tempting the unicorn to attack ‎  B. making use of the tree as a protection ‎  C. hiding quickly behind the unicorn ‎  D. having the unicorn horn stuck in the tree ‎  [解析] 细节理解题。由文中第二段可知,选项C在该段中没被提到。‎ ‎  [答案] C ‎  6. The last paragraph is mainly about________.‎ ‎  A. the properties of the unicorn horn ‎  B. the users of the unicorn horn ‎  C. the price of the unicorn horn ‎  D. the comparison between the unicorn horn and the rhino horn ‎  [解析] 主旨大意题。由文中最后一段的首句"...the unicorn's horn was said to have the power to offer protection against poison."可知,该段都是围绕着独角兽的角展开叙述的,说明独角兽的角是重要的财产。‎ ‎  [答案] A ‎  7. In the last paragraph, the word "unicorn" is in quotation marks(引号) because________.‎ ‎  A. the cup is designed only for a royal family ‎  B. the unicorn does not exist in reality ‎  C. the unicorn is the rarest animal in the world ‎  D. the medicinal value of the horn is appreciated ‎  [解析] 推理判断题。最后一段的最后两句可知,犀牛角的产品是真实存在的,而独角兽并不真正存在。‎ ‎(第55篇)‎ ‎  A ‎  Western New Bridge Library Announcement Shortened Library Hours for Spring Break Library Hours have been shortened to 7 hours a day (9∶00 a. m. 4∶00 p. m. ) for Spring Break from March 24 to March 30.‎ Coming Events ‎◆On Monday, March 24, at 10∶30 a. m. , Scott Sutton, a children's writer, will tell stories to kids over seven. Sutton' s attractive style will surely inspire everyone present !‎ ‎◆At 2∶00 p. m. , on March 25, a lecture will be delivered in Room 201, which focuses on the development of writing skill. Famous tutors from the district won' t disappoint you.‎ ‎◆At 1∶00 p. m. ,on March 26, the Georgetown Musicians will present an Irish Folk Concert, which will be entertaining for the entire family. Come for the music and stay to check out some relevant books for the rest of the week!‎ ‎◆On Thursday, March 27, at 2∶00 p. m. , the annual Children' s Gathering will take place in Room 201, the second floor. Pick up an invitation in the Children' s Room and return your RSVP (回复)to reserve your seat at the table by 3∶00 p. m. on Tuesday, March 25. Only children are allowed in the Gathering.‎ ‎◆At 10∶30 a.m. on Friday, March 28, Enzo Monfre of the hit kids' science show, ENZOology, will bring Fossils Live! Surely Enzo will take the audience back in time, deep beneath the surface of the earth, to uncover the mysteries of killer dinosaurs, and more. Enzo recently appeared on the Ellen DeGeneres Show - come and see him at the library!‎ Please note: In case of emergency, please call the Help Desk at 926 3736 and follow the procedures outlined on the voice message. The calldown service is staffed 24 hours a day, 7 days a week for emergencies. The Help Desk supplies service to you all the year round! For questions about all these, please contact_hld_@_wnbl._corg.‎ Come for the great Fun; Stay for the relevant Books!‎ ‎  [语篇解读] 本文是一则通知,属于应用文。文章主要介绍了寒假期间学校图书馆作息时间的变化及将要举行的一系列活动。‎ ‎  1.Tim wants to polish his writing, it is best for him to attend the related activity on________.‎ ‎  A. Monday        B. Tuesday ‎  C. Thursday D. Friday ‎  [解析] 推理判断题。根据Coming Events中的第一、二两条信息判断,提高写作技能的活动是在March 25举行的,而由第一条信息可知March 24是星期一,所以选B。‎ ‎  [答案] B ‎  2.Which of the following descriptions about the annual Children' s Gathering is true?‎ ‎  A. Tickets are a must in order to attend it.‎ ‎  B. Only those who have received an invitation are admitted in the Gathering.‎ ‎  C. You have to make a reply to the invitation to make sure you get a seat.‎ ‎  D. You' d better apply in advance because there are not enough seats for everyone.‎ ‎  [解析] 细节判断题。根据第四条信息可知,要想参加该活动,需要在儿童室取请帖并回复以预订座位,因此C项正确。‎ ‎  [答案] C ‎  3.Which of the following is NOT true according to the announcement?‎ ‎  A. The children who are interested in the mysteries of killer dinosaurs can attend the Ellen DeGeneres Show on Friday.‎ ‎  B. Children can' t attend all the activities with their parents.‎ ‎  C. Children can choose more than one of the activities.‎ ‎  D. Children can borrow some relevant books for the activities.‎ ‎  [解析] 细节判断题。根据第五条信息的最后一句可知A选项中的the Ellen DeGeneres Show并非在假期所举办的活动。‎ ‎  [答案] A ‎  4.If there is an emergency, you should________.‎ ‎  A. call the Help Desk ‎  B. contact hld @ wnbl. corg ‎  C. stay in the library and wait ‎  D. ring 926 3637‎ ‎  [解析] 细节理解题。由注意事项中的第一句话可知答案。‎ ‎  [答案] A ‎  5.The announcement is mainly intended for________.‎ ‎  A. staff members of the library ‎  B. parents of the school children ‎  C. the school children ‎  D. volunteers of the activities ‎  [解析] 推理判断题。根据通知内容可知,活动是为学生举办的,这则通知主要是让他们了解活动的时间、地点及相关内容。‎ ‎  [答案] C ‎  ‎ ‎   (第56篇)‎ ‎  ‎ ‎  ●Ms Tan, you' ve referred to your new novel as your eighth book.‎ ‎  That's because it took me six or seven attempts at a second novel before I started and completed this one.‎ ‎  ●Why do you think you had so many false starts?‎ ‎  I would say that my reasons were wrong: I was trying to prove that I wasn't just a mother-daughter storyteller, or I was trying to prove that I didn' t just have to write about ‎ things that were strictly Chinese or Chinese-American. Those were never the right reasons for writing those early stories. And I could never come up with other, better reasons for continuing them.‎ ‎  ●What kept you going on this book?‎ ‎  This book was different because it was based on my mother' s real life. The reason for writing it became more personal and emotional. After The Joy Luck Club came out, my mother was always explaining to people that she wasn' t any of the mothers in that book. And at one point she said to me, "Next book tells my true story." And then she started telling me things I never knew before. She also told me many, many stories,because my mother doesn' t generalize. The book really grew out of that.‎ ‎  ●Have you ever visited China?‎ ‎  Yes. I' ve been there twice: about three years ago and then again last November, both times with my mother and my husband.‎ ‎  ●Was it difficult to capture the Chinese-American dialect without sounding like a parody(拙劣的模仿)?‎ ‎  No, because it' s the language I' ve heard all my life from my mother. She speaks English as it' s direct translation from Chinese. But it's more than that: Her language also has more imagery than English.‎ ‎  ●Can you think of an example?‎ ‎  Somebody might say to me, "Don't work so hard. You'll kill yourself." My mother will say to me,"Why do you press all your brains out on this page for someone else?" So it' s very vivid. That' s the way she talks.‎ ‎  ●Have many readers told you that the Chinese mother in your book reminded them of the typical Jewish (有癖好的) mother?‎ ‎  Many people have told me that. I think the mother-daughter relationship is very intense in both cases. Culturally there is an acceptance that mothers have the power to tell their children, especially their daughters, how to conduct their lives - not simply up until the time they are 18, but for the rest of their lives. However, when children grow up in a different culture from their parents', they tend to keep more secrets from their parents. The children think, "They just wouldn' t understand that I had to do this." And that can really create a gap, and it can grow as the number of secrets grows.‎ ‎  [语篇解读] 这是一篇人物访谈纪实。通读这篇访谈录,我们可以了解华裔美国作家Ms Tan创作第二部作品的经历。‎ ‎  1.Based on the questions in this interview, what do you think Ms Tan' s profession is?‎ ‎  A. A journalist. B. A storywriter.‎ ‎  C. An interviewer. D. An interviewee.‎ ‎  [解析] 推理判断题。根据文中的"your new novel,a second novel"等关键词及访谈内容可知,Ms Tan是一名小说家。C、D两项有一定的干扰性,注意题干中问的是"profession(职业)"。‎ ‎  [答案] B ‎  2.What' s TRUE about Tan' s second book?‎ ‎  A. It' s about her real life in America.‎ ‎  B. The name of the book is The Joy Luck Club.‎ ‎  C. It is the result of many times of careful thought.‎ ‎  D. It includes many works of her mother.‎ ‎  [解析] 细节判断题。根据文中的"it took me six or seven attempts at a second novel before I started and completed this one"可知,Tan在完成这部书的写作之前曾经历过六、七次尝试,说明这本书是她经过多次深思熟虑的产物。‎ ‎  [答案] C ‎  3.Which question is NOT answered in the interview?‎ ‎  A. How does she think of her mother' s language?‎ ‎  B. How many books does she plan to write?‎ ‎  C. When did she visit China?‎ ‎  D. How is generation gap created?‎ ‎  [解析] 细节理解题。"她计划写几本书?"这个问题在文中找不到答案,其他三项文中均有涉及。‎ ‎  [答案] B ‎  4.We can infer that________.‎ ‎  A. Tan' s mother is a good storyteller ‎  B. Tan plans to write another book about her mother ‎  C. Tan plans to return to China ‎  D. Tan' s mother is hard to communicate with because of personality ‎  [解析] 推理判断题。根据文中的"She also told me many,many stories,because my mother doesn't generalize"和"So it's very vivid. That's the way she talks"可知,Tan的母亲是一位善于讲故事的人。‎ ‎  [答案] A ‎  5.The last paragraph mainly talks about________.‎ ‎  A. how to keep secrets from parents ‎  B. how to deal with the mother-daughter relationship ‎  C. how to conduct the lives ‎  D. how the generation gap comes about ‎  [解析] 段落主旨题。此段大意是:传统地认为母亲有教育孩子们一生如何生活的权力,而孩子们随着年龄的增长,他们不愿将自己的秘密告诉父母,从而产生了代沟。因此本段主要阐述的是代沟是如何产生的。‎ ‎  [答案] D ‎  ‎ ‎(第57篇)‎ ‎  "Luck is a matter of preparation meeting opportunity," said the American talk show host Oprah Winfrey. I' ve never watched her show, but when a self-made billionaire gives life advice it' s probably worth listening to.‎ ‎  Her point is that blind luck is very rare. You may have to be lucky to find a good job these days but that does not mean you should sit at home waiting for the opportunity to come to you. If you' re a Chinese,you may already be familiar with the tale of a farmer waiting by a tree stump (树桩) for a rabbit to run out and break its neck.‎ ‎  A book by the UK psychologist Richard Wiseman, called The Luck Factor, argues we can all make ourselves luckier. It' s not about going to a temple to burn some incense (香) in hopes that the gods will give you good fortune; it' s practical advice you can follow each day.‎ ‎  Wiseman conducted an experiment as part of his studies. First he divided volunteers into two groups:those who said they were lucky in life and those who said they were not. He gave everyone a newspaper and asked them to look through it to count how many photographs it had inside. On average, the unlucky people took about two minutes to count the photographs while the lucky people took just seconds. Why? On the second page of the newspaper, a command, "Stop counting. There are 43 photographs in this newspaper," was written in big letters. The unlucky people mostly did not spot the message.‎ ‎  It's easy to compare this situation to a young person looking for jobs in a local paper. They might search so hard for one type of position that they miss an even better opportunity. People who are "lucky", in fact, keep an open mind and don't go through the same routine every day.‎ ‎  I first came to China in 2002 when it was considered a rather strange thing to do. Like many foreigners,my plan was to teach English for one year. Seven years later, and still here, I' ve had many great opportunities such as writing for newspapers and magazines. I did not dream these would have been possible. I' ve also never been sick, had an accident, got into a fight or had problems with the police. Coincidence? After reading about Professor Wiseman' s studies I think not.‎ ‎  As Wiseman advises, I usually trust my own judgment. Your friends and parents may give you advice based on rational thinking, but it's important to consider how you feel about each choice you make. Your feeling acts as a warning for a potential problem.‎ ‎  Finally, try to turn bad luck into good. Even if you do fall down and break a leg, the time spent at home can be used wisely to study English.‎ ‎  [语篇解读] 本文为议论文。作者认为命运只垂青那些有所准备的人。身处逆境,不要怨天尤人,要想方设法化厄运为好运。每个人都可以做自己命运的主人。‎ ‎  1.Which of the following proverbs most agrees with the writer' s point?‎ ‎  A. Make the best of a bad job.‎ ‎  B. Rome was not built in a day.‎ ‎  C. All is not gold that glitters.‎ ‎  D. A good heart conquers ill fortune.‎ ‎  [解析] 推理判断题。Make the best of a bad job意为"身处山穷水尽,力争柳暗花明",与最后一段作者的观点"try to turn bad luck into good(把厄运变好运)"最为接近。故答案为A。Rome was not built in a day意为"伟业非一日可成";All is not gold that glitters意为"闪光的东西并不都是金子";A good heart conquers ill fortune意为"善心克厄运"。‎ ‎  [答案] A ‎  2.What do you know about Oprah Winfrey?‎ ‎  A. She became famous through her family background.‎ ‎  B. She was very lucky and seldom suffered setbacks in her life.‎ ‎  C. She is a British talk show host.‎ ‎  D. She became successful by her own effort.‎ ‎  [解析] 细节理解题。根据第一段中的a self-made billionaire可知美国talk show主持人oprah Winfrey是从白手起家,通过自我奋斗而成名的。‎ ‎  [答案] D ‎  3.The writer quoted the Chinese tale of a farmer in order to show________.‎ ‎  A. luck is in your own hand ‎  B. bad luck can turn into good ‎  C. you should not sit at home waiting for the opportunity to come to you ‎  D. man can conquer nature ‎  [解析] 推理判断题。根据第二段内容可以判断,作者引用中国"守株待兔"的故事是为了说明"人们不应该待在家里等待机会的降临"的观点。‎ ‎  [答案] C ‎  4.From the experiment Wiseman drew the conclusion that________.‎ ‎  A. lucky people are quick-minded ‎  B. unlucky people are slow to read ‎  C. lucky people often have an open mind ‎  D. unlucky people are more creative ‎  [解析] 细节理解题。根据第四、五段内容,尤其是第五段最后一句可知,Wiseman通过试验得出结论:"幸运的人"具有开放的心态,做事不墨守成规。‎ ‎  [答案] C ‎  5.The underlined word "spot" is the closest in meaning to"________".‎ ‎  A. recognize B. mark ‎  C. locate D. receive ‎  [解析] 词义猜测题。根据上文语境及The unlucky people mostly did not spot the message可知spot在句中用作及物动词,是"发现,识别"之意。‎ ‎  [答案] A ‎  [长难句解读] You may have to be lucky to find a good job these days but that does not mean you should sit at home waiting for the opportunity to come to you.‎ ‎  but前后为两个并列分句。在第一个分句中不定式短语to find a good job用作结果状语。第二个分句含有一个宾语从句,现在分词短语waiting for the opportunity to come to you作伴随状语。‎ ‎(第58篇)‎ ‎  Can you believe your eyes? A recent experiment suggests that the answer to that question may depend on your age.‎ ‎  Martin Doherty, a psychologist at the University of Stirling in Scotland, led the team of scientists. In this experiment, Doherty and his team tested the perception(观察力) of some people, using pictures of some orange circles. The researchers showed the same pictures to two groups of people. The first group included 151 children aged 4 to 10, and the second group included 24 adults aged 18 to 25.‎ ‎  The first group of pictures showed two circles alone on a white background. One of the circles was larger than the other, and these people were asked to identify the larger one. Four-year-olds identified the correct circle 79 percent of the time. Adults identified the correct circle 95 percent of the time.‎ ‎  Next, both groups were shown a picture where the orange circles, again of different sizes, were surrounded by gray circles. Here's where the trick lies in. In some of the pictures, the smaller orange circle was surrounded by even smaller gray circles - making the orange circle appear larger than the other orange circle, which was the real larger one. And the larger orange circle was surrounded by even bigger gray circles - so it appeared to be smaller than the real smaller orange circle.‎ ‎  When young children aged 4 to 6 looked at these tricky pictures, they weren't fooled-they were still able to find the bigger circle with roughly the same accuracy as before. Older children and adults, on the other hand, did not do as well. Older children often identified the smaller circle as the larger one, and adults got it wrong most of the time.‎ ‎  As children get older, Doherty said, their brains may develop the ability to identify visual context. In other words, they will begin to process the whole picture at once: the tricky gray circles, as well as the orange circle in the middle. As a result, they're more likely to fall for this kind of visual trick.‎ ‎  1. Doherty and his team of scientists did an experiment to evaluate________.‎ ‎  A. children's and adults' eye-sight ‎  B. people's ability to see accurately ‎  C. children's and adults' brains ‎  D. the influence of people's age ‎  [解析] 考查细节理解能力。第二段第二行可以得出结论。‎ ‎  [答案] B ‎  2. When asked to find the larger circle,________.‎ ‎  A. children at 6 got it wrong 79 % of the time with no gray ones around ‎  B. only adults over 18 got it right 95% of the time with gray ones around ‎  C. children at 4 got it right about 79 % of the time with gray ones around ‎  D. adults got it right most of the time with gray ones around ‎  [解析] 考查细节理解能力。第三段清晰地说明了辨别圆圈的比例。‎ ‎  [答案] C ‎  3. According to the passage, we can know that________.‎ ‎  A. a smaller orange circle appears bigger on a white background ‎  B. an orange circle appears bigger than a gray one of the same size ‎  C. a circle surrounded by other circles looks bigger than its real size ‎  D. a circle surrounded by bigger ones looks smaller than its real size ‎  [解析] 考查细节理解能力,对第四段整体的理解和第五段最后一句可以得出答案。‎ ‎  [答案] D ‎  4. Visual context may work when children get older than________.‎ ‎  A. 4 B. 6‎ ‎  C. 10 D. 18‎ ‎  [解析] 考查推理判断能力。从第五段和第六段第一句判断得出答案。‎ ‎  [答案] B ‎  5. Why are younger children not fooled?‎ ‎  A. Because they are smarter than older children and adults.‎ ‎  B. Because older people are influenced by their experience.‎ ‎  C. Because people's eyes become weaker as they grow older.‎ ‎  D. Because their brain can hardly notice related things together.‎ ‎  [解析] 考查推理判断能力。第四、五、六段提及到46岁小孩和6岁以上小孩的区别是判断的依据。‎ ‎  [答案] D ‎(第59篇)‎ ‎  Bill Fulton doesn' t remember losing his wallet, but its return helped him remember the past. The leather stayed smooth and the cowboy design unblemished (完美无缺的). And when he looked inside, the contents brought back memories from 1946, when he apparently dropped the wallet behind the balcony bleachers (露天看台) in the Baker Middle School gym.‎ ‎  Fulton' s Social Security Card and bicycle license, bearing the address where he lived during his teenage years, were positioned in their respective compartments, apparently untouched since the year after World War Il ended.‎ ‎  Worker Nathan Osborne found the wallet - along with old homework, lost library books and a 1964 talent show program - while removing the bleachers for renovations on June 17. It was brought to Fulton' s door the following day by Melanie Trindle, the Baker Middle School secretary.‎ ‎  Middle School Principal Mindi Vaughan said the brown pine bleachers were connected to the gym balcony' s brick wall and had remained in the same place since the school, known as the Helen M. Stack Building, opened in 1936.‎ ‎  Fulton, 78, said he probably lost the wallet while cheering for the Baker High basketball team with a group of friends. Fulton said the bicycle ID was needed because he delivered medicine for Rodamar Drug.He was surprised, however, that his bicycle ID wasn't inside. He said he always kept it there.‎ ‎  But rather than focus on what was inside the wallet, Fulton said the recovery has led him to reflect on his life - one that took him to the Korean War and Berlin before the return to Baker City. He worked at Ellingson Lumber Company for 30 years, from March 1964 to April 1994.‎ ‎  Since his retirement, Fulton has enjoyed spending time with his 11yearold black dog, Smokey. The two often hike the nearby mountains.‎ ‎  Trindle' s knock on the door, wallet in hand, induced a slight ripple in his life, prompting him to consider the times he hadn't thought about in many years. "Where did all the times go?" Fulton said with a deep sigh. "It' s hard to believe that the times have gone so fast."‎ ‎  [语篇解读] 本文是记叙文。一个丢失了几十年的钱包居然被找到,而且还给了失主。钱包勾起了失主对过去岁月的回忆。‎ ‎  1.When the wallet was found, it________.‎ ‎  A. was still in good condition ‎  B. was a little damaged ‎  C. had just lost color ‎  D. was just repaired ‎  [解析] 细节理解题。从文章第一段第二句可知,钱包的皮还很光滑,上面的图案也保持完好,因此A项正确。‎ ‎  [答案] A ‎  2.The wallet was important to Fulton because ________.‎ ‎  A. he could get his bicycle ID back ‎  B. it called up his memories of the past ‎  C. it could help him to find his student ID ‎  D. it made him think of his school days ‎  [解析] 推理判断题。综合全文尤其是第一段和最后一段的信息可知,钱包勾起了Fulton对往昔岁月的回忆。‎ ‎  [答案] B ‎  3.Fulton lost the wallet when________.‎ ‎  A. he was doing his homework on the balcony bleachers ‎  B. he was doing some exercise in the Baker Middle School gym ‎  C. he was removing the bleachers for renovations on June 17‎ ‎  D. he was watching a basketball match with his friends ‎  [解析] 细节理解题。从文章第五段可知,Fulton和朋友们在一起看篮球赛,在为一支篮球队助威时他丢失了钱包。‎ ‎  [答案] D ‎  4.After he retired, Fulton________.‎ ‎  A. lived a peaceful and pleasant life ‎  B. often regretted losing his wallet ‎  C. lived a hard life by raising dogs in the mountains ‎  D. quite missed his past life ‎  [解析] 推理判断题。分析文章倒数第二段可知,Fulton退休以后经常牵着狗到山里散步,可见生活非常惬意。‎ ‎  [答案] A ‎  5.Which of the following best describes Fulton' s feeling when the wallet was returned?‎ ‎  A. Thankful.       B. Overjoyed.‎ ‎  C. Surprised. D. Doubtful.‎ ‎  [解析] 推理判断题。文章虽然没有明确说明Fulton接到钱包时的反应,但丢了那么多年的钱包失而复得,他应该感到吃惊。‎ ‎  [答案] C ‎  [长难句解读] And when he looked inside,the contents brought back memories from 1946,when he apparently dropped the wallet behind the balcony bleachers(露天看台)in the Baker Middle School gym.语意:当他看钱包里面时,里面的东西把他的记忆带回到了1946年,当时他显然是把钱包丢在了贝克中学体育场的露天看台上。‎ ‎  本句是一个复合句。第一个when引导时间状语从句,第二个when引导非限制性定语从句,修饰1946。‎ ‎(第60篇)‎ ‎  Pop stars today enjoy what once only belonged to the royalty (皇室). Wherever they go, people turn out in their thousands to greet them. The crowds go wild trying to catch a brief glimpse of their smiling, colorfully dressed idols. The stars are transported in their chauffeur driven RollsRoyces, private helicopters or executive aeroplanes. They are surrounded by a ‎ permanent entourage(随从) of managers, press agents and bodyguards. Photographs of them appear regularly in the press and all their comings and goings are reported,for, like royalty, pop stars are news. If they enjoy many of the privileges of royalty, they certainly share many of the inconveniences as well. It is dangerous for them to make unscheduled appearances in public.They must be constantly shielded from the adoring crowds who idolize them. They are no longer private individuals, but public property. The financial rewards they receive for this sacrifice cannot be calculated, for their rates of pay are great.‎ ‎  And why not? Society has always rewarded its top entertainers lavishly. The great days of Hollywood have become legendary: famous stars enjoyed fame, wealth and adulation (奉承) on an alltime scale. By today' s standards, the excesses of Hollywood do not seem quite so spectacular. A single gramophone record nowadays may earn much more in royalties than the films of the past ever did. The competition for the title "Top of the Pops" is fierce, but the rewards are truly huge.‎ ‎  It is only right that the stars should be paid in this way. Don't the top men in industry earn enormous salaries for the service they perform to their companies and their countries? Pop stars earn vast sums in foreign currency - often more than large industrial companies - and the taxman can only be grateful for their massive annual contributions to the exchequer(国库). So who would begrudge them their rewards?‎ ‎  It' s all very well for people in boring jobs to complain about the successes and rewards of others. People who make envious remarks should remember that the most famous stars represent only the tip of the iceberg. For every famous star, there are hundreds of others struggling to earn a living. A man working in a steady job and looking forward to a pension at the end of it has no right to expect very high rewards. He has chosen security and peace of mind, so there will always be a limit to what he can earn. But a man who attempts to become a star is taking enormous risks. He knows at the outset that only a handful of competitors ever get to the very top. He knows that years of concentrated effort may be rewarded with complete failure.But he knows, too, that the rewards for success are very high indeed: they are the payback for the huge risks involved and once he makes it, he will certainly earn them. That' s the essence of private enterprise.‎ ‎  [语篇解读] 明星的生活总是令人羡慕不已。他们的生活中有光鲜的一面:不菲的收入,以豪华汽车和私人飞机代步,一言一行都成为人们关注的焦点......然而,我们也应看到,他们活得很累,他们为成功所付出的艰辛和代价是一般人难以想象的。‎ ‎  1.The author develops the passage mainly by ________.‎ ‎  A. comparing different ideas ‎  B. giving explanations ‎  C. inferring ‎  D. listing typical examples ‎  [解析] 篇章结构题。本文主要通过解释向读者阐明道理。‎ ‎  [答案] B ‎  2.The underlined word "begrudge" in the third paragraph is closest in meaning to ________.‎ ‎  A. be jealous of B. be satisfied with ‎  C. be anxious about D. be crazy about ‎  [解析] 词义猜测题。结合上下文的语境判断本句意为:谁会对他们所得到的感到妒忌、不满呢?接下来作者讲了一般人从事枯燥但稳定的工作,所以这里意思是:从事稳定工作的人会妒忌这些明星。‎ ‎  [答案] A ‎  3.According to the passage, which of the following can match the view of the author?‎ ‎  A. He who laughs last laughs best.‎ ‎  B. If you venture nothing, you will gain nothing.‎ ‎  C. He who makes no mistakes makes nothing.‎ ‎  D. Success belongs to the persevering.‎ ‎  [解析] 推理判断题。根据最后一段可知,想成为明星就要冒更大的风险,然而一旦成功,回报也是丰厚的,那些选择了稳定工作的人是没有权利期待高额回报的。"If you venture nothing,you will gain nothing."意为:不入虎穴,焉得虎子,即要成功就要冒险。‎ ‎  [答案] B ‎  4.The author holds a(n) ________attitude towards the high income of pop stars.‎ ‎  A. critical B. approval ‎  C. optimistic D. indifferent ‎  [解析] 推理判断题。根据全文可知,作者认为明星们理应得到高收入,因为他们为此付出了高昂的代价。‎ ‎  [答案] B ‎  5.Which of the following is true according to the passage?‎ ‎  A. People are blind in idolizing stars.‎ ‎  B. There is fierce competition in becoming pop stars.‎ ‎  C. The government taxes pop stars very little.‎ ‎  D. Pop stars' life is more luxurious than that of royalty.‎ ‎  [解析] 细节判断题。根据文章最后一段的"He knows at the outset that only a handful of competitors ever get to the very top."以及第二段的最后一句可知答案。‎ ‎  [答案] B ‎(第1篇)‎ ‎  Do American children still learn handwriting in school? In the age of the keyboard, some people seem to think handwriting lessons are on the way out. 90% of teachers say they are required to teach handwriting.But studies have yet to answer the question of how well they are teaching it. One study published this year found that about three out of every four teachers ‎ say they are not prepared to teach handwriting. Some teachers are teaching handwriting by providing instruction for 10 15 minutes a day, and then other teachers who basically teach it for 60 to 70 minutes a day - which really for handwriting is pretty much.‎ ‎  Many adults remember learning that way - by copying letters over and over again. Today' s thinking is that short periods of practice are better. Many experts also think handwriting should not be taught by itself.Instead, they say it should be used as a way to get students to express ideas. After all, that is why we write.‎ ‎  Handwriting involves two skills. One is legibility,_which means forming the letters so they can be read. The other is fluency - writing without having to think about it. Fluency continues to develop up until high school.‎ ‎  But not everyone masters these skills. Teachers commonly report that about onefourth of their kids have poor handwriting. Some people might think handwriting is not important any more because of computers and voice recognition programs.‎ ‎  But Steve Graham at Vanderbilt says word processing is rarely done in elementary school, especially in the early years. American children traditionally first learn to print, and then to write in cursive, which connects the letters. But guess what we learned from a spokeswoman for the College Board, which administers the SAT college admission test. More than 75 percent of students choose to print their essay on the test rather than write in cursive.‎ ‎  [语篇解读] 文章介绍了在键盘普及的年代美国的书法教学情况。‎ ‎  1.We can learn ________ from Paragraph 1.‎ ‎  A. teaching handwriting is a basic requirement in teaching job ‎  B. most teachers prefer to teach handwriting ‎  C. teachers spend little time in teaching handwriting ‎  D. a keyboard has taken the place of the handwriting entirely ‎  [解析] 推理判断题。根据第一段中"90% of teachers say they are required to teach handwriting"可知书法教学是基本的教学要求,A项为正确推论。根据"One study published this year found that about three out of every four teachers say they are not prepared to teach handwriting"可知,很多老师不愿意教书法课,B项错误;根据第一段末句可知C项错误;根据全段判断,只是说键盘普及使一些人认为书法渐渐过时了,D项错误。‎ ‎  [答案] A ‎  2.Which of the following is WRONG for traditional handwriting in the USA?‎ ‎  A. The students are taught by practicing a long period.‎ ‎  B. The letters are repeated many times.‎ ‎  C. Handwriting includes two skills.‎ ‎  D. To write in cursive is taught first.‎ ‎  [解析] 细节认定题。根据第二段可知,传统的书法教学重复很多遍并且持续时间长,因此A、B两项内容正确。根据第三段第一句可知C项内容正确;根据最后一段第二句可知D项内容错误。‎ ‎  [答案] D ‎  3.The underlined word "legibility" in Paragraph 3 means ________.‎ ‎  A. easy to read B. complex ‎  C. unexpected D. unreadable ‎  [解析] 推测词义题。根据下文"which means forming the letters so they can be read"可推测画线词的意思是"清楚,易读"。‎ ‎  [答案] A ‎  4.The best title for the passage is ________.‎ ‎  A. How to improve handwriting in school ‎  B. Right or wrong: the death of handwriting ‎  C. Handwriting involves two skills ‎  D. Handwriting lessons are on the way out ‎  [解析] 标题归纳题。作者对美国书法教学的现状作了叙述,没有自己的主观评论,故选择B。A项没有涉及,C项只是其中一部分内容,D项只是一部分人的观点。‎ ‎  [答案] B ‎  5.The author' s attitude towards whether still to learn handwriting in school is________.‎ ‎  A. negative B. objective ‎  C. critical D. optimistic ‎  [解析] 态度判断题。根据全文的叙述,作者只是将美国书法教学的现状和不同人的观点呈现出来,并没有发表自己的见解,因此作者持客观的态度。‎ ‎  [答案] B ‎  [长难句解读] One study published this year found that about three out of every four teachers say they are not prepared to teach handwriting.今年公布的一项研究发现,大约每四个教师中就有三个说他们不准备教书法了。published this year为过去分词短语作后置定语,found后面为宾语从句,其中又含有一个宾语从句they are not prepared to teach handwriting。‎ ‎   (第2篇)‎ ‎  You know how wonderful you are, and you know that others know how wonderful you are, but what do you do when admiration crosses over the line into jealousy? For most teens there will come a day when you realize that one of your friends is jealous and that this jealousy is hurting your friendship. When this happens it can seem like there is nothing that you can do, but the good news is that there is. Don't let jealousy spoil your relationships. Tackle it headon and you might be back to normal much sooner than you think.‎ ‎  It can be hard to walk up to a friend and ask them what the problem is, but if you want to save your friendship you'll have to do just that. Don't approach them and ask why they are jealous of you (unless of course you want to appear totally conceited), just take some time alone with them and let them know that you've been feeling like there's been something coming between you. If they refuse to respond, then use the opportunity to explain how you ‎ have been feeling. Chances are that something you say will strike a nerve and your friend will open up as well.‎ ‎  When you figure out what is annoying your friend, ask him or her what (s)he thinks would make the situation better. If, for example, (s)he says that (s)he feels like (s)he doesn't get to spend any time with you because of your being off with your new friends from tee swim team then maybe you could invite her along the next time or block off one day a week for just the two of you. Remember, though, that whatever solution you decide on should be a compromise. Don't limit your own talents or opportunities simply because your friend is unhappy. Try instead to include him or her in your new life and see how that works out.‎ ‎  Even the best of friendships can be tinged by jealousy. This destructive emotion is rarely productive and can turn best friends into worst enemies. Before taking extreme action, chat with your jealous friend to see if the two of you can work out a compromise.If you can't, be prepared to know exactly how far you will go to keep your friendand how far you won't.‎ ‎  解读:本文是关于人际关系的话题。尤其是青少年朋友之间会因为嫉妒而使相互间的友谊被破坏,本文着重讲述如何处理被朋友嫉妒及处理过程中可能遇到的困难。‎ ‎  1. According to the author, the jealousy emotion is________.‎ ‎  A. normal B. productive ‎  C. destructive D. extreme ‎  [解析] 细节理解题,由第一段第二句...that this jealousy is hurting,your friendship.可知。‎ ‎  [答案] C ‎  2. Which of the following is NOT mentioned as a way to confront your friend when jealousy happens?‎ ‎  A. Walking up to him/her and asking him/her why he/she is jealous of you.‎ ‎  B. Walking up to him/her and asking him/her what the problem is.‎ ‎  C. Spending some time with him/her and letting him/her know how you feel.‎ ‎  D. Spending some time with him/her and letting him/her know you think there's something between you.‎ ‎  [解析] 细节理解题。由第二段第二句Don't approach them and ask why they are jealous of you可知。‎ ‎  [答案] A ‎  43. The underlined "it" in the first paragraph refers to________.‎ ‎  A. friendship B. relationship ‎  C. admiration D. jealousy ‎  [解析] 细节理解题。由文章第一段可知。‎ ‎  [答案] D ‎  4. What can be inferred from the last two paragraphs?‎ ‎  A. There's always a solution to solve the problem of jealousy.‎ ‎  B. Jealousy can turn best friends into worst enemies.‎ ‎  C. You may lose a friend to keep your own gifts, chances or self development.‎ ‎  D. You should go a long way with your friend to work out a solution ‎  [解析] 推理判断题。‎ ‎  [答案] C ‎  5. The purpose of the passage is________.‎ ‎  A. to explain what causes jealousy ‎  B. to offer some advice on making friends ‎  C. to introduce the way to cope with a jealous friend ‎  D. to explain how destructive the jealous emotion is ‎  [解析] 作者意图题。通读全文可知,本文酌写作目的是介绍处理有嫉妒心朋友的方法。‎ ‎  [答案] C ‎(第61篇)‎ ‎  Honesty comes in many forms. First there's self-honesty. Is what people see the real article or do you appear_through_smoke_and_mirrors? I find that if I try to be something I'm not,I feel unsure of myself and take out a part from my PBA(personal bank account). I love how singer Judy Garland put it, "Always be a firstclass version (版本) of yourself, instead of a secondclass version of somebody else."‎ ‎  Then there's honesty in our actions. Are you honest at school,with your parents, and with your boss? If you've ever been dishonest,I think we all have, try being honest, and notice how whole it makes you feel. Remember, you can't do wrong and feel right. This story by Jeff is a good example of that:‎ ‎  In my second year of study, there were three kids in my math class who didn't do well. I was really good at it. I would charge them three dollars for each test that I helped them pass. I'd write on a little piece of paper all the right answers, and hand them off.‎ ‎  At first I felt like I was making money, kind of a nice job. I wasn't thinking about how it could hurt all of us. After a while I realized I shouldn't do that anymore, because I wasn't really helping them. They weren't learning anything, and it would only get harder down the road. Cheating certainly wasn't helping me.‎ ‎  It takes courage to be honest when people all around you are getting away with cheating on tests, lying to their parents, and stealing at work. But, remember, every act of honesty is a deposit(储蓄)into your PBA and will build strength.‎ ‎  [语篇解读] 诚实可以反映在生活的各个方面,对于我们来说,首先要做一个诚实的自我。‎ ‎  1.The underlined part "appear through smoke and mirrors" in the first paragraph means"________".‎ ‎  A. to be honest ‎  B. to he unreal ‎  C. to become clear ‎  D. to come from an imagined world ‎  [解析] 猜测词义题。第一段第二句:人们看到的是真实的你还是出现在雾里、镜里的你?由此可知:appear through smoke and mirrors为"to be unreal",表示"不真实"的意思。‎ ‎  [答案] B ‎  2.Which of the following can best explain Judy Garland's words?‎ ‎  A. Be your true self rather than follow others.‎ ‎  B. Don't copy others or you can't be the first class.‎ ‎  C. Make efforts to be the first instead of the second.‎ ‎  D. Don't learn from others unless they're excellent.‎ ‎  [解析] 推理判断题。综观全文,这句话可以理解为:做一个真正的自我,不要一味效仿别人。‎ ‎  [答案] A ‎  3.What does the author expect to show by Jeff's story?‎ ‎  A. Honesty can be of great help.‎ ‎  B. A bad thing can be turned into a good one.‎ ‎  C. Helping others cheat can do good to nobody.‎ ‎  D. One should realize the wrong in his bad deeds.‎ ‎  [解析] 事实细节题。第二段最后一句告诉我们:通过讲述Jeff的故事,作者希望人们做错了事情,应该自己意识到错误。‎ ‎  [答案] D ‎  4.In the last paragraph the author mainly wants to express________.‎ ‎  A. one must be brave to be honest ‎  B. it's difficult to be honest when others are not ‎  C. one should be honest when making a deposit ‎  D. honesty in one's actions can help him in the future ‎  [解析] 主旨大意题。最后一段告诉我们:人们减实的举动,会对自己的未来有益。‎ ‎  [答案] D ‎  ‎ ‎(第62篇)‎ ‎  Teenagers who drink alcohol are at higher risk of becoming victims of violence, a Cardiff University study has found.‎ ‎  A team from the School of Dentistry' s Violence Research Group studied drinking habits in children aged 11 - 16 in England. They found not only a link between drink and violence but also that children who drank were more likely to be hit, even if they weren' t violent themselves.‎ ‎  The researchers are now calling for measures to prevent alcohol misuse to reduce injury risk. Current policy focuses on reducing aggression but this research shows that there should be equal effort to reduce victimization.‎ ‎  More than 4,000 children were surveyed at 13 schools at four local authorities in the North, the Midlands, London, and the South. The study found that 25% of 11-year-olds were drinking monthly and 3.6% daily, with 12.8% admitting to getting drunk 3 to 5 times a year. By the age of 16, 40% were drinking weekly and 6.2% were drinking every day. The research also showed 22.6% of 16-year-olds were getting drunk more than 21 times a year.‎ ‎  The study, which has just been published in the Journal of Adolescence, found a strong link between frequency of drinking and frequency of hitting other people.‎ ‎  However, children who reported drinking monthly were also three times more likely to be hit. Adolescents who drank but didn't get into fights were more likely to be hit than those who did fight.‎ ‎  Professor Jonathan Shepherd, who led the research, said a lot of previous alcohol-related violence work had focused on the offenders rather than the victims. His team is calling for more prevention work from parents and teachers in the first two years of secondary school by taking advantage of the "teachable moment",that is, immediately after a student has missed school because of drunkenness.‎ ‎  Previous work by Professor Jonathan Shepherd has shown drinkers may be more at risk of violence because of reduced physical co-ordination, poor decision-making in threatening situations and isolation while out late at night.‎ ‎  He said, "This new study seems to be the first to show a direct link between alcohol misuse and victimization. There now needs to he much more effort put into reducing alcohol misuse in order to reduce injury."‎ ‎  [语篇解读] 本文介绍了一项关于青少年酗酒的研究。该研究发现,酗酒的青少年更容易成为暴力的受害者。‎ ‎  1. The main idea of the passage is that teenagers who drink alcohol________.‎ ‎  A. often offend other people ‎  B. often become injured ‎  C. are more likely to become victims of violence ‎  D. are more likely to fight with others ‎  [解析] 主旨大意题。根据第一段中的"Teenagers who drink alcohol are at higher risk of becoming victims of violence"可知答案为C。‎ ‎  [答案] C ‎  2.The underlined word "aggression" in the third paragraph probably means "________".‎ ‎  A. violence         B. sad feelings ‎  C. bad manners D. drunkenness ‎  [解析] 词义猜测题。根据上下文尤其是本句后半句可知,aggression与victimization意思相反,所以aggression是"暴力行为"的意思。‎ ‎  [答案] A ‎  3.The research argues that more effort should be made to________.‎ ‎  A. reduce aggression ‎  B. prevent fighting ‎  C. reduce victimization ‎  D. prevent drinking alcohol ‎  [解析] 细节理解题。根据文章第三段最后一句中的"this research shows that there should be equal effort to reduce victimization"可知答案为C。‎ ‎  [答案] C ‎  4.Drinkers may be more at risk of violence because of all the following EXCEPT ________.‎ ‎  A. reduced physical co-ordination ‎  B. isolation while out late at night ‎  C. a higher frequency of hitting other people ‎  D. poor decision-making in threatening situations ‎  [解析] 细节理解题。根据文章倒数第二段可知答案。‎ ‎  [答案] C ‎  5.Which of the following statements is NOT true according to Professor Shepherd' s research?‎ ‎  A. The frequency of children getting drunk increases with age.‎ ‎  B. His previous alcohol-related violence work had focused on the offenders.‎ ‎  C. Some children missed school because of drunkenness.‎ ‎  D. This new study shows a direct link between alcohol misuse and victimization.‎ ‎  [解析] 细节理解题。B项偷换概念,原文为"a lot of previous alcohol-related violence work had focused on the offenders",这些研究不一定就是这位教授以前的研究。‎ ‎  [答案] B ‎(第63篇)‎ ‎  Audrey Hepburn won an Academy Award as Best Actress for her first major American movie, Roman Holiday, which was released in 1953. But she is remembered as much for her aid work as for her acting.‎ ‎  Born in Belgium in 1929, Audrey' s father was British and her mother was Dutch.‎ ‎  Audrey was sent to live at a British school for part of her childhood. During World War II , she lived and studied in the Netherlands. Her mother thought it would be safe from German attacks. Audrey studied dance as a teenager and during college when she returned to London after the war. But she realized she wasn' t going to be a ballerina. So she began taking acting parts in stage shows. Later she began to get small parts in movies.‎ ‎  But it was Audrey Hepburn' s move to America that brought her true fame. In 1951 she played the character "Gigi" in the Broadway play of the same name to great critical praise. Two years later, Roman Holiday made her a star at the age of 24.‎ ‎  Audrey made more than 25 movies. Among her most popular roles was Holly Golightly in Breakfast at Tiffany's in 1961. Three years later she played Eliza Doolittle in My Fair Lady.‎ ‎  She was married two times and had one son by each husband. In 1989, the UN Children' s Fund named Audrey a goodwill ambassador. She travelled all over the world in support of UNICEF projects. The UN agency said she was a tireless worker. She often gave 15 interviews a day to gain money and support for UNICEF projects.‎ ‎  Audrey Hepburn often said her loyalty to UNICEF was the result of her experiences as a child during World War Ⅱ. She said she knew what it was like to be starving and to be saved by international aid. She was a goodwill ambassador until her death in 1993 from colon cancer.‎ ‎  [语篇解读] 本文介绍著名影星Audrey Hepburn(奥黛丽·赫本)的生平事迹。1953年主演的《罗马假日》使她一举成名。从1989年直至1993年去世,赫本一直担任联合国儿童基金会的特使,为拉美和非洲儿童提供了很大帮助。‎ ‎  1.In Paragraph 1, "her aid work" means ________.‎ ‎  A. winning an Academy Award as Best Actress ‎  B. taking acting parts in stage shows ‎  C. making movies ‎  D. acting as a goodwill ambassador for UNICEF ‎  [解析] 推理判断题。此题应根据全文内容来判断。第一段是全文的概括,其重心在后面,即奥黛丽·赫本尽管获得奥斯卡最佳女主角奖,但她担当的援助工作同样使人铭记。再结合文章接下来的叙述可知答案为D。‎ ‎  [答案] D ‎  2.The reason why Audrey lived and studied in the Netherlands was that ________.‎ ‎  A. she wanted to be a ballerina ‎  B. her parents were from Britain ‎  C. the education there was excellent ‎  D. it was safe there ‎  [解析] 细节理解题。根据第三段中的"Her mother thought it would be safe from German attacks"和"she returned to London after the war"可知D项正确。‎ ‎  [答案] D ‎  3.We can infer from the passage that ________.‎ ‎  A. Audrey' s parents lived in Germany during World War Ⅱ ‎  B. Audrey lived in America in the 1950s ‎  C. Audrey was made to give up dancing ‎  D. the character "Gigi" in the Broadway play was her most popular role ‎  [解析] 推理判断题。根据第二段和第三段前半部分可知,A项错误;根据第三段中"she realized she wasn't going to be a ballerina"可知是她主动放弃的,C项错误;根据第四段中"In 1951 she played the character'Gigi'in the Broadway play"及后文可判断,这个角色并不是她扮演的最有名的角色,因此D项错误。由文章第四、五两段可知B项正确。‎ ‎  [答案] B ‎  4.________is NOT mentioned in the passage about Audrey Hepburn.‎ ‎  A. Marriage B. Identity ‎  C. Contribution D. Religion ‎  [解析] 细节理解题。可以运用排除法。她的婚姻、身份和贡献在文章中都提到了,唯独没有提及她的宗教信仰,故选D。‎ ‎  [答案] D ‎  5.________is the right order for Audrey' s life.‎ ‎  ①The first time she began to play in movies.‎ ‎  ②She returned to London from the Netherlands.‎ ‎  ③She won an Academy Award as Best Actress.‎ ‎  ④She travelled all over the world in support of UNICEF projects.‎ ‎  ⑤She played a part in My Fair Lady.‎ ‎  A.②①③⑤④ B. ①②③⑤④‎ ‎  C. ②①⑤③④ D. ①②⑤③④‎ ‎  [解析] 排列顺序题。结合文章内容可以判断A项顺序正确。‎ ‎  [答案] A ‎  [长难句解读] But it was Audrey Hepburn's move to America that brought her true fame.是奥黛丽·赫本移居美国给她带来了真正的名声。这是一个强调句型,强调主语Audrey Hepburn's move to America。‎ ‎(第64篇)‎ ‎  The world is not coming to an end on December 21, 2012, the US space agency insisted Monday in a rare campaign to dispel widespread rumors fueled by the Internet and a new Hollywood movie.‎ ‎  The latest big screen offering from Sony Picture, 2012, arrives in theaters on Friday, with a 200milliondollar production about the end of the world supposedly based on myths (虚构的故事) backed by the Mayan calendar. It is claimed that the end of time will come as a Planet X - or Nibiru - heads toward or collides with the Earth.‎ ‎  Some websites accuse NASA of hiding the truth on the planet' s existence, but the US space agency denounced such stories as an "Internet hoax(恶作剧) ". "There is no factual basis for these claims," NASA said in a questionandanswer posting on its website. "If such a collision were real, astronomers would have been tracking it for at least the past decade, and it would be visible by now to the naked eye. Obviously, it does not exist. Credible scientists worldwide know ‎ of no threat associated with 2012," it insisted. "After all, our planet has been getting along just well for more than four billion years," added NASA.‎ ‎  There is another planet, Eris, floating in space. But the small planet similar to Pluto will remain safely fixed in the outer solar system and it can come no closer than four billion miles to the Earth, according to NASA.‎ ‎  Initial theories set the disaster for May 2003, but when nothing happened the date was moved forward to the winter in 2012 to coincide with the end of a cycle of the ancient Mayan calendar.‎ ‎  NASA insisted the Mayan calendar in fact does not end on December 21, 2012, as another period begins immediately afterward.‎ ‎  And even if the planets were to line up as some have forecast, the effect on our planet would be "negligible", NASA said.‎ ‎  "And while comets and asteroids (小行星) have always hit the Earth, big hits are very rare," NASA noted. The last major impact was believed to be 65 million years ago,resulting in the end of dinosaurs.‎ ‎  "We have already determined that there are no threatening asteroids as large as the one that killed the dinosaurs," the space agency said.‎ ‎  [语篇解读] 好莱坞灾难大片《2012》使网络上盛传的"2012年12月21日为世界末日"这一所谓的玛雅预言再次进入人们的视野。为此,美国宇航局重申,2012年12月21日并不是世界末日,传言中的X星球将与地球相撞也并不属实。‎ ‎  1.________played a key role in the spread of the rumors.‎ ‎  A. A new book ‎  B. The Internet and a new Hollywood movie ‎  C. NASA ‎  D. An Indian calendar ‎  [解析] 细节理解题。根据第一段中to dispel widespread rumors fueled by the Intermet and a new Hollywood movie可知因特网和一部好莱坞新片对传播世界末日的谣言起到了推波助澜的作用。‎ ‎  [答案] B ‎  2.The film 2012 is________.‎ ‎  A. about the end of the world ‎  B. based on the scientific prediction ‎  C. well accepted in the science field ‎  D. about a romantic love story ‎  [解析] 细节理解题。根据第二段第一句可知《2012》这部电影是关于世界末日的。根据文章内容可先排除D项;由第二段中based on myths(虚构的故事)可知B项错误;由宇航局的讲话可推出C项错误。‎ ‎  [答案] A ‎  3.We can infer that________.‎ ‎  A. people didn't take the rumor seriously ‎  B. Planet X - or Nibiru does exist ‎  C. astronomers have been tracking Planet X for over ten years ‎  D. the rumor caused a panic among some people ‎  [解析] 推理判断题。根据the US space agency insisted Monday in a rare campaign to dispel widespread rumors可知谣言传播之广以至于美国宇航局不得不出来辟谣,由此可见事态严重,故可判断谣言有可能造成了一些人的恐慌。‎ ‎  [答案] D ‎  4.NASA thinks that Eris________.‎ ‎  A. might pose a threat to the Earth ‎  B. doesn't pose any threat to the Earth ‎  C. is too far away to be visible ‎  D. is similar to our planet, where life might exist ‎  [解析] 细节理解题。根据第四段内容可知Eris行星并没有对我们的地球构成威胁。‎ ‎  [答案] B ‎  5. Which of the following is the best title?‎ ‎  A. New Hollywood movie 2012‎ ‎  B. December 21, 2012, NOT the end of the world ‎  C. End of the ancient Mayan calendar ‎  D. How rumors came into being ‎  [解析] 主旨大意题。本文主要讲美国宇航局驳斥"2012年12月21日是世界末日"的谣言。故B项为最佳标题。‎ ‎  [答案] B ‎  [长难句解读] The latest big screen offering from Sony Picture, 2012, arrives in theaters on Friday, with a 200milliondollar production about the end of the world supposedly based on myths(虚构的故事) backed by the Mayan calendar.‎ ‎  本句为简单句。句子的主语为The latest big screen offering,意为"最新银幕作品"。介词短语"with a..."作状语。过去分词短语supposedly based on myths作定语,修饰the end of the world。过去分词短语backed by the Mayan calendar作定语,修饰myths。‎ ‎  ‎ ‎   (第65篇)‎ ‎  Angus MacLeod was fifty and he had spent his entire life as a shepherd in the remote mountainous region of northwest Scotland. He had never owned a radio or television nor had he travelled further than forty kilometres from his birthplace. His knowledge of the world was gained only from his brief trips to the village to sell his sheep and buy food. However, he had ‎ lately come to rely on the growing number of hillwalkers in the region as sources of international news.‎ ‎  In the spring of 1992, a Spanish student came across Angus high in the mountains. Eager to practise his English he engaged Angus in conversation. The student told Angus of the forthcoming Olympic Games in Barcelona. Excited by the student's colourful description of Spain and the Games, Angus decided to attend the event in person and two months later arrived in Barcelona.‎ ‎  The ticketless Angus stood outside the stadium with his canny brain working overtime,watching a growing number of individuals entering the stadium through a small entrance at the side. He noticed that they all seemed to be carrying objects. Perhaps they were workmen, he thought. He moved closer and watched.‎ ‎  Within minutes a thin young man came along carrying an extremely long pole. He approached the official at the door and said "Pole Vault". The official moved to the side and the man entered. Next came a heavyset man with a spear. Angus wondered how a man could carry such a dangerous weapon in a modern city. The man shouted "Javelin" and was presently admitted. Angus was puzzled. Perhaps they were all connected with security. His puzzlement grew when a huge man appeared with a steel ball in his hand. He shouted "Shot Putt" and walked past the official.‎ ‎  It suddenly dawned on Angus that these people were competitors. He opened his programme and sure enough under the heading "events" he saw the three words the men had said. Angus laughed to himself as a plan began to take shape in his mind. First he needed to buy one or two things.‎ ‎  An hour later he reappeared dressed in a tracksuit with "Scotland" written across the chest.Over his shoulder he carried a roll of barbed wire (铁丝网). Smiling to himself he walked up to the official and as casually as he could, shouted, "Fencing!"‎ ‎  1. Which was Angus' new way of getting information about the outside world?‎ ‎  A. Travelling from his birthplace.‎ ‎  B. Listening to radio and watching TV.‎ ‎  C. Going to the nearby village to sell sheep.‎ ‎  D. Talking to hillwalkers in the region.‎ ‎  [答案] D ‎  2. What are "Javelin", "Pole Vault", "Shot Putt" and "Fencing" in the passage?‎ ‎  A. Olympic events.‎ ‎  B. Names of competitors.‎ ‎  C. Security weapons.‎ ‎  D. Names of countries.‎ ‎  [答案] A ‎  3. From the passage we can learn that________.‎ ‎  A. more and more people went to Scotland to practise their English ‎  B. the Spanish student's description of his country excited Angus ‎  C. Angus had been planning a trip to Barcelona to see the Olympic Games ‎  D. Angus became a member of the Scottish Olympic team ‎  [答案] B ‎  4. From the passage we know that Angus seemed to be________.‎ ‎  A. poorlyinformed B. intelligent ‎  C. strongminded D. athletic ‎  [答案] A ‎  5. What do you expect the official would do in the end?‎ ‎  A. Help him carry the wire.‎ ‎  B. Refuse to let him in.‎ ‎  C. Give him a close inspection.‎ ‎  D. Lead him to the competition.‎ ‎  [答案] B ‎(第66篇)‎ ‎  In the quest for better health, many people turn to doctors, selfhelp books or herbal supplements. But they overlook a powerful weapon that could help them fight illness and depression, speed recovery, slow aging and prolong life: their friends.‎ ‎  Researchers are only now starting to pay attention to the importance of friendship and social networks in overall health. A 10year Australian study found that older people with a large circle of friends were 22 percent less likely to die during the study period than those with fewer friends. A large 2007 study showed an increase of nearly 60 percent in the risk for obesity among people whose friends gained weight. And last year, Harvard researchers reported that strong social ties could promote brain health as we age.‎ ‎  "In general, the role of friendship in our lives isn' t well appreciated," said Rebecca G. Adams, a professor of sociology at the University of North Carolina, Greensboro. "There are a lot of things on families and marriage, but very little on friendship. It baffles me. Friendship has a bigger impact on our psychological wellbeing than family relationships."‎ ‎  Bella DePaulo, a visiting psychology professor at the University of California, Santa Barbara, whose work focuses on single people and friendships, notes that in many studies, friendship has an even greater effect on health than a spouse or a family member.‎ ‎  It isn' t entirely clear why friendship has such a big effect. It may be because people with strong social ties also have better access to health services and care. Beyond that, however, friendship clearly has a profound psychological effect. People with strong friendships are less likely than others to get colds, perhaps because they have lower stress levels.‎ ‎  Last year, researchers studied 34 students at the University of Virginia, taking them to the base of a steep hill and fitting them with a weighted backpack. They were then asked to estimate the steepness of the hill. Some participants stood next to friends during the exercise, while others were alone.‎ ‎  The students who stood with friends gave lower estimates of the steepness of the hill. And the longer the friends had known each other, the less steep the hill appeared.‎ ‎  "People with stronger friendship networks feel like there is someone they can turn to," said Karen A.Roberto, director of the center for gerontology (老年医学) at Virginia Tech." Friendship is an undervalued resource. The consistent message of these studies is that friends make your life better."‎ ‎  [语篇解读] 如何保持身心健康?相信每个人对此都很感兴趣并能给出自己的答案。本文作者向我们介绍了一个有利于身心健康的,很重要又长期被人们忽视的因素--友谊。‎ ‎  1.What' s the main idea of the passage?‎ ‎  A. People should make friends as many as possible.‎ ‎  B. Friendships do great good to human health.‎ ‎  C. People with friends have optimistic attitude.‎ ‎  D. Several studies have been done on friendships.‎ ‎  [解析] 主旨大意题。文章通过几个研究结果说明友谊在促进人类身心健康方面作用巨大。‎ ‎  [答案] B ‎  2.Which of the following is true?‎ ‎  A. People haven't attached importance to friendships.‎ ‎  B. A friend can replace doctors and selfhelp books.‎ ‎  C. People with many friends may have higher stress levels.‎ ‎  D. People who have friends tend to be fat.‎ ‎  [解析] 正误判断题。从文章第三段可知,人们尚未认识到友谊对人们身心健康的重要性。其他选项均与原文信息不符。‎ ‎  [答案] A ‎  3.The author mentioned the study in the last three paragraphs to indicate that ________.‎ ‎  A. students should climb the steep hill with friends ‎  B. it takes a long time to make friends ‎  C. a friend in need is a friend indeed ‎  D. people accompanied by friends remain optimistic in the face of difficulty ‎  [解析] 推理判断题。倒数第二段中"The students who stood with friends gave lower estimates of the steepness of the hill。"表明与朋友在一起的学生估计出来的坡度更小,这意味着他们面对困难能保持相对乐观的心态。‎ ‎  [答案] D ‎  4.In Paragraph 2 the author aims to tell us ________.‎ ‎  A. friendship has a strong physical effect on people's health ‎  B. we cannot be too careful when making friends ‎  C. older people are likely to have fewer friends ‎  D. 60 percent of the people without friends will become fat when they get old ‎  [解析] 段落大意题。根据本段开头(主题句)可知答案。而且主题句后的三个研究结论都与"友谊影响身体健康"有关。‎ ‎  [答案] A ‎  5.According to Karen A. Roberto, ________.‎ ‎  A. the effect of friendships is overvalued ‎  B. studies on friendships make our life better ‎  C. friends are more important than family members ‎  D. friends can help each other when faced with problems ‎  [解析] 推理判断题。由文章最后一段的第一句可推断出,有问题的时候朋友之间可以相互帮助,A、B、C三项不符合原文。‎ ‎  [答案] D ‎  [长难句解读]‎ ‎  1.But they overlook a powerful weapon that could help them fight illness and depression, speed recovery, slow aging and prolong life: their friends.‎ ‎  that引导定语从句,修饰a powerful weapon;该定语从句中有四个并列的谓语动词;their friends和a powerful weapon构成同位关系。‎ ‎  2.Bella DePaulo, a visiting psychology professor at the University of California, Santa Barbara, whose work focuses on single people and friendships, notes that in many studies, friendship has an even greater effect on health than a spouse or a family member.‎ ‎  本句中a visiting psychology professor at the University of California,Santa Barbara是Della DePaulo的同位语;whose work focuses on...是定语从句,修饰Bella DePaulo;notes是本句的谓语动词,后面接宾语从句。‎ ‎  ‎ ‎(第67篇)‎ ‎  "Sesame Street" has been called "the longest street in the world". That is because the television program by that name can now be seen in so many parts of the world. That program became one of American's exports soon after it went on the air in New York in 1969.‎ ‎  In the United States more than six million children watch the program regularly. The viewers include more than half the nation's pre-school children, from every kind of economic(经济的), racial(种族的), and geographical group.‎ ‎  Although some educators object to certain elements in the program, parents praise it highly. Many teachers also consider it a great help, though some teachers find that problems ‎ arise when first graders who have learned from "Sesame Street" are in the same class with children who have not watched the program.‎ ‎  Tests have shown that children from all racial, geographical, and economic backgrounds have benefited from watching it. Those who watch it five times a week learn more than occasional(偶然的) viewers. In the US the program is shown at different hours during the week in order to increase the number of children who can watch it regularly.‎ ‎  The programs all use songs, stories, jokes, and pictures to give children a basic understanding of numbers, letters and human relationships. But there are some differences. For example, the Spanish program, produced in Mexico City, devotes more time to teaching whole words than to teaching separate letters.‎ ‎  Why has "Sesame Street" been so much more successful than other children's shows? Many reasons have been suggested. People mention the educational theories(理论) of its creators, the support by the government and private(私人的) businesses, and the skillful use of a variety of TV tricks. Perhaps an equally important reason is that mothers watch it along with their children. This is partly because famous adult stars often appear on it. But the best reason for the success of the program may be that it makes every child watching feel able to learn. The child finds himself learning, and he wants to learn more.‎ ‎  1. "Sesame Street" is actually _______.‎ ‎  A. a street in the US B. a program for children ‎  C. a program for teachers D. a program for students ‎  2. Children who often watch the program _______.‎ ‎  A. can have problems in school B. will find it a great help ‎  C. will take no interest in their studies D. will be well educated ‎  3. What is special about the program?‎ ‎  A. It offers great fun.‎ ‎  B. It makes children feel able to learn.‎ ‎  C. It is shown at different hours during the week.‎ ‎  D. Children learn and enjoy themselves while watching.‎ ‎  4. Why is "Sesame Street" so popular in the world?‎ ‎  A. Because it is supported by the government and businesses.‎ ‎  B. Because it uses a variety of skillful tricks.‎ ‎  C. Because mothers watch it along with their children.‎ ‎  D. Because it makes every child watching it feel able to learn.‎ ‎  5. The best title for this passage can be _______.‎ ‎  A. TV Programs B. Educating Children ‎  C. Sesame Street D. A Great Success ‎  本文介绍了电视节目Sesame Street所产生的社会影响及它成功的原因。‎ ‎  1. B。细节题。根据文章第2段第1句more than six million children watch the program regularly可推知此题答案为B。‎ ‎  2. B。推断题。根据文章第4段第1句Tests have shown that children...have benefited from watching it 可推知此题答案为B。‎ ‎  3. D。推断题。根据文章第5段第1句The programs all use songs, stories, jokes, and pictures to give children a basic understanding of numbers, letters and human relationships可推知此题答案为B。‎ ‎  4. D。推断题。根据文章最后一段倒数第2句the best reason for the success of the program may be that it makes every child watching feel able to learn 可推知此题答案为D。‎ ‎  5. C。主旨题。本文主要介绍电视节目Sesame Street 所产生的影响及它成功的原因,所以选C。‎ ‎(第68篇)‎ ‎  Ruben lived in a small village. There was no school there and he had to study in a school in the town. It was nearly five kilometres away from his village. His father couldn't buy a bike for him and he had to go to school on foot. He got up early in the morning. Usually he had to run there in order not to miss the first class. He kept running every day and it was helpful to him. He's strong and tall now and he ran faster than any other man in his village. He took part in several sports meetings and won some medals. The young man is proud of and always wants to have a race with others.‎ ‎  One day his mother was ill and his father told him to buy some medicine for headache in the town. The young man got there soon. There were many people in the chemist's shop while he was waiting there. And when his turn came, he couldn't find his money. An old woman hinted(暗示)a young man had stolen it. He saw the thief had already left the shop. He ran towards him quickly. The thief found it and began to run. He was happy and tried to catch up with him.‎ ‎  "Let's have a race and see who will run faster," Ruben called out behind.‎ ‎  Soon after that he caught up with the thief. But he didn't stop and went on running. At a crossing one of his friends asked, "What are you running for, Ruben?"‎ ‎  "I'm running after(追)a thief. "‎ ‎  "Where's the thief, then?"‎ ‎  "He wants to show that he'll run faster than me, but he's fallen behind!"‎ ‎1. Ruben had to run to school because ______________.‎ A. he had no bike B. he hoped to win some medals C. his village was a little far from the school D. he was afraid to be late for class ‎2.______________ , so he won some medals.‎ A. Ruben is good at running B. Ruben went to school on time C. Ruben often took part in the sports meetings.‎ D. Ruben likes to have a race with others ‎3. The phrase "a chemist's shop" in the story means ______________.‎ A. 化工商店 B. 药店 C. 化工厂 D. 制药厂 ‎4. Ruben's money was stolen ______________.‎ A. on his way to the town B. before he went in the shop C. when he was waiting in the shop D. after he had left the shop ‎5. Ruben was happy because ______________.‎ A. he could easily catch the thief B. he had a chance(机会)to have a race C. he knew who had stolen his money D. he would win another medal that day ‎1. D。细节理解题。在所给的几个原因当中, Ruben怕迟到才是最主要的。‎ ‎2. A。细节理解题。Ruben赢得奖牌是因为他跑得快。‎ ‎3. B。猜测词义题。Ruben要买的是零售药品。自然在药店, 因此, B项是正确选项。‎ ‎4. C。细节理解题。‎ ‎5. B。推理判断题。‎ ‎(第69篇)‎ ‎1970 was "World Conservation Year". The United Nations wanted everyone to know that the world was in danger. They hoped that the governments would act quickly in order to conserve nature. Here is one example of the problem. At one time there were 1300 different plants, trees and flowers in Holland, but now only 860 remain. The others have been destroyed by modern man and his technology. We are changing the earth, the air and the water, and everything that grows and lives. We can't live without these things. If we continue like this, we shall destroy ourselves.‎ What will happen in the future? Perhaps it is more important to ask "What must we do now?" The people who will be living in the world of tomorrow are the young of today. A lot of them know that conservation is necessary. Many are hoping to save our world. They plant trees, build bridges across rivers in forests and so on. In a small town in the United States a large group of girls cleaned the banks of eleven kilometres of their river. Young people may hear about conservation through a record called "No, One's Going to Change our World." It was made by Scatles, Cliff Richard and other singers. The money from it will help to conserve wild animals.‎ ‎1. There are few plants, trees and flowers in Holland now because _______.‎ A. there has been a lot of conservation in Holland B. Holland does not need so many plants, trees and flowers C. many plants, trees and flowers don't grow there any more D. some plants, trees and flowers are dangerous ‎2. We shall destroy ourselves if we don't _______.‎ A. improve our technology in planting trees B. hear about the record called "No, One's Going to Change our World"‎ C. try our best to save the world D. change the earth ‎3. "No, One's Going to Change our World." was _______.‎ A. an important book published in 1970‎ B. a record calling on people to conserve nature C. an idea that nobody would accept D. a rule worked out by the United States ‎4. What is the most important thing for us to do to save our world?‎ A. We should clean the banks of our rivers.‎ B. We should know what will happen in the future.‎ C. We should know what we should do and do it now.‎ D. We should plant more trees and flowers.‎ ‎5. What's the main idea of the passage?‎ A. 1970 was "World Conservation Year".‎ B. The United Nations wanted everybody to know that the world is in danger.‎ C. Conservation is necessary.‎ D. It is the young people who are helping to save our world.‎ ‎1.C。推理判断题。根据第一自然段内容可知。‎ ‎2.C。细节理解题。根据第一自然段最后两句We can't live without these things. If we continue like this, we shall destroy ourselves.可知。‎ ‎3.B。推理判断题。根据文章最后一自然段倒数第三句Young people may hear about conservation through a record called "No, One's Going to Change our World. "可知。‎ ‎4.C。推理判断题。根据文章第二自然段可推出此答案。‎ ‎5.C。主旨大意题。文章最后一自然段中点明了主题:A lot of them know that conservation is necessary.‎ ‎(第70篇)‎ ‎  Five years ago, David Smith wore an expensive suit to work every day. "I was a clothes addict," he jokes. "I used to carry a fresh suit to work with me so I could change if my clothes got wrinkled." Today David wears casual clothes-khaki pants and sports shirt-to the office. He ‎ hardly ever wears a necktie. "I'm working harder than ever," David says, "and I need to feel comfortable."‎ ‎  More and more companies are allowing their office workers to wear casual clothes to work in the United States. The change from formal to casual office wear has been gradual. In the early 1990s, many companies allowed their employees to wear casual clothes on Friday(but only on Friday). This became known as "dress-down Friday" of "casual Friday". "What started out as an extra one-day-a-week benefit for employees has really become an everyday thing." said business consultant Maisly Jones.‎ ‎  Why have so many companies started allowing their employees to wear casual clothes? One reason is that it's easier for a company to attract new employees if it has a casual dress code. "A lot of young people don't want to dress up for work," says the owner of a software company, "so it's hard to hire people if you have a conservative dress code." Another reason is that people seem happier and more productive when they are wearing comfortable clothes. In a study conducted by Levi Strauss and Company, 85 percent of employers said that they believe that casual dress improves employee morale. Only 4 percent of employers said that casual dress has a negative impact on productivity. Supporters of casual office wear also argue that a casual dress code helps them save money. "Suits are expensive, if you have to wear one every day," one person said. "For the same amount of money, you can buy a lot more casual clothes."‎ ‎1. David Smith refers to himself as having been "a clothes addict," because _______.‎ A. he often wore khaki pants and a sports shirt B. he couldn't stand a clean appearance C. he wanted his clothes to look neat all the time D. he didn't want to spend much money on clothes ‎2. David Smith wears casual clothes now, because _______.‎ A. they make him feel at ease when working B. he cannot afford to buy expensive clothes C. he looks handsome in casual clothes D. he no longer works for any company ‎3. According to this passage, which of the following statements is false?‎ A. Many employees don't like a conservative dress code.‎ B. Comfortable clothes make employees more productive.‎ C. A casual clothes code is welcomed by young employees.‎ D. All the employers in the U. S. are for casual office wear.‎ ‎4. According to this passage, which of the following statements is true?‎ A. Company workers started to dress down about twenty years ago.‎ B. Dress-down has become an everyday phenomenon since the early 1990s.‎ C. "Dress-down Friday" was first given as a favor from employers.‎ D. Many workers want to wear casual clothes to impress people.‎ ‎5. In this passage, the following advantages of casual office wear are mentioned in the passage except _______.‎ A. saving employees' money B. making employees more attractive ‎  C. improving employees' motivation D. making employees happier ‎  从20世纪90年代起,美国许多公司开始允许员工星期五可以不统一着装上班。现在越来越多的公司许可领员工穿便装上班了,这对调节他们的心情,调动其工作热情有一定的作用。‎ ‎  1. C。语义理解题。大卫自嘲以前是服装爱好者,衣服一起皱,他就得换上备穿的服装。‎ ‎  2. A。语义理解题。从第 1 段最后一句话 "I'm working harder than ever," David says, "and I need to feel comfortable." 可找到答案。‎ ‎  3. D。细节判断题。根据最后一段中的 Only 4 percent of employers said that casual dress has a negative impact on productivity 可知还有一些雇主不赞同穿便装上班。‎ ‎  4. C。细节判断题。从第 2 段最后 1 句 "What started out as an extra one-day-a-week benefit for employees has really become an everyday thing." 可以做出正确判断。‎ ‎  5. B。推断题。通读最后一段可知选项A(为雇员省钱)、B(提高雇员的工作热情)、D(使雇主高兴)在原文中均被提到,只有A未被提到。‎ ‎(第71篇)‎ ‎  Mankind is still evolving(进化), according to a recent study by a team of scientists led by a Yale University evolutionary biologist. But research suggests that the natural selection process is no longer driven by survival, perhaps because of differences in women's fertility(能生育性).‎ ‎  The team examined 2,238 women in Framingham, Massachusetts, the US and studied the medical histories of some 14,000 residents since 1948.‎ ‎  They found that strong,slightly fat women tend to have more children. And if this trend were to continue for the next 10 generations, by 2409 the average woman would be 2 cm shorter,1 kg heavier, and have a healthier heart.‎ ‎  That rate of evolution is pretty similar to what we see in plants and animals. Humans_don'_t_seem_to_be_any_exception,_the team said.‎ ‎  Other recent genetic research backs up that belief. One study by John Hawks, an anthropologist at the University of Wisconsin at Madison, found that some 1,800 human gene variations (变异) are common in recent generations because of evolution. Among the changes are mutations (突变) that allow people to digest milk or resist malaria(疟疾) and others that govern brain development.‎ ‎  Since the Industrial Revolution, modern humans have grown taller and stronger. It' s easy to assume that evolution is making humans fitter. But according to anthropologist Peter McAllister, the contemporary male has evolved physically into "the sorriest example of male ever on the planet". An average Neanderthal woman, McAllister notes, could have beat Arnold Schwarzenegger in an armwrestling (腕力) match. And early Australian Aborigines could easily defeat Usain Bolt in a 100-m dash.‎ ‎  However, Douglas Ewbank, of the University of Pennsylvania, says that cultural factors tend to have a much greater impact than natural selection on future generations, so people tend to ignore evolution' s effects.‎ ‎  And, despite evidence indicating evolution still works, Steve Jones, an evolutionary biologist at University College London, thinks that the impact of evolution on humans is not the most important. "What makes humans what we are is in our minds, in our society, and not in our evolution," said Jones.‎ ‎  [语篇解读] 科学家们发现人类还在不断地进化,但是他们认为自然选择对人类进化的影响已经没那么大了。‎ ‎  1.What does the underlined sentence "Humans don' t seem to be any exception" imply?‎ ‎  A. Plants and animals are evolving faster than humans.‎ ‎  B. Evolution doesn't work in humans any longer.‎ ‎  C. The rate of evolution in plants and animals also applies to humans.‎ ‎  D. Humans are unique in their evolution.‎ ‎  [解析] 推理判断题。根据第四段内容可知,人类的进化速度和动植物的很相似,人类并不例外,由此可判断动植物的进化速度也适用于人类。‎ ‎  [答案] C ‎  2.Why is the recent genetic research quoted in the fifth paragraph?‎ ‎  A. To support the belief that mankind is still evolving.‎ ‎  B. To show human gene variations are common in recent generations.‎ ‎  C. To prove modern humans have grown fitter.‎ ‎  D. To suggest that the natural selection process is no longer driven by survival.‎ ‎  [解析] 细节理解题。根据第五段中的Other recent genetic research backs up that belief可知最近的其他基因研究也支持"人类还在进化"这一观点,和文章开头的主题句照应。‎ ‎  [答案] A ‎  3.According to the passage, we can infer that Arnold Schwarzenegger ________.‎ ‎  A. used to be a famous actor ‎  B. is an American governor ‎  C. is very weak ‎  D. is very strong ‎  [解析] 推理判断题。第六段中举施瓦辛格的例子是为了证明在人类越来越健壮的进化中,男性成为最令人遗憾的例子。由对施瓦辛格的了解和文段内容可判断阿诺德·施瓦辛格很强壮,力气很大。‎ ‎  [答案] D ‎  4.According to Douglas Ewbank, which of the following factors plays a greater part in the future generations?‎ ‎  A. Culture. B. Evolution.‎ ‎  C. Natural selection. D. Genetic mutations.‎ ‎  [解析] 细节理解题。根据倒数第二段内容可知,在未来的后代中,起更大作用的是文化因素。‎ ‎  [答案] A ‎  5.What is the main idea of the passage?‎ ‎  A. Mankind is still evolving.‎ ‎  B. Women will get fatter.‎ ‎  C. The history of human evolution.‎ ‎  D. Future women tend to have more children.‎ ‎  [解析] 主旨大意题。本文第一句就是主题句,后面都是围绕着"人类还在不断地进化"这一观点进行论述的。‎ ‎  [答案] A ‎  [长难句解读] 1.Among the changes are mutations(突变)that allow people to digest milk or resist malaria(疟疾) and others that govern brain development.‎ ‎  本句主语很长,为了句子结构的平衡进行了倒装。主句的主语是mutations and others,定语从句that allow people to digest milk or resist malaria修饰mutations,定语从句that govern brain development修饰others。‎ ‎  2.And, despite evidence indicating evolution still works, Steve Jones, an evolutionary biologist at University College London, thinks that the impact of evolution on humans is not the most important.‎ ‎  本句的主语是Steve Jones, an evolutionary biologist at University College London是Steve Jones的同位语,that引导一个宾语从句。despite evidence为介词短语作状语,后面的非谓语动词短语作evidence的后置定语。‎ ‎(第72篇)‎ ‎  THURSDAY, July 9 ( HealthDay News) - A new study that found that a lowercalorie diet slowed the aging process in monkeys could be the best proof yet that restricted diets might do the same for humans.‎ ‎  "The big question in aging research is, 'Will caloric restriction in species closely related to humans slow aging?'" said Richard Weindruch, senior author of a paper appearing in the July 10 issue of Science.This is the first clear demonstration that, in a primate species, we' re inducing a slowdown of the aging process - showing increased survival, resistance to disease, less brain atrophy and less muscle loss.‎ ‎  "This predicts humans would respond similarly," added Weindruch, professor of medicine at the University of Wisconsin and an investigator at the Veterans Hospital in Madison. Another expert noted that, despite some highly publicized studies in certain species, the link between restricted eating and longer lifespan has been far from proven.‎ ‎  "The idea that dietary restriction extends lifespan in all species is not true. Many strains of rats and mice do not respond. In some strains, it' s actually deleterious," explained Felipe Sierra, director of the biology of aging program at the US National Institute on Aging (NIA),‎ ‎ which supported the new study. "The fact that it doesn't work in some mice but it does seem to work in monkeys is surprising and it gives us hope."‎ ‎  But there' s a larger question: how to change humans' increasingly lax eating habits. "This_finding_doesn't_give_me_hope_that_humans_are_going_to_go_into_dietary_restriction," added Marianne Grant, a registered dietitian at Texas A&M Health Science Center Coastal Bend Health Education Center in Corpus Christi.‎ ‎  [语篇解读] 这是一篇新闻报道,研究人员通过对动物的研究证明低热量的饮食可能会延长人的寿命。‎ ‎  1.The underlined word "restricted" means"________".‎ ‎  A. limited B. satisfied ‎  C. reserved D. fascinated ‎  [解析] 词义猜测题。根据第一段可知,低热量的饮食可以延长猴子的寿命,这一试验证明有节制的饮食可能同样会延长人的寿命。‎ ‎  [答案] A ‎  2.Which of the following statements about Weindruch is true?‎ ‎  A. This experiment indicates restricted diets could extend humans' lifespan.‎ ‎  B. Adequate sleep is another way to reduce the speed of aging process.‎ ‎  C. The experiment was a failure in some species such as mice.‎ ‎  D. The experiment which had been tested on humans got a great success.‎ ‎  [解析] 细节理解题。根据第三段第一句话可知答案为A。‎ ‎  [答案] A ‎  3.According to Felipe Sierra' s idea, we know that________.‎ ‎  A. the restricted diets experiment really had no distinct effect on rats and mice ‎  B. humans are finding new ways to slowdown the aging process of all the species ‎  C. the experiment is unnecessary to be carried on because of the response of the mice ‎  D. the experiment confirmed that restricted diets extend lifespan in all species ‎  [解析] 细节理解题。从第四段的第二句"Many strains of rats and mice do not respond."可知答案。‎ ‎  [答案] A ‎  4.It can be inferred from the underlined sentence that ________.‎ ‎  A. the result of the experiment shows that humans are not pigging out nowadays ‎  B. nowadays humans are turning unhealthy diets into lowercalorie diets ‎  C. how to change humans' careless eating habits is a long way to go ‎  D. the experiment changed humans' increasingly careless eating habits ‎  [解析] 细节理解题。由画线句子可知人们目前并没有进入到控制饮食的状态中,也就是说要改变人们漫不经心的饮食习惯还有很长的路要走。‎ ‎  [答案] C ‎  5.The author most likely wrote this text to________.‎ ‎  A. compare the two experiments on rats and monkeys ‎  B. illustrate the data from the research ‎  C. call on people to pay attention to their daily diets ‎  D. introduce a lively debate to amuse the reader ‎  [解析] 写作目的题。作者的目的是通过介绍试验"低热量的饮食可能会延长人的寿命",呼吁人们注意控制饮食。‎ ‎  [答案] C ‎  [长难句解读] Another expert noted that, despite some highly publicized studies in certain species, the link between restricted eating and longer lifespan has been far from proven.‎ ‎  本句的主句为Another expert noted that,其中that引导的是一个宾语从句。从句用despite表示让步关系,后面的far from proven意为"远没有得到证明"。‎ ‎(第73篇)‎ ‎  ‎ ‎  The treadle pump is based on a design developed in the 1970s by Norwegian engineer Gunnar Barnes. It can be made locally.‎ ‎  The treadle pump is easy to build from bamboo or other wood and two metal cylinders (圆筒) with pistons (活塞). The pistons go up and down as a person stands on lever devices - treadles - and uses a natural walking motion.‎ ‎  How many hours a day the pump needs to be operated depends on the season and how much water is needed for crops. It could be two hours a day. It could be seven hours a day.‎ ‎  Small children sometimes stand with their parents on the treadles. Everyone in the family can take turns operating the pump.‎ ‎  The Acumen Fund is a nonprofit group that invests in business projects to fight poverty. It studied the effects of treadle pumps in the Indian state of Uttar Pradesh. Uttar Pradesh has three treadle pump manufacturers and more than seventythree thousand pumps.‎ ‎  Acumen reported that families using them ate more vegetables, because they were able to grow more to eat and to sell. Many of these families also drank more milk, because they bought a cow with their added earnings. Men with treadle pumps often no longer have to leave the farm to seek extra work in cities.‎ ‎  The pump can also improve education. Farmers often use their extra earnings to buy books for their children or to pay for schooling.‎ ‎  A farmer in Zambia said he hoped to have enough money in three years to buy a dieselpowered pump (柴油泵). Then he could grow more crops over a larger area.‎ ‎  But the world economic crisis has had an effect on some farmers. IDE executive director Zenia Tata says some who were able to buy diesel pumps now do not have enough money to buy fuel. So they are using their treadle pumps again.‎ ‎  [语篇解读] 脚踏泵因其简单、易行、实用之特点,正在世界的广大贫困地区被推广,以帮助农民脱贫致富,且已取得了显著的效果。‎ ‎  1.The treadle pump ________.‎ ‎  A. is invented by an American engineer ‎  B. is mainly made of metal ‎  C. consists of two cylinders with pistons ‎  D. consists of two cylinders with an engine ‎  [解析] 事实细节题。根据第二段第一句可知答案为C。‎ ‎  [答案] C ‎  2.What does the author want to tell us about the treadle pump in the second paragraph?‎ ‎  A. Its history.‎ ‎  B. Its working principle.‎ ‎  C. Its structure.‎ ‎  D. Its usage and effects.‎ ‎  [解析] 作者意图题。A和D显然不对,C有一定干扰性,但其不能概括第二句,介绍工作原理时,一般要同时介绍其结构,故选B。‎ ‎  [答案] B ‎  3.It can be inferred from the passage that ________.‎ ‎  A. it is easy to learn how to operate the treadle pump ‎  B. it is difficult to build a treadle pump ‎  C. the treadle pump is used more in winter than in summer ‎  D. operating the treadle pump can keep fit ‎  [解析] 推理判断题。B和C与文中内容相悖,可排除。易错选D,但文中没有依据,无法推知。根据第四段,小孩也可帮忙,家庭中每个人都可参与,可推知A为答案。‎ ‎  [答案] A ‎  4.Each treadle pump manufacturer in the Indian state of Uttar Pradesh has produced on average about ________.‎ ‎  A. 18, 000 pumps B. 24, 000 pumps ‎  C. 37, 000 pumps D. 73, 000 pumps ‎  [解析] 数据推算题。根据第五段第三句可知,该州共三家生产商,共有七万三千多架脚踏泵,平均每家生产两万四千架左右,故选B。‎ ‎  [答案] B ‎  5.Why does the farmer in Zambia hope to buy a dieselpowered pump?‎ ‎  A. Because he wants to grow crops on a larger scale.‎ ‎  B. Because he hates to leave the farm to seek fortune in cities.‎ ‎  C. Because a dieselpowered pump is cheaper than before.‎ ‎  D. Because oil is much cheaper than before.‎ ‎  [解析] 事实细节题。根据倒数第二段第二句可知答案。‎ ‎  [答案] A ‎  [长难句解读] IDE executive director Zenia Tata says some who were able to buy diesel pumps now do not have enough money to buy fuel.‎ ‎  该句为双重复合句。主句主语为IDE executive director Zenia Tata,谓语为says,后面为省略了that的宾语从句。该宾语从句的主干部分为some do not have enoueh money to buy fuel,who were able to buy diesel pumps为定语从句,修饰主语some。‎ ‎(第74篇)‎ ‎  It was near dusk, and Kim Cooper and her husband, Steve, were trucking through northern Kentucky transportation auto parts from Louisville to Detroit for a goods company. "Steve, wake up!" she shouted. "There's a truck on fire!"‎ ‎  Inside the burning truck, Ronnie Sanders, 38, was fighting for his life. He'd been running a heavy load of tractors and forklifts from Georgia to Indianapolis when a van in front of him stopped suddenly in traffic on the icy road. As Ronnie bore down, he could see children in the backseat. The truck's bulk would probably protect him from the worst of the impact, but the force of 23 tons would likely crush(压碎) everyone inside the van.‎ ‎  "I figured instead of killing other people, I'd just put the truck in the ditch. "At the bottom, rocks cut a fuel tank, which caught fire. A tree branch destroyed the windshield and knocked Ronnie unconscious. He came to life a couple of minutes later to find the cabin flames and his legs on fire.‎ ‎  Steve dashed to Ronnie, who was hanging headfirst from the passenger door. Ronnie had used his pocketknife to cut himself free from the driver'sside seat belt only to get his boot trapped in another one. Steve climbed into the burning cab to free him.‎ ‎  He tried three times to pull Ronnie out before finally freeing him. But Ronnie's legs were still burning, so Steve laid him on the ground, ripped off his own shirt, and beat the flames with it. He'd managed to drag him about 20 yards when one of the truck's 150gallon fuel tanks exploded.‎ ‎  Both Steve and Ronnie paid a price for risking their lives for strangers. Ronnie spent two months in the hospital and received skin grafts on both of his legs. Steve suffered smoke breathing and minor burns, and shrapnel from the fuel tank explosion broke a tooth.‎ ‎  In February, the Coopers received a Hero of the Highway award from the Open Road Foundation for rescuing an injured driver. Steve insists Ronnie is the real hero: "If he hadn't gone into the ditch, he would have hit that van. It was his decision to drive off the road."‎ ‎  "I feel pretty good about it," says Ronnie. "A lot of people could have been hurt."‎ ‎  解读:本文为记叙文,主要讲述了the Coopers途中遇到Ronnie开的大卡车着火了;Ronnie为了不造成更大的伤害,不顾自身的安危,将卡车开进水沟里,而Steve则竭尽全力救助Ronnie。‎ ‎  1. According to the passage, Ronnie's putting the truck in the ditch contributes to________.‎ ‎  A. his concern for his own safety ‎  B. his poor driving skill ‎  C. the icy road ‎  D. his concern for others' lives ‎  [解析] 细节推断题。由文章的第三段第一句话"I figured instead of killing other people,I'd just put the truck in the ditch."可推断出。‎ ‎  [答案] D ‎  2. What probably made Ronnie come to life after driving the truck in the ditch?‎ ‎  A. The noise from the highway.‎ ‎  B. The call from the Coopers.‎ ‎  C. The pain from the burning legs.‎ ‎  D. The heat of the fire.‎ ‎  [解析] 细节推断题。根据文章的第三段最后一句话He came to life a couple of minutes later to find the cab in flames and his legs on fire.利用排除法。‎ ‎  [答案] C ‎  3. How did Steve put out the flames in Ronnie's legs?‎ ‎  A. He beat the flames with Ronnie's shirt.‎ ‎  B. He beat the flames with his own shirt.‎ ‎  C. He put out the fire with a tank of water.‎ ‎  D. He put out the fire with a bag of sand.‎ ‎  [解析] 细节理解题。根据文章第五段第二句话,Steve撕下自己的衬衫,用它来扑灭(Ronnie双腿上的)火焰。‎ ‎  [答案] B ‎  4. What does the underlined word "shrapnel" in Paragraph 6 mean?‎ ‎  A. A branch of a tree.‎ ‎  B. A small piece of metal.‎ ‎  C. A wave of the explosion.‎ ‎  D. The heat of the explosion.‎ ‎  [解析] 词义猜测题。根据下文shrapnel from the fuel tank explosion broke a tooth可知是"油箱爆炸产生的金属片打碎了一颗牙"。‎ ‎  [答案] B ‎  5. Which of the following can be the best title of this passage?‎ ‎  A. Heroes: The Coopers ‎  B. Heroes: Roadside Flames ‎  C. Accidents on the Icy Road ‎  D. Accidents in the Ditch ‎  [解析] 确定文章标题。综观全文,A,C,D三项不够全面;而B项即表明事故,也表明了事故中竭力营救他人的英雄。‎ ‎  [答案] B ‎(第75篇)‎ ‎  Astronauts aboard the space station Celebrated a space first on Wednesday by drinking water that had been recycled from their urine and sweat and water that condenses from exhaled air. They said "cheers" ,clicked drinking bags and toasted NASA workers on the ground who were sipping their own version of recycled drinking water.‎ ‎  The recycling system is needed for astronaut stations on the moon and Mars. It also will save NASA money because it won' t have to ship up as much water to the station by space shuttle or cargo rockets.Besides, it' s important as the space station is about to expand from three people living on board to six.‎ ‎  The recycling system had been brought up to the space station last November by space shuttle Endeavour, but it couldn' t be used until samples were tested back on Earth. So when it came time to actually drink up, NASA made a big deal of it. The threeman crew stood, holding their drinks and congratulated engineers in two NASA centers that worked on the system.‎ ‎  "This is something that had been the stuff of science fiction," Barratt said before taking a small mouthful. "The taste is worth trying."‎ ‎  NASA deputy space shuttle manager LeRoy Cain called it "a huge milestone".‎ ‎  The new system takes the combined urine of the crew from the toilet, moves it to a big tank, where the water is boiled off, and the vapor is collected. The rest of the urine is thrown away. The water vapor is mixed with water from air condensation, and then it goes through filters. When six crew members are aboard it can make about six gallons from urine in about six hours.‎ ‎  "Some people may find the idea of drinking recycled urine distasteful, but it is also done on Earth with a lot longer time between urine and the tap"said Marybeth Edeen, the space station's national lab manager.‎ ‎  The technology NASA developed for this system has already been used for quick water purification after the 2004 Asian tsunami, Edeen said.‎ ‎  "We are happy to have this water work through the system - we' re happy to have it work through our systems,"Barratt said.‎ ‎  [语篇解读] 本文讲的是国际空间站宇航员试饮尿液循环水。‎ ‎  1.According to the text, the recycling system is important because________.‎ ‎  A. it makes traveling to the moon for the common person possible ‎  B. with it NASA won' t need to ship any water up to the space station ‎  C. it can help to meet the need for more water after the crew are expanded ‎  D. it protects the environment in space by reducing the amount of waste ‎  [解析] 细节理解题。根据第二段中的"it's important as the space station is about to expand from three people living on board to six"可知C项正确。‎ ‎  [答案] C ‎  2.What is the RIGHT time order of the following events?‎ ‎  a. The samples of the recycled water were tested on Earth.‎ ‎  b. Astronauts celebrated the space first of making water from urine.‎ ‎  c. The recycling system was brought up to the space station.‎ ‎  d. The technology for the system was used to quickly purify water.‎ ‎  A. a,d,c,b B. a,c,b,d ‎  C. d,a,c,b D. d,c,a,b ‎  [解析] 细节理解题。根据倒数第二段的"The technology NASA developed for this system has already been used for quick water purification after the 2004 Asian tsunami"可知d发生最早。根据第三段的" The recycling system had been brought up to the space station last November... couldn' t be used until samples were tested back on Earth"可知c发生在a的前面,因此D项正确。‎ ‎  [答案] D ‎  3.From Para. 6 we can know________.‎ ‎  A. the theory behind the recycling system ‎  B. the effect of the recycling system ‎  C. the process of the recycling system ‎  D. an opinion on the recycling system ‎  [解析] 段落概括题。该段主要讲述的是使用尿液循环系统将尿制成水的过程。‎ ‎  [答案] C ‎  4.What did Edeen say about recycled urine and the recycling system?‎ ‎  A. The taste of recycled urine is not as good as common water.‎ ‎  B. The recycling system has made a science fiction story come true.‎ ‎  C. The idea of drinking recycled urine makes astronauts feel unpleasant.‎ ‎  D. It takes a longer time on Earth to make water from urine than in space.‎ ‎  [解析] 细节理解题。根据第七段中的"with a lot onger time between urine and the tap"可知D项正确。‎ ‎  [答案] D ‎  5.What is the best title for the passage?‎ ‎  A. The research of NASA has made great progress ‎  B. New technology is used in the space station ‎  C. Drink up: space station recycles urine to water ‎  D. Good news: water recycled from urine tastes good ‎  [解析] 标题概括题。本文报道了国际空间站的宇航员试饮尿液循环水,"Drink up"生动反映饮用时的情景,"space station recycles urine to watert"点明了制造技术。故C项最佳。‎ ‎  [答案] C ‎  [长难句解读] Astronauts aboard the space station celebrated a space first on Wednesday by drinking water that had been recycled from their urine and sweat and water that condenses from exhaled air.‎ ‎  主句是:Astronauts celebrated a space first on Wednesday by drinking water.aboard the space station和that had been recycled from their urine and sweat and water充当定语分别修饰名词Astronauts和water。‎ ‎(第76篇)‎ ‎  LONDON (Reuters) - Quiet please - Britain' s Queen Elizabeth is preparing to have her swans counted.‎ ‎  Buckingham Palace has announced that the annual Swan Upping, a tradition dating back to the 12th century which involves a census of the swan population on the River Thames, will be conducted by the Queen' s official Swan Marker from July 20 24.‎ ‎  "With the assistance of the Queen' s Swan Warden, Professor Christopher Perrins of the University of Oxford, the swans and young cygnets (小天鹅) are also assessed for any signs of injury or disease," Buckingham Palace said in announcing the count.‎ ‎  The process involves the Swan Marker, David Barber, rowing up the Thames for five days with the Swan Warden in traditional skiffs while wearing special scarlet uniforms and counting, weighing and measuring swans and cygnets.‎ ‎  It may seem eccentric, but it is very important to the Queen, According to custom, Britain' s sovereign owns all unmarked, mute swans in open water, but the Queen now exercises the right only on stretches of the Thames and its nearby tributaries.‎ ‎  In medieval times, the Swan Marker would not only travel up the river counting the swans, but would catch as many as possible as they were soughtafter for banquets and feasts.‎ ‎  This year, the Swan Marker and the Swan Warden are particularly keen to discover how much damage is being caused to swans and cygnets by attacks from dogs and from discarded fishing tackle (渔具).‎ ‎  It is also an important year because Queen Elizabeth has decided to join her team of Swan Uppers for part of the census. She will follow them up the river and visit a local school project on the whole subject of swans, cygnets and the Thames.‎ ‎  "Education and conservation are essential to the role of Swan Upping and the involvement of school children is always a rewarding experience," Buckingham Palace said.‎ ‎  [语篇解读] 泰晤士河上每年都要举行一个重要的活动:数天鹅。英国皇室的这一传统已经延续了好几个世纪。‎ ‎  1. In medieval times,________.‎ ‎  A. swans were better protected than now ‎  B. a lot of swans were killed by dogs ‎  C. swans were a delicious dish on royal banquets ‎  D. common people could catch the swans ‎  [解析] 细节理解题。从文章倒数第四段可知,在中世纪天鹅被捉住,然后成为皇家宴会上的美味。‎ ‎  [答案] C ‎  2.We can infer from the passage that the process of counting the swans ________.‎ ‎  A. remains almost unchanged in the past years ‎  B. involves a lot of royal members ‎  C. sometimes lasts longer than before ‎  D. is always guarded by special soldiers ‎  [解析] 推理判断题。从文章第四段对数天鹅过程的描述以及参与清点的官员的穿着打扮可知,其过程几乎没有变化,因为这是一项传统活动。‎ ‎  [答案] A ‎  3.This year, the Swan Marker and the Swan Warden mainly want to find out________.‎ ‎  A. the exact number of swans and cygnets ‎  B. how a local school project is going on ‎  C. how much damage the swans and cygnets suffer ‎  D. how education and conservation are carried out along the Thames ‎  [解析] 细节理解题。从文章倒数第三段内容可知答案为C。‎ ‎  [答案] C ‎  4.Which of the following is the main idea of the passage?‎ ‎  A. Britain' s Queen is concerned about swans.‎ ‎  B. Britain' s Queen orders a count of swans.‎ ‎  C. An old tradition in Buckingham Palace.‎ ‎  D. Queen Elizabeth will count swans herself.‎ ‎  [解析] 主旨大意题。文章第一段是本文的主题段,英国女王下令清点泰晤士河上的天鹅的数量。‎ ‎  [答案] B ‎  5.The underlined word "tributaries" can be best replaced by ________.‎ ‎  A. districts B. banks ‎  C. trees D. branches ‎  [解析] 词义理解题。根据语境和常识可知,此处是指在泰晤士河及其支流上生活的天鹅。故答案为D。‎ ‎  [答案] D ‎  [长难句解读] In medieval times, the Swan Marker would not only travel up the river counting the swans, but would catch as many as possible as they were soughtafter for banquets ‎ and feasts.在中世纪,负责清点天鹅的人不仅沿河清点天鹅的数量,还会尽可能多地捕捉一些,因为天鹅当时是宴会桌上的美餐。‎ ‎  本句是一个并列句,not only...but(also)...引导并列句,后面的as引导原因状语从句。‎ ‎(第77篇)‎ ‎  Happiness is contagious,_researchers reported on Thursday.‎ ‎  People with the most social connections - spouses, friends, neighbors and relatives - were also the happiest, the data showed. "Each additional happy person makes you happier," Christakis said.‎ ‎  "Imagine that I am connected to you and you are connected to others and others are connected to still others. It is this fabric of humanity, like an American patch quilt."‎ ‎  Each person sits on a differentcolored patch. "Imagine that these patches are happy and unhappy patches. Your happiness depends on what is going on in the patch around you," Christakis said.‎ ‎  "It is not just happy people connecting with happy people, which they do. Above and beyond, there is this contagious process going on."‎ ‎  And happiness is more contagious than unhappiness, they discovered.‎ ‎  "If a social contact is happy, it increases the likelihood that you are happy by 15 percent," Fowler said. "A friend of a friend, or the friend of a spouse or a sibling (兄弟姐妹), if they are happy, increases your chances by 10 percent," he added.‎ ‎  A happy thirddegree friend - the friend of a friend of a friend - increases a person's chances of being happy by 6 percent.‎ ‎  "But every extra unhappy friend increases the likelihood that you'll be unhappy by 7 percent," Fowler said.‎ ‎  The finding is interesting and it is useful, too, Fowler said.‎ ‎  "Among other benefits, happiness has been shown to have an important effect on reduced mortality (死亡率), pain reduction, and improved cardiac (心脏的) function. So better understanding of how happiness spreads can help us learn how to promote a healthier society," he said.‎ ‎  The study also fits in with other data suggested in 1984 that having $ 5,000 extra increased a person's chances of becoming happier by about 2 percent.‎ ‎  "A happy friend is worth about $ 20,000," Christakis said.‎ ‎  [语篇解读] 美国科学家的一项最新研究成果表明,快乐感可以互相传递。这项研究成果为我们构建更为健康、和谐的社会提供了依据和方法。‎ ‎  1.According to the research, your happiness ________.‎ ‎  A. has nothing to do with your workmates or schoolmates ‎  B. has something to do with anyone who has a close relationship with you ‎  C. depends on those who are in favour of you or are against you ‎  D. has little to do with what social connections you have ‎  [解析] 推理判断题。根据第二段、第三段内容可知,快乐感会不断传递和扩散,你的配偶、朋友、邻居、亲戚等的快乐指数会直接影响你的快乐程度,故选B。‎ ‎  [答案] B ‎  2.The underlined word "contagious" in the first paragraph means ________.‎ ‎  A. infectious B. beautiful ‎  C. effective D. prior ‎  [解析] 词义猜测题。第一句是全文的主题句。由文章第二、三、四段可知,快乐感可以在人们之间传播,故contagious的意思是"有感染力的,有传染性的"。‎ ‎  [答案] A ‎  3.It can be inferred that________.‎ ‎  A. happiness spreads as fast and widely as unhappiness ‎  B. unhappiness spreads faster and more widely than happiness ‎  C. happiness spreads faster and more widely than unhappiness ‎  D. the spread of unhappiness has not been studied by researchers ‎  [解析] 推理判断题。根据第六段"And happiness is more contagious than unhappiness,they discovered."一句可推知选C。‎ ‎  [答案] C ‎  4.Which of the following will increase your chances of becoming happier most?‎ ‎  A. Being in a party with a happy atmosphere.‎ ‎  B. A happy experience of your brother or your parents.‎ ‎  C. A happy trip to a foreign country of your friends.‎ ‎  D. Happiness of your friend' s friend.‎ ‎  [解析] 事实细节题。根据第七、第八段讲述的别人的快乐感对你的影响几率可知,排在最前的是"If a social contact is happy",故选A。‎ ‎  [答案] A ‎  5.What does Christakis mean by saying "A happy friend is worth about $ 20,000" ?‎ ‎  A. A happy friend can help you bring in a wealth of $ 20,000.‎ ‎  B. A happy friend is a wealth which is worth about $ 20,000.‎ ‎  C. A happy friend increases your chances of being happy by 2%.‎ ‎  D. A happy friend increases your chances of being happy by 8 %.‎ ‎  [解析] 句意理解题。结合上一段中的"that having $5,000 extra increased a person's chances of becoming happier by about 2 percent"可知,一个快乐的朋友带给你的快乐相当于大约$20,000带给你的快乐,也就是$5,000的四倍,即一个快乐的朋友给你带来快乐的几率为8%,故选D。‎ ‎  [答案] D ‎  [长难句解读] The study also fits in with other data suggested in 1984 that having $ 5,000 extra increased a person's chances of becoming happier by about 2 percent.‎ ‎  该句为主从复合句,主句为The study also fits in with other data,suggested in 1984为过去分词作定语修饰data,后面that引导的是同位语从句,说明data的内容。该从句的主语为having $5,000 extra,a person's chances为宾语,of becoming happier为定语修饰chances,by about 2 percent为程度状语。‎ ‎(第78篇)‎ ‎  The Parthenon in Athens is a building with a long and complex history. Built nearly 2,500 years ago as a temple celebrating the Greek goddess Athena, it was for thousands of years the church of the Virgin Mary of the Athenians, then a mosque (清真寺), and finally a ruin. The building was changed and the sculptures much damaged over the centuries. By 1800 only about half of the original sculptural decoration remained.‎ ‎  Between 1801 and 1805, Lord Elgin, the British ambassador to the Ottoman Empire, which controlled Athens, acting with the full knowledge and permission of the Ottoman authorities,removed about half of the remaining sculptures from the fallen ruins and from the building itself.Lord Elgin loved Greek history and transported the sculptures back to Britain. The arrival of the sculptures in London had a huge effect on the European public, greatly increasing interest in ancient Greek culture and influencing contemporary artistic trends. These sculptures were acquired from Lord Elgin by the British Museum in 1816 and since then they have all been on display to the public, free of charge.‎ ‎  Since the early 1980s, however, the Greek government has argued for the permanent removal to Athens of all the Parthenon sculptures in the British Museum. They have also challenged the British Museum Board of Trustees' legal title to the sculptures.‎ ‎  The British Museum, however, insists that it exists to tell the story of cultural achievement throughout the world, from the dawn of human history over two million years ago until the present day. The museum considers itself an important resource for the world: the breadth and depth of its collection allows the world public to reexamine cultural identities and explore the complex network of interconnected world cultures.‎ ‎  It also says that, within the context of this unparalleled collection, the Parthenon sculptures are an important representation of ancient Athenian civilization. Each year millions of visitors admire the artistry of the sculptures and gain insights on how ancient Greece influenced - and was influenced by - the other civilizations that it encountered.‎ ‎  1. For most of its history people went to the Parthenon to________.‎ ‎  A. admire the goddess Athena ‎  B. pray to their god ‎  C. search for sculptures ‎  D. learn about its complex history ‎  [答案] B ‎  2. The underlined "it" (in Paragraph 4) refers to "________".‎ ‎  A. the British Museum ‎  B. the Greek government ‎  C. the Parthenon ‎  D. the British Museum Board of Trustees ‎  [答案] A ‎  3. What can we infer from the passage?‎ ‎  A. The sculptures introduced ancient Greek culture to the west.‎ ‎  B. Ancient Greek culture has greatly influenced world culture.‎ ‎  C. The British Museum has made much money from the Parthenon sculptures.‎ ‎  D. The British Museum is the place most capable of preserving these sculptures.‎ ‎  [答案] B ‎  4. What can we learn about Lord Elgin from the passage?‎ ‎  A. He is greatly admired in Greece.‎ ‎  B. He worked for the Ottoman Empire.‎ ‎  C. He saved the Parthenon sculptures from being destroyed.‎ ‎  D. He had a deep interest in Greek culture.‎ ‎  [答案] D ‎  5. The author's main intention in writing this passage is to tell________.‎ ‎  A. the history of the Parthenon and its sculptures ‎  B. what people can see in the British Museum ‎  C. why the British Museum refuses to return the sculptures ‎  D. the influence of Greece on British culture ‎  [答案] C ‎(第79篇)‎ ‎  If you travel to a new exhibit at the San Francisco Conservatory of Flowers, you will have chances to see some meat-eating plants. Take bladderworts, a kind of such plant, for example. They appear so small and grow in a quiet pond. But these are the fastest-known killers of the plant kingdom, able to capture a small insect in 1/50 of a second using a trap door!‎ ‎  Once the trap door closes on the victim, the enzymes(酶) similar to those in the human stomach slowly digest the insect. When dinner is over, the plant opens the trap door and is ready to trap again.‎ ‎  Meat-eating plants grow mostly in wet areas with soil that doesn' t offer much food value. In such conditions, these amazing plants have developed insect traps to get their nutritional needs over thousands of years. North America has more such plants than any other continent.‎ ‎  Generally speaking, the traps may have attractive appearance to fool the eye, like pitcher plants, which get their name because they look like beautiful pitchers (a container like a bottle) full of nectar(花蜜).‎ ‎  The Asian pitcher plant, for example, has bright colors and an attractive half-closed lid. Curious insects are tempted to come close and take a sip, and then slide down the slippery (光滑的)slope to their deaths.‎ ‎  Hair-like growths along the pitcher walls ensure that nothing cab escape, and the digestive enzymes can get to work. A tiny insect can be digested in a few hours, but a fly takes a couple of days.‎ ‎  Some of these pitchers are large enough to hold two gallons. Meateating plants only eat people in science fiction movies, but sometimes a bird or other small animals will discover that a pitcher plant isn' t a good place to get a drink.‎ ‎  [语篇解读] 本文是一篇说明文,介绍了一些特别的植物--靠捕食动物获得生长所需养分的"食肉植物"。‎ ‎  1.From the first paragraph, we can know that bladderworts can ________.‎ ‎  A. capture 50 small insects in a second ‎  B. capture an insect in the shortest time ‎  C. be found floating on a quiet lake ‎  D. digest an insect in 1/50 of a second ‎  [解析] 细节理解题。第一段指出,bladderworts能在1/50秒的时间内捕获一只小昆虫,是植物中已知的捕获动物速度最快的,因此选B。‎ ‎  [答案] B ‎  2.If the trap door of a meateating plant is closed, the plant ________.‎ ‎  A. is fooling insects into taking a sip ‎  B. is producing nectar ‎  C. is tempting insects to come close ‎  D. is enjoying a dinner ‎  [解析] 细节判断题。第二段说,一旦"陷阱门"将猎物关起来,类似于人胃里的那些消化酶就开始慢慢地消化它了,由此判断D正确。‎ ‎  [答案] D ‎  3.Meateating plants can grow in wet and poor soil because they ________.‎ ‎  A. can get nutrition from animals ‎  B. don' t need much food value ‎  C. can make the most of such conditions ‎  D. have developed digestive enzymes ‎  [解析] 推理判断题。这些植物生长在潮湿贫瘠的土壤中,为获得生长所需的养分,在数千年的进化中,它们演化出了捕获昆虫的"陷阱",由此推出它们主要从动物中获取养分,而不是从土壤中获取养分。‎ ‎  [答案] A ‎  4.Which of the following is the picture of the Asian pitcher plant?‎ ‎  ‎ ‎  [解析] 推理判断题。根据第四、五、六段可知亚洲捕虫草主要有这些特征:状如水瓶,颜色鲜亮,有半掩的盖子,综合此三项可知,只有A项最合适。‎ ‎  [答案] A ‎  5.According to the passage, which of the following is true?‎ ‎  A. South Asia has the most meateating plants.‎ ‎  B. Some of the meateating plants can even eat a human.‎ ‎  C. Hairlike growths in the plants help trap insects.‎ ‎  D. The traps of these plants have a dull color.‎ ‎  [解析] 细节理解题。依据文章第三段最后一句可知,北美洲的食肉植物最多,故A错。从最后一段中的第二句可知,食肉植物是不吃人的,所以B项错。从文章倒数第二段可知,这些须状的东西可以防止小昆虫逃跑,故C项正确。从文章第四段可知,食肉植物为了吸引昆虫一般有漂亮的外形,故D项错。‎ ‎  [答案] C ‎(第80篇)‎ ‎  After years of research and testing, the hybrid car was developed and put on the market. It' s an interesting and exciting new improvement in today' s world as we look for better ways to protect the quality of the air we breathe and conserve our natural resources.‎ ‎  The quality of our air is affected by many different things. But one of the largest sources of air pollution is the burning of fossil fuels such as coal and gasoline which is used to power a car' s engine. The EPA has set national standards to help control the level of harmful pollutants sent off into the air, and the automobile industry has acted by producing a hybrid car that uses less gas and therefore causes less pollution.‎ ‎  ‎ ‎  A hybrid car is a combination of a regular car that runs on gasoline and an electric car that is battery powered. Some people tend to think that since the hybrid car is partially electric, you have to plug it in to charge it. But that's not how it works. The 144volt battery pack is actually recharged through the energy that is produced when the car's brakes are used. This is referred to as "regenerative braking" , because it generates electricity.‎ ‎  Although the hybrid car still runs on gasoline most of the time, this helps it use less gas than a regular car. When the driver stops at a traffic light, the engine automatically shuts off to save fuel. Then, as soon as the driver puts the car in gear and touches the gas pedal, the engine starts back up.‎ ‎  Have you ever ridden in a car with someone who ran out of gas? That probably wouldn' t happen if you were riding in a hybrid car. It flashes a waming on its computer screen that says, "I am low on gas". When it completely runs out, the warning reads,"YOU ARE NOW OUT OF GAS!" Then the electric power supply kicks in to let the driver travel a few more miles to a gas station.‎ ‎  [语篇解读] 本文主要介绍了一种新型汽车hybrid cars的优点和生产这种汽车的原因。‎ ‎  1.Which of the following is true according to the passage?‎ ‎  A. The hybrid car will be put on the market.‎ ‎  B. Hybrid cars run faster than regular cars.‎ ‎  C. When the hybrid car stops at a traffic light, the driver will get a warning.‎ ‎  D. The burning of fossil fuels is one of the largest sources of air pollution.‎ ‎  [解析] 细节判断题。根据第二段的"But one of the largest sources of air pollution is the burning of fossil fuels such as coal and gasoline which is used to power a car's engine."可知答案。‎ ‎  [答案] D ‎  2.According to the passage, the word "hybrid" refers to ________.‎ ‎  A. a hightech discovery ‎  B. energy saving ‎  C. a new invention ‎  D. a combination of two things ‎  [解析] 词义猜测题。由文章的第三段的第一句"A hybrid car is a combination of a regular car that runs on gasoline and an electric car that is battery powered."可知答案。‎ ‎  [答案] D ‎  3."Regenerative breaking" is an important process of the hybrid car because ________.‎ ‎  A. it allows the car to come to a quick stop ‎  B. it gives the driver a smoother ride ‎  C. it produces energy to charge the battery ‎  D. it can control the speed of the car ‎  [解析] 细节理解题。由文章的第三段的"The 144volt battery pack is...because it generates electricity."可知答案。‎ ‎  [答案] C ‎  4.Which of the following is right about the EPA?‎ ‎  A. They designed hybrid cars.‎ ‎  B. They outlawed (宣布......为不合法) the burning of fossil fuels.‎ ‎  C. They set important guidelines that help control pollution.‎ ‎  D. Their main purpose is to protect endangered plants and animals.‎ ‎  [解析] 细节理解题。由文章的第二段的第三句可知答案。‎ ‎  [答案] C ‎  5.Which of the following is the reason for developing hybrid cars?‎ ‎  A. They use less gas than regular cars.‎ ‎  B. They' re safer to drive than most cars.‎ ‎  C. They're more modern than other cars.‎ ‎  D. They cost less than regular cars.‎ ‎  [解析] 细节理解题。由第四段的第一句"Although the hybrid car still runs on gasoline most of the time,this helps it use less gas than a regular car."可知答案。‎ ‎  [答案] A ‎(第81篇)‎ ‎  Bananas are one of the world' s most important food crops. They are also one of the most valuable exports. Bananas do not grow from seeds. Instead, they grow from existing plants. Bananas are threatened by disease because all the plants on a farm are copies of each other. They all share the same genetic weaknesses. For example, the Cavendish banana is most popular in North American and European markets. However,some kinds of fungus organisms easily infect the Cavendish. Black Sigatoka disease affects the leaves of Cavendish banana plants. The disease is controlled on large farms by putting chemicals on the plant' s leaves. Farmers put anti-fungal chemicals on their crops up to once a week.‎ ‎  Another fungal disease is more serious. Panama disease attacks the roots of the banana plant. There is no chemical treatment for this disease. Infected plants must be destroyed. Panama disease has affected crops in Southeast Asia, Australia and South Africa. There is concern that it may spread to bananas grown in the Americas. This could threaten an important export product for Central and South America.‎ ‎  The International Network for the Improvement of Banana and Plantain supports research on bananas. The group has headquarters in France and other offices in the major banana-growing areas of the world. The group says that more research must be done to develop improved kinds of bananas. The group says that fungal diseases mainly affect only one kind of banana. In fact, there are five hundred different kinds of bananas. Food and Agriculture Organization of the United Nations has said that the Cavendish banana represents only 10% of world production.‎ ‎  The U.N. agency says farmers should grow different kinds of bananas. This protects against diseases that affect only one kind. Experts warn that disease may cause the Cavendish banana to disappear. This happened earlier to another popular banana because of its genetic weakness against disease.‎ ‎  [语篇解读] 香蕉由于遗传性的抗病弱点,在不久的将来有的品种可能会消失。文章列举了两种威胁Cavendish香蕉的真菌病,国际组织建议农民种植更多种类的香蕉。‎ ‎  1.What does this passage mainly tell us?‎ ‎  A. Bananas are the world' s most important food crops.‎ ‎  B. The risk to a popular banana shows need to grow other kinds.‎ ‎  C. There are five hundred different kinds of bananas.‎ ‎  D. How to grow bananas in different countries.‎ ‎  [解析] 主旨大意题。根据全文内容和结构判断,文章主要介绍威胁Cavendish香蕉的真菌类疾病,进而显示出种植其他种类香蕉的必要性。故答案为B。‎ ‎  [答案] B ‎  2.Bananas are threatened by disease because ________.‎ ‎  A. they grow from seeds ‎  B. they are one of the most valuable exports ‎  C. the only way to prevent it is to put chemicals on their leaves ‎  D. they have genetic weaknesses against disease ‎  [解析] 细节理解题。根据第一段内容可知,香蕉易受病害威胁是因为其遗传上的抗病弱点。后文也有叙述。‎ ‎  [答案] D ‎  3.Panama disease________.‎ ‎  A. doesn't belong to fungal disease ‎  B. affects the leaves of banana plants ‎  C. destroys bananas more seriously than Black Sigatoka disease ‎  D. has spread to bananas all over the world ‎  [解析] 细节判断题。根据第二段前两句可知A项错误;根据第二段第二句可知这种疾病威胁香蕉作物的根部,B项错误;根据第二段后三句可知,这种香蕉疾病已影响到东南亚、澳大利亚和南非,有可能传播到美洲,D项错误。由文章前两段分别对两种真菌病的描述可知,C项正确。‎ ‎  [答案] C ‎  4.We can infer from Paragraph 3 that ________.‎ ‎  A. the center of the group is in the US ‎  B. the Cavendish banana covers only a small part of the yield of bananas ‎  C. the key to solving the disease is to research all kinds of bananas ‎  D. each fungal disease affects five hundred different kinds of bananas ‎  [解析] 推理判断题。根据第三段中"The group has headquarters in France"可知其中心位于法国,A项错误;根据第三段第三句可知,应对这种疾病的办法是培育改良品种,而不是调查所有的品种,C项错误;根据第三段第四句可知D项错误。由第三段最后一句可判断B项正确。‎ ‎  [答案] B ‎  5.According to the passage, which information is right?‎ ‎  A. The Cavendish banana can mainly be imported from North America and Europe.‎ ‎  B. Panama disease affects the leaves of Cavendish banana plants.‎ ‎  C. The Cavendish banana won't die out in the future.‎ ‎  D. The Cavendish banana makes up only one tenth of world production.‎ ‎  [解析] 细节认定题。根据第一段中"the Cavendish banana is most popular in North American and European markets"可知这种香蕉在北美和欧洲市场最受欢迎,而不是从这两个地区进口,A项错误;根据第二段第二句可知B项错误;根据文章最后一段第三句可知C项错误;根据第三段最后一句可以判断D项正确。‎ ‎  [答案] D ‎  [长难句解读] This happened earlier to another popular banana because of its genetic weakness against disease.由于天生的抗病弱点,另一种受欢迎的香蕉早些时候灭绝了。because of引导原因状语。注意this指代的是上一句所说的情况。‎ ‎(第82篇)‎ ‎  "Can I see my baby?" the happy new mother asked. When the bundle was in her arms and she moved the fold of cloth to look upon his tiny face, she gasped. The doctor turned quickly and looked out of the tall hospital window. The baby had been born without ears.‎ ‎  Time proved that the baby' s hearing was perfect. It was only his appearance that was imperfect. When he rushed home from school one day and threw himself into his mother's arms, she sighed, knowing that his life was to be misfortunate. He cried out the tragedy, "A boy, a big boy... called me a freak (怪人)."‎ ‎  He grew up, handsome. A favourite with his fellow students, he might have been class president, but for that. He developed a gift, a talent for literature and music. "But you might communicate with other young people," his mother blamed him, but felt a kindness in her heart.‎ ‎  Two years went by. One day, his father said to the son, "You' re going to the hospital, son. Mother and I have someone who will donate the ears you need. But it' s a secret." The operation was a brilliant success, and a new person emerged.‎ ‎  Later he married and entered the diplomatic service. One day, he asked his father, "Who gave me the ears? Who gave me so much? I could never do enough for him or her." "I do not believe you could," said the father, "but the agreement was that you are not to know... not yet."‎ ‎  The years kept their secret, but the day did come. He stood with his father over his mother' s casket. Slowly, tenderly, the father stretched forth a hand and raised the thick, reddish brown hair to reveal the mother had no outer ears.‎ ‎  "Mother said she was glad she never let her hair be cut," his father whispered gently, "and nobody ever thought Mother less beautiful, did they?"‎ ‎  [语篇解读] 本文讲述了一个感人的故事。儿子天生没有耳朵,母亲将自己的耳朵捐献给了儿子。母亲去世后,父亲才将这个秘密告诉儿子。‎ ‎  1.Why did Mother gasp when she saw her newborn baby?‎ ‎  A. Because her son had a tiny face.‎ ‎  B. Because she saw her son crying.‎ ‎  C. Because her son was born imperfect.‎ ‎  D. Because her son was in her arms.‎ ‎  [解析] 细节理解题。由第一段最后一句话"The baby had been born without ears."可知答案。‎ ‎  [答案] C ‎  2.Which word can describe Mother' s feeling when the son threw himself into her arms?‎ ‎  A. Nervous.       B. Sympathetic.‎ ‎  C. Proud. D. Angry.‎ ‎  [解析] 细节理解题。由第二段的"When he rushed home from school one day and threw himself into his mother's arms,she sighed,knowing that his life was to be misfortunate."可知答案。‎ ‎  [答案] B ‎  3.Who gave the son the ears?‎ ‎  A. A doctor. B. His father.‎ ‎  C. His mother. D. A stranger.‎ ‎  [解析] 细节理解题。从文章最后可知是母亲献出了自己的耳朵。‎ ‎  [答案] C ‎  4.The underlined word "reveal" in the last but one paragraph means "________".‎ ‎  A. see B. show ‎  C. find D. search ‎  [解析] 词义推测题。父亲拨开母亲的头发,揭示了母亲将自己的耳朵捐给儿子的真相,因此,此处reveal是"显示"的意思。‎ ‎  [答案] B ‎  5.The best title for the passage would be ________.‎ ‎  A. Mother's hair ‎  B. An unforgettable memory ‎  C. Who gave me the ears?‎ ‎  D. Who is my best respectable person?‎ ‎  [解析] 标题归纳题。是谁给了我耳朵?这是贯穿文章的线索。故选C。‎ ‎  [答案] C ‎  [长难句解读] When he rushed home from school one day and threw himself into his mother's arms,she sighed,knowing that his life was to be misfortunate. When he...arms是一个时间状语从句,主句是she sighed,knowing是现在分词作状语,表示原因,后面又跟了一个宾语从句,不定式短语to be misfortunate作表语,表示他的人生注定不幸。‎ ‎(第83篇)‎ ‎  When times get tough, we all look for ways to cut back. When we're hungry, we eat at home instead of going out. We take buses instead of taxis. And we wear our old designer jeans ‎ just a few months longer. With college expenses at all-time highs, high school students are eager to do anything to cut the cost of a university education.‎ ‎  One cost-cutting proposal is to allow college students to get a bachelor' s degree in three years instead of four. Educational institutions have been actively exploring ways to make the learning process more efficient. But there's a question: Would the quality of undergraduate(本科生)education suffer? Few US universities have formally approved a "three-year degree" model.‎ ‎  I doubt that mainstream North American colleges will carry out a three-year curriculum(课程) any time soon. For one thing, most universities already allow highly qualified students to graduate early by testing out of certain classes and obtaining a number of college credits(学分). In addition, at famous universities, the committee who determine which courses are required and which courses are electives are unlikely to suddenly "throw out" one quarter of the required credits. Professors will resist "diluting(稀释)" the quality of the education they offer.‎ ‎  In my opinion, a quality four-year education is always superior to a quality three-year education. A college education requires sufficient time for a student to become skilled in their major and do coursework in fields outside their major. It is not a good idea to water down education, any more than it' s not a good idea to water down medicine. If we want to help students find their way through university, we should help them understand early on what knowledge and skills they need to have upon graduation. We should allow students to test out of as many courses as possible. We should give them a chance to earn money as interns(实习生)in meaningful part-time jobs that relate to their university studies, such as the five-year co-op program at Northeastern University.‎ ‎  [语篇解读] 本文为议论文。作者认为把大学四年的学制缩短为三年不是一个好主意,保证足够的时间才能保证大学教育的质量。‎ ‎  1.Which of the following can be the best title?‎ ‎  A. It' s time to shorten the learning process ‎  B. Best learning takes place over time ‎  C. University education should be watered down ‎  D. College education calls for reform ‎  [解析] 主旨大意题。作者认为把大学四年的学制缩短为三年不是一个好主意,学好大学功课是需要付出时间的,故B项最佳。‎ ‎  [答案] B ‎  2.We can learn from the passage that ________.‎ ‎  A. most American universities are against the "three-year degree" model ‎  B. many famous US universities are considering adopting the "three-year degree" model ‎  C. professors are willing to accept the "three-year degree" model ‎  D. The "three-year degree" model can make college learning more efficient ‎  [解析] 细节理解题。根据第二段中的Few US universities have formally approved a "three-year degree" model可知几乎没有几所美国大学认可"三年本科学位"模式。‎ ‎  [答案] A ‎  3.In most US universities,________.‎ ‎  A. college students are offered the co-op program ‎  B. electives' credits make up one quarter of the required credits ‎  C. all students are required to finish four-year education before graduation ‎  D. some excellent students can graduate ahead of time ‎  [解析] 细节理解题。根据第三段中的most universities already allow highly qualified students to graduate early可知特别优秀的大学生可以提前毕业。‎ ‎  [答案] D ‎  4.We can infer that________.‎ ‎  A. the author is a college professor ‎  B. the author thinks the cost of a university education is too high for people to afford ‎  C. the author considers the university education quality very important ‎  D. the author pays special attention to the all-round development of college students ‎  [解析] 推理判断题。在第四段作者给出了自己的观点,从"A college education requires sufficient time for a student to become skilled in their major...""It is not a good idea to water down education"等处可以看出作者非常看重大学教育质量。‎ ‎  [答案] C ‎  5.The first paragraph serves as a(n)________.‎ ‎  A. explanation B. definition ‎  C. introduction D. comment ‎  [解析] 篇章结构题。本题要求学生推断文章的结构。根据第一段的内容可知第一段是用来引入话题的,故C项最佳。‎ ‎  [答案] C ‎(第84篇)‎ ‎  I once had my Chinese MBA students brainstorming on "two-hour business plans". I separated them into six groups and gave them an example: a restaurant chain. The more original their idea, the better, I said. Finally, five of the six groups presented plans for restaurant chains. The sixth proposed a catering(餐饮) service. Though I admitted the time limit had been difficult, I expressed my disappointment.‎ ‎  My students were middle managers, financial analysts and financiers from state-owned enterprises and global companies. They were not without talent or opinions, but they had been shaped by an educational system that rarely stressed or rewarded critical thinking or inventiveness. The scene I just described came in different forms during my two years' teaching ‎ at the school. Papers were often copied from the Web and the Harvard Business Review. Case study debates were written up and just memorized. Students frequently said that copying is a superior business strategy, better than inventing and creating.‎ ‎  In China,every product you can imagine has been made and sold. But so few well-developed marketing and management minds have been raised that it will be a long time before most people in the world can name a Chinese brand.‎ ‎  With this problem in mind, partnerships with institutions like Yale and MIT have been established. And then there's the "thousand talent scheme": this new government program is intended to improve technological modernization by attracting top foreign trained scientists to the mainland with big money. But there are worries about China's research environment. It's hardly known for producing independent thinking and openness, and even big salary offers may not be attractive enough to overcome this.‎ ‎  At last, for China, becoming a major world creator is not just about setting up partnerships with top Western universities. Nor is it about gathering a group of well-educated people and telling them to think creatively. It's about establishing a rich learning environment for young minds. It's not that simple.‎ ‎  1. Why does the author feel disappointed at his students?‎ ‎  A. Because there is one group presenting a catering service.‎ ‎  B. Because the six groups made projects for restaurant chains.‎ ‎  C. Because all the students copied a case for the difficult topic.‎ ‎  D. Because the students' ideas were lacking in creativeness.‎ ‎  [解析] 考查推理判断能力。根据第一段整体的信息作出判断和推理。‎ ‎  [答案] D ‎  2. Which of the following scenes is NOT considered as lack of creation?‎ ‎  A. Papers were often downloaded from the Internet.‎ ‎  B. Students often said that copying is a preferable business strategy.‎ ‎  C. Students combine knowledge and critical thoughts to solve a problem.‎ ‎  D. Case study debates were written up as well as recited.‎ ‎  [解析] 考查判断推理能力。第二段整体判断,特别是最后一句。‎ ‎  [答案] C ‎  3. The underlined word "scheme" in the forth paragraph means________.‎ ‎  A. timetable B. theme ‎  C. project D. policy ‎  [解析] 考查猜词能力。第四段的第2句"this new government program"中this指代前面的"thousand talent scheme",而scheme和program为同义词,C中project为program同义词。‎ ‎  [答案] C ‎  4. We can infer from the passage that________.‎ ‎  A. China can make and sell any product all over the world ‎  B. high pay may not solve the problem of China's research environment ‎  C. cooperation with institutions has been set up to make a Chinese brand ‎  D. the new government program are aimed at encouraging imagination ‎  [解析] 考查推理判断能力。文中没有直接表达出来,根据最后一句得出,"even big salary offers may not be attractive enough to overcome this."中this指的是China's learning environment存在的问题即problem,也是指It's hardly known for producing independent thinking and openness这个问题。‎ ‎  [答案] B ‎  5. Which is the best title of the passage?‎ ‎  A. Look for a New Way of Learning.‎ ‎  B. Reward Creative Thinking.‎ ‎  C. How to Become a Creator.‎ ‎  D. Establish a technical Environment.‎ ‎  [解析] 考查主旨判断题。从整体和最后一段判断,整篇文章是讲关于教育出现的问题--有了丰富的学习环境,及新的学习方法,才能有创造性的研究氛围。‎ ‎  [答案] A ‎(第85篇)‎ ‎  Throughout the world, boys and girls prefer to play with different types of toys. Boys typically like to play with cars and trucks, while girls typically choose to play with dolls. Why is this? A traditional sociological explanation is that boys and girls are socialized and encouraged to play with different types of toys by their parents, peers, and the "society". Growing scientific evidence suggests, however, that boys' and girls' toy preferences may have a biological origin.‎ ‎  In 2002, Gerianne M. Alexander of Texas A&M University and Melissa Hines of City University in London surprised the scientific world by showing that monkeys showed the same sextypical toy preferences as humans. In the study, Alexander and Hines gave two masculine toys (a ball and a police car), two feminine toys (a soft doll and a cooking pot), and two neutral toys (a picture book and a stuffed dog) to 44 male and 44 female monkeys. They then assessed the monkeys' preference for each toy by measuring how much time they spent with each. Their data showed that male monkeys showed significantly greater interest in the masculine toys, and the female monkeys showed significantly greater interest in the feminine toys. The two sexes did not differ in their preference for the neutral toys.‎ ‎  If children' s toy preferences were largely formed by gender socialization, as traditional sociologists' claim, in which their parents give "genderappropriate" toys to boys and girls, how can these male and female monkeys have the same preferences as boys and girls?They were never socialized by humans, and they had never seen these toys before in their lives.‎ ‎  [语篇解读] 本文是科普阅读。男孩和女孩在玩玩具方面具有明显的性别差异。这种差异是后天习得的,还是与生俱来的呢?科学家对此进行了研究。‎ ‎  1.Traditional sociologists believe boys' and girls' toy preferences ________.‎ ‎  A. are passed down from their parents ‎  B. have a biological origin ‎  C. have nothing to do with gender socialization ‎  D. are largely formed in later life ‎  [解析] 细节理解题。从文章第一段的第四句和最后一段可知,传统的社会学家认为男孩、女孩对玩具的偏好差异是后天形成的。‎ ‎  [答案] D ‎  2.The study by Alexander and Hines shows that monkeys________.‎ ‎  A. also play toys as humans do ‎  B. also have a sextypical toy preference ‎  C. have no toy preferences ‎  D. like to play different toys at different time ‎  [解析] 细节理解题。文章第二段的第一句说明,猴子在玩玩具时,也有性别差异。‎ ‎  [答案] B ‎  3.Alexander and Hines carried out the study to ________.‎ ‎  A. find out why boys and girls prefer different toys ‎  B. test the intelligence of monkeys ‎  C. test whether monkeys like to play toys ‎  D. find more evidence for traditional sociology ‎  [解析] 推理判断题。从文章第一段最后一句和最后一段可知,用猴子做研究的目的是证明男孩和女孩在玩玩具时所表现出的偏好是与生俱来的,而不是后天习得的。‎ ‎  [答案] A ‎  4.According to the study, if given a stuffed dog, ________.‎ ‎  A. only the male monkeys showed interest ‎  B. the female monkeys showed more interest ‎  C. the male and female monkeys showed the same interest ‎  D. neither the male nor the female monkeys showed any interest ‎  [解析] 细节理解题。从文章第二段最后一句可知,如果给一种中性的玩具,猴子表现出的兴趣没有性别差异。‎ ‎  [答案] C ‎  5.Masculine toys are mainly intended for________.‎ ‎  A. monkeys  B. adult ‎  C. boys D. girls ‎  [解析] 细节理解题。从文章第二段可知,这种玩具是跟女孩玩的玩具相对的,因此是指男孩玩具。‎ ‎  [答案] C ‎(第86篇)‎ ‎  When Steve Maxwell graduated from college, he had an engineering degree and a hightech job - but he couldn't balance his checkbook. "I took one finance class in college but dropped it to go on a ski trip," says the 45yearold father of three, who lives in Windsor, Colorado. "I actually had to go to my bank and ask them to teach me how to read my statement."‎ ‎  One of the biggest obstacles to making money is not understanding it: Thousands of us avoid investing because we just don't get it. But to make money, you must be financially literate. "It bothered me that I didn't understand this stuff," says Steve, "so I read books and magazines about money management and investing, and I asked every financial whiz (能手) I knew to explain things to me."‎ ‎  He and his wife started applying the lessons: They made a point to live below their means. They never bought on impulse, always negotiated better deals (on their cars,cable bills, furniture) and stayed in their home long after they went for an expensive vacation. They also put 20 percent of their annual salary into investments.‎ ‎  Within ten years, they were millionaires, and people were coming to Steve for advice. "Someone would say, 'I need to refinance my house - what should I do? 'A lot of times,I wouldn't know the answer, but I'd go to find it and learn something in the process," he says.‎ ‎  In 2003, Steve quit his job to become part owner of a company that holds personal finance seminars (研讨会) for employees of corporations like WalMart. He also started going to real estate investment seminars, and it's paid off: He now owns $ 30 million worth of investment properties, including apartment complexes, a shopping mall and a quarry.‎ ‎  "I was an engineer who never thought this life was possible, but all it truly takes is a little selfeducation," says Steve. "You can do anything once you understand the basics."‎ ‎  [语篇解读] 经过不断的学习,史蒂夫从不懂理财的工程师发展成为善于理财的百万富翁。‎ ‎  1. When Steve Maxwell graduated from college, he________.‎ ‎  A. couldn't balance his work and life ‎  B. made up his mind to become a millionaire ‎  C. didn't know how to manage his money ‎  D. often went to banks to borrow money ‎  [解析] 细节理解题。根据第一段的he couldn't balance his checkbook和I actually had to go to my bank and ask them to teach me how to read my statement可知,Steve Maxwell 刚毕业后不懂得如何理财。‎ ‎  [答案] C ‎  2. The underlined part "live below their means" (in Paragraph 3) probably means "________".‎ ‎  A. take effective measures ‎  B. live a miserable life ‎  C. spend more money than they had ‎  D. spend less money than they had ‎  [解析] 词义猜测题。根据下文"他们从来不冲动地买东西,买东西时尽量获得低价,减少外出度假计划"等线索可以推断出,史蒂夫一家在尽量减少开支。‎ ‎  [答案] D ‎  3. Since Steve Maxwell became a millionaire, he________.‎ ‎  A. has been unwilling to help others ‎  B. hasn't stopped learning from practice ‎  C. has been willing to follow others' advice ‎  D. has stopped to invest in houses ‎  [解析] 推理判断题。根据第四段的A lot of times,I wouldn't know the answer,but I'd go find it and learn something in the process可以看出,史蒂夫总是在教别人理财的过程中学到一些东西。‎ ‎  [答案] B ‎  4. What would be the best title for the passage?‎ ‎  A. A selfmade engineer ‎  B. How to balance your checkbook ‎  C. Don't avoid investing ‎  D. Educate yourself to become a millionaire ‎  [解析] 标题归纳题。史蒂夫从开始的根本不懂理财到最后成为百万富翁,靠的是不断的学习--向别人讨教,从书本上学习,D项点出了他成功的秘诀。文章最后一段的selfeducation是关键词。‎ ‎  [答案] D ‎  5. It can be inferred from the passage that________.‎ ‎  A. selfeducation is very important to one's life ‎  B. everyone should learn how to invest in their life ‎  C. Steve Maxwell was quite interested in finance classes in college ‎  D. Steve became rich because he saved every penny he had earned ‎  [解析] 细节推断题。根据文章的最后一段可知,自主学习对每一个人的人生有重大的影响,故可知选A。文中提到了Steve投资赚钱韵事情,但是无法推断出每个人都要学习如何投资;根据文章首段可知Steve为了滑雪旅行放弃了财经课,可判断C不对;D项与原文不符合。‎ ‎(第87篇)‎ ‎  Lisa was running late. Lisa, 25, had a lot to do at work, plus visitors on the way: her parents were coming in for Thanksgiving from her hometown. But as she hurried down the subway stairs, she started to feel uncomfortably warm. By the time she got to the platform,Lisa ‎ felt weak and tired - maybe it hadn't been a good idea to give blood the night before, she thought. She rested herself against a post close to the tracks.‎ ‎  Several yards away, Frank ,43, and his girlfriend, Jennifer,found a spot close to where the front of the train would stop. They were deep in discussion about a house they were thinking of buying.‎ ‎  But when he heard the scream, followed by someone yelling,"Oh, my God, she fell in!"Frank didn't hesitate. He jumped down to the tracks and ran some 40 feet toward the body lying on the rails. "No ! Not you ! "his girlfriend screamed after him.‎ ‎  She was right to be alarmed. By the time Frank reached Lisa,he could feel the tracks shaking and see the light coming. The train was about 20 seconds from the station.‎ ‎  It was hard to lift her. She was just out. But he managed to raise her the four feet to the platform so that bystanders could hold her by the arms and drag her away from the edge. That was where Lisa briefly regained consciousness,felt herself being pulled along the ground,and saw someone else holding her purse.‎ ‎  Lisa thought she'd been robbed. A woman held her hand and a man gave his shirt to help stop the blood pouring from her head. And she tried to talk but she couldn't,and that was when she realized how much pain she was in.‎ ‎  Police and fire officials soon arrived, and Frank told the story to an officer. Jennifer said her boyfriend was calm on their 40minute train ride downtown-just as he had been seconds after the rescue,which made her think about her reaction at the time. "I saw the train coming and I was thinking he was going to die," she explained.‎ ‎  [语篇解读] 本文讲述了Frank在地铁站,冒着被列车轧死的危险救助了因身体虚弱而失去知觉的Lisa的经过。‎ ‎  1.What was the most probable cause for Lisa's weakness?‎ ‎  A. She had run a long way.‎ ‎  B. She felt hot in the subway.‎ ‎  C. She had done a lot of work.‎ ‎  D. She had donated blood the night before.‎ ‎  [解析] 事实细节题。从第一段"...it hadn't been a good idea to give blood..."可知,Lisa怀疑自己虚弱的原因是因为献血了。‎ ‎  [答案] D ‎  2.Why did Jennifer try to stop her boyfriend?‎ ‎  A. Because they would miss their train.‎ ‎  B. Because he didn't see the train coming.‎ ‎  C. Because she was sure Lisa was hard to lift.‎ ‎  D. Because she was afraid the train would kill him.‎ ‎  [解析] 事实细节题。从最后一段"...I was thinking he was going to die."可知,Frank的女友担心Frank会被列车轧死。‎ ‎  [答案] D ‎  3.How did Frank save Lisa?‎ ‎  A. By lifting her to the platform.‎ ‎  B. By helping her rise to her feet.‎ ‎  C. By pulling her along the ground.‎ ‎  D. By dragging her away from the edge.‎ ‎  [解析] 事实细节题。从第五段"...he managed to raise her the four feet to the platform..."可知,Frank把Lisa抱到了站台边。‎ ‎  [答案] A ‎  4.When did Lisa become conscious again?‎ ‎  A. When the train was leaving.‎ ‎  B. After she was back on the platform.‎ ‎  C. After the police and fire officials came.‎ ‎  D. When a man was cleaning the blood from her head.‎ ‎  [解析] 事实细节题。从第五段"...felt herself being pulled along the ground..."可知,Lisa是在被抱到站台上后才恢复了知觉。‎ ‎  [答案] B ‎  5. The passage is intended to________.‎ ‎  A. warn us of the danger in the subway ‎  B. show us how to save people in the subway ‎  C. tell us about a subway rescue ‎  D. report a traffic accident ‎  [解析] 主旨大意题。全文讲述了Lisa在地铁站因晕倒而差点遇险,不过幸而得到Frank及时相救的故事。‎ ‎  [答案] C ‎(第88篇)‎ ‎  Computer programmer David Jones earns £ 35,000 a year designing new computer games,yet he cannot find a bank ready to let him have a credit card(信用卡). Instead,he has been told to wait another two years,until he is 18. The 16yearold works for a small firm in Liverpool, where the problem of most young people of his age is finding a job. David's firm releases (推出) two new games for the fast growing computer market each month.‎ ‎  But David's biggest headache is what to do with his money. Even though he earns a lot, he cannot drive a car, take out a mortgage (抵押贷款), or get credit cars. David got his job with the Liverpool-based company four months ago, a year after leaving school with six O-levels and working for a time in a computer shop. "I got the job because the people who run the firm ‎ knew I had already written some programs, "he said. David spends some of his money on records and clothes, and gives his mother 50 pounds a week. But most of his spare time is spent working.‎ ‎  "Unfortunately, computing was not part of our studies at school,"he said. "But I had been studying it in books and magazines for four years in my spare time. I knew what I wanted to do and never considered staying on at school. Most people in this business are fairly young, anyway. "David added:"I would like to earn a million and I suppose early retirement (退休)is a possibility. You never know when the market might disappear."‎ ‎  [语篇解读] 本文介绍了16岁的大卫参加了工作以及其所遇到的困难。‎ ‎  1. In what way is David different from people of his age?‎ ‎  A. He often goes out with friends.‎ ‎  B. He lives with his mother.‎ ‎  C. He has a handsome income.‎ ‎  D. He graduated with six O-levels.‎ ‎  [解析] 事实细节题。根据第一段第一句,可知他有可观的收入。‎ ‎  [答案] C ‎  2. What is one of the problems that David is facing now?‎ ‎  A. He is too young to get a credit card.‎ ‎  B. He has no time to learn driving.‎ ‎  C. He has very little spare time.‎ ‎  D. He will soon lose his job.‎ ‎  [解析] 事实细节题。根据第一段第一句中...yet he cannot find a bank ready to let him have a credit card.可知他太小了而不能从银行办理信用卡。‎ ‎  [答案] A ‎  3. Why was David able to get the job in the company?‎ ‎  A. He had done well in all his exams.‎ ‎  B. He had written some computer programs.‎ ‎  C. He was good at playing computer games.‎ ‎  D. He had learnt to use computers at school.‎ ‎  [解析] 事实细节题。根据第二段第四句I got the job because the people who run the firm knew I had already written some programs...可知B项正确。‎ ‎  [答案] B ‎  4. Why did David decide to leave school and start working?‎ ‎  A. He received lots of job offers.‎ ‎  B. He was eager to help his mother.‎ ‎  C. He lost interest in school studies.‎ ‎  D. He wanted to earn his own living.‎ ‎  [解析] 推理判断题。根据最后一段Unfortunately,computing was not part of our studies at school...I knew what I wanted to do and never considered staying on at school.可推出此题应选C。‎ ‎  [答案] C ‎(第89篇)‎ ‎  "In only six days I lost seven pounds of weight."‎ ‎  "Two full inches in the first three days !"‎ ‎  These are the kinds of statements used in magazine, newspaper,radio and television ads, promising new shapes and new looks to those who buy the medicine or the device. The promoters of such products say they can shape the legs, slim the face, smooth wrinkles,or in some other way add to beauty or desirability.‎ ‎  Often such products are nothing more than money-making things for their promoters. The results they produce are questionable, and some are dangerous to health.‎ ‎  To understand how these products can be legally promoted to the public,it is necessary to understand something of the laws covering their regulation. If the product is a drug, FDA ( Food and Drug Administration) can require proof(证明) under the Food, Drug, and Cosmetic Act that it is safe and effective before it is put on the market. But if the product is a device,FDA has no authority to require pre-marketing proof of safety or effectiveness. If a product already on the market is a danger to health, FDA can request the producer or distributor to remove it from the market voluntarily, or it can take legal action, including seizure (查封) of the product.‎ ‎  One notable case a few years ago involved an electrical device called the Relaxacisor, which had been sold for reducing the waistline. The Relaxacisor produced electrical shocks to the body through contact pads. FDA took legal action against the distributor to stop the sale of the devices on the grounds that it was dangerous to health and life.‎ ‎  Obviously, most of the devices on the market have never been the subject of court proceedings(法律诉讼), and new devices appear continually. Before buying, it is up to judge the consumer to judge the safety or effectiveness of such items.‎ ‎  [语篇解读] 一些所谓的塑身、健美、美容的商品广告,商家和促销商仅仅是为了挣钱,而实际的结果令人怀疑,有些还有害于健康。事实证明,许多投放在市场上的医疗设备并不受法律约束,在购买前消费者要自己判断它们的安全和效果。‎ ‎  1. It can be inferred that the ads mentioned in the text are________.‎ ‎  A. objective        B. costly ‎  C. unreliable D. illegal ‎  [解析] 推理判断题。由第四段可知,促销商仅仅为了赚钱,而这些产品的效果令人怀疑。‎ ‎  [答案] C ‎  2. Which of the following is true according to the text?‎ ‎  A.The court is in charge of removing dangerous products.‎ ‎  B. New products are more likely to be questionable.‎ ‎  C. The production of a device must be approved by FDA.‎ ‎  D. The promoters usually just care about profits.‎ ‎  [解析] 事实细节题。由第四段第一句可知,促销商仅仅为了赚钱。‎ ‎  [答案] D ‎  3. FDA can ask for the proof of safety and effectiveness of a product________.‎ ‎  A. if it is a drug ‎  B. if it is a device ‎  C. if its consumers make complaints ‎  D. if its distributors challenge FDA's authority ‎  [解析] 事实细节题。由第五段的第二句可知。‎ ‎  [答案] A ‎  4. The Relaxacisor is mentioned as________.‎ ‎  A. a product which was designed to produce electricity ‎  B. a product whose distributor was involved in a legal case ‎  C. a successful advertisement of a beauty product ‎  D. an example of a quality beauty product ‎  [解析] 事实细节题。由第六段可知,Relaxacisor是一种有害于身体的医疗设备,并因此引发了一个案件。‎ ‎  [答案] B ‎  5. The author intends to ________.‎ ‎  A. make consumers aware of the promoters' false promises ‎  B. show the weakness of the law on product safety ‎  C. give advice on how to keep young and beautiful ‎  D. introduce the organization of FDA ‎  [解析] 推理判断题。综观全文,作者的目的是想让消费者意识到广告的虚假承诺。‎ ‎  [答案] A ‎(第90篇)‎ ‎  Transport Guide ‎  The Brisbane City Council (BCC) is responsible for bus and ferry services within the city limits and suburbs. Most buses will either arrive at the city or an interchange where connecting buses can be caught. BCC buses operate from 5∶30 am to 11∶00 pm Monday to Thursday and 5∶30 am to 12∶00 am on Fridays. On weekends and public holidays buses operate less frequently. Pre-paid bus tickets can be purchased from the QUT (Queensland University of Technology) bookshop, the campus news agency, most other news agencies and general stores,and any BCC Customer Service Center. Short-term students at QUT cannot use their ID cards to gain a discount fare on BCC public transport. You will need to buy an adult ticket to travel. Bus fares are dependent on the number of zones you have to travel. There are several types of tickets.‎ Zone Cost ( AUD )‎ Single Daily Off-peak Daily Ten trip Saver Weekly Monthly ‎1‎ ‎2.50‎ ‎4.20‎ ‎3.50‎ ‎16.80‎ ‎16.80‎ ‎67.20‎ ‎2‎ ‎2.90‎ ‎5.00‎ ‎4.10‎ ‎20.00‎ ‎20.00‎ ‎80.00‎ ‎3‎ ‎3.30‎ ‎5.80‎ ‎4.70‎ ‎23.20‎ ‎23.20‎ ‎92.80‎ ‎  Single:one way ticket to reach your destination, including transfers within 2 hours.‎ ‎  Daily:unlimited travel within the zones.‎ ‎  Off-peak Daily:discounted unlimited travel between 9∶00 am and 3∶30 pm and after 7∶00 pm Monday to Friday,and all day on weekends and public holidays.‎ ‎  Weekly: unlimited travel within the zones for one week from the date of issue.‎ ‎  Monthly:unlimited travel within the zones for one calendar month from the date of issue.‎ ‎  Ten-trip Saver: 10 trips at any time within the zones on buses and ferries only.‎ ‎  Transport routes, timetables and fare information are available from:‎ ‎  Public Transport Information Centre ‎  69 Ann Street( corner of George St)‎ ‎  Brisbane City ‎  Phone 13 12 30( Transport Information Service)‎ ‎  [语篇解读] 本文是一篇应用文体裁的文章,主要介绍几种交通方式。‎ ‎  1. The transport guide above is most likely provided by ________.‎ ‎  A. Public Transport Information Centre ‎  B. the Brishane City Council ‎  C. Queensland University of Technology ‎  D. BCC Customer Service Centres ‎  [解析] 事实细节题。根据文章第一段,得知本文交通指南是由Queensland University of Technology提供的,所以答案为C。‎ ‎  [答案] C ‎  2. We can learn from the passage that________.‎ ‎  A. buses are scheduled as usual on weekends and public holidays ‎  B. regular students at QUT need to buy adult tickets ‎  C. prepaid tickets can be bought from the Public Transport Information Centre ‎  D. Ten trip Savers can be used at off-peak time ‎  [解析] 事实细节题。根据文章中的"Ten-trip Saver:10 trips at any time within the zones on buses and ferries only,"得知,Ten-trip Savers可用于任何时间。所以答案为D。‎ ‎  [答案] D ‎  3. An exchange student staying at QUT for five days has to travel between zones every day. What type of ticket would he probably buy?‎ ‎  A. Single. B. Weekly.‎ ‎  C. Off-peak Daily. D. Ten-trip Saver.‎ ‎  [解析] 事实细节题。根据文章第五段"Weekly:unlimited travel within the zones for one week from the date of issue."得知,在一周内可用Weekly。所以答案为B。‎ ‎  [答案] B ‎(第91篇)‎ ‎  I know what you're thinking: pizza (比萨饼)? For breakfast? But the truth is that you can have last night's leftovers in the a. m. if you want to.‎ ‎  I know lots of women who skip breakfast(不吃早餐), and they have a ton of different excuses for doing it. Some say they don't have time, others think they're saving calories (卡路里) , still others just don't like breakfast food.‎ ‎  But the bottom line is that eating in the morning is very important when you're trying to lose weight. "Eating just about anything from 300 to 400 calories would be better than ‎ nothing at all," says Katherine Brooking, R. D. , who developed the super-easy eating plan for this year's "SELF CHALLENGE". And even pizza can be healthy if it's loaded with vegetables,and you stick to one small piece.‎ ‎  Breakfast is one meal I never miss, and the same goes for most weight loss success stories. Research shows that eating breakfast keeps you from overeating later in the day. Researchers at the University of Southern California found that breakfast skippers have a bigger chance of gaining weight than those who regularly have a morning meal.‎ ‎  So eat something in the morning, anything. I know plenty of friends who end up having no breakfast altogether,and have just coffee or orange juice. I say,try heating up last night's leftovers-it may sound crazy, but if it works for you, do it! I find if I tell myself,"You can always eat it tomorrow, "I put away the leftovers instead of eating more that night. Try it...you may save yourself some pre-bed-time calories. And watch your body gain the fat-burning effects.‎ ‎  [语篇解读] 随着科技的进步、社会的发展,人们的生活节奏也越来越快。为了节省时间或者是为了保持苗条身材,很多人不吃早餐。然而这样做是绝对不科学的,尤其对于那些想要减肥的人是极其不利的。‎ ‎  1. The word" leftovers" in Paragraph 1 probably means________.‎ ‎  A. food remaining after a meal ‎  B. things left undone ‎  C. meals made of vegetables ‎  D. pizza topped with fruit ‎  [解析] 猜测词义题。从第一段last night可以猜测出leftovers的含义为"剩饭"。‎ ‎  [答案] A ‎  2. What can we infer from the text?‎ ‎  A. Working women usually have breakfast in a hurry.‎ ‎  B. Many people have wrong ideas about breakfast.‎ ‎  C. There are some easy ways of cooking a meal.‎ ‎  D. Eating vegetables helps save energy.‎ ‎  [解析] 推理判断题。第二段告诉我们很多人有种种借口不吃早餐,第三段开始"but"一词告诉我们这样做是不对的。‎ ‎  [答案] B ‎  3. According to the last paragraph,it is important to________.‎ ‎  A. eat something for breakfast ‎  B. be careful about what you eat ‎  C. heat up food before eating it ‎  D. eat calorie-controlled food ‎  [解析] 事实细节题。最后一段第一句告诉我们:早餐吃什么都可以,但要吃。‎ ‎  [答案] A ‎  4.The text is written mainly for those________.‎ ‎  A. who go to work early ‎  B. who want to lose weight ‎  C. who stay up late ‎  D. who eat before sleep ‎  [解析] 主旨大意题。第三段第一句以及第四段第一句都提示我们:本文主要写给那些想减肥的人。‎ ‎  [答案] B ‎(第92篇)‎ ‎  When I was seven my father gave me a Timex,ray first watch. I loved it, wore it for years, and haven't had another one since it stopped ticking a decade ago. Why? Because I don't need one. I have a mobile phone and I'm always near someone with an iPod or something like that. All these devices(装置) tell the time - which is why, if you look around, you'll see lots of empty wrists; sales of watches to young adults have been going down since 2007.‎ ‎  But while the wise have realized that they don't need them,others-apparently including some distinguished men of our time - are spending total fortunes on them. Brands such as Rolex, Patek Philippe and Breitling command shocking prices, up to £250,000 for a piece.‎ ‎  This is ridiculous. Expensive cars go faster than cheap cars. Expensive clothes hang better than cheap clothes. But these days all watches tell the time as well as all other watches. Expensive watches come with extra functions - but who needs them? How often do you dive to 300 metres into the sea or need to find your direction in the area around the South Pole? So why pay that much of five years' school fees for watches that allow you to do these things?‎ ‎  If justice were done, the Swiss watch industry should have closed down when the Japanese discovered how to make accurate watches for a five-pound note. Instead the Swiss reinvented the watch ,with the aid of millions of pounds' worth of advertising, as a message about the man wearing it. Rolexes are for those who spend their weekends climbing icy mountains; a Patek Philippe is for one from a rich or noble family; a Breitling suggests you like to pilot planes across the world.‎ ‎  Watches are now classified as "investments" (投资). A 1994 Patek Philippe recently sold for nearly £350,000, while 1960s Rolexes have gone from £15,000 to £30,000 plus in a year. But a watch is not an investment. It's a toy for self-satisfaction, a matter of fashion. Prices may keep going up-they've been rising for 15 years. But when fashion moves on, the owner of that £350,000 beauty will suddenly find his pride and joy is no more a good investment than my childhood Timex.‎ ‎  [语篇解读] 本文从小时候父亲买的那块表说起,简单说明了"表"曾经辉煌的历史和现在的年轻人,包括我,由于各种新设备的出现,如移动电话等上面都可识别时间,不再戴"表"似乎成了一种趋势,然而令人啼笑皆非的是尽管"表"针对年轻人的销售量越来越小,却越来越高档化。‎ ‎  1. The sales of watches to young people have fallen because they________.‎ ‎  A. have other devices to tell the time ‎  B. think watches too expensive ‎  C. prefer to wear an iPod ‎  D. have no sense of time ‎  [解析] 事实细节题。此题比较简单。从第一段直接可以得到答案,特别是I have a mobile phone and I'm always near someone with an iPod or something like that.All these devices tell the time."表"在年轻人当中销售下降的主要原因就是有移动电话等诸如此类的其他设备可替代。‎ ‎  [答案] A ‎  2. It seems ridiculous to the writer that________.‎ ‎  A. people dive 300 metres into the sea ‎  B, expensive clothes sell better than cheap ones ‎  C. cheap cars don't run as fast as expensive ones ‎  D. expensive watches with unnecessary functions still sell ‎  [解析] 事实细节题。从第三段Expensive watches come with extra functions...So why pay that much of five years'school fees for watches that allow you to do these things?和第四段Instead the Swiss reinvented the watch,with the aid of millions of pounds' worth of advertising,as a message about the man wearing it.可以得知,拥有不必要功能的名表还在销售。‎ ‎  [答案] D ‎  3. What can be learnt about Swiss watch industry from the passage?‎ ‎  A. It targets rich people as its potential customers.‎ ‎  B. It's hard for the industry to beat its competitors.‎ ‎  C. It wastes a huge amount of money in advertising.‎ ‎  D. It's easy for the industry to reinvent cheap watches.‎ ‎  [解析] 推理判断题。根据第四段Rolexes are for those who spend their weekends climbing icy mountains;a Patek Philippe is for one from a rich or noble family;a Breitling suggest you like to pilot planes across the world.可以推测出,瑞士的制表产业已经瞄准富人作为它的潜在顾客。‎ ‎  [答案] A ‎  4. Which would be the best title for the passage?‎ ‎  A. Timex or Rolex?‎ ‎  B. My Childhood Timex ‎  C. Watches? Not for Me!‎ ‎  D. Watches - a Valuable Collection ‎  [解析] 主旨大意题。买表吗?我不要!从第一段和最后一段最后一句的总结全文主旨:不买表。‎ ‎  [答案] C ‎(第93篇)‎ ‎  All too often, a choice that seems sustainable (可持续的)turns out on closer examination to be problematic. Probably the best example is the rush to produce ethanol(乙醇) for fuel from corn. Corn is a renewable resource-you can harvest it and grow more, almost limitlessly. So replacing gas with corn ethanol seems like a great idea.‎ ‎  One might get a bit more energy out of the ethanol than that used to make it, which could still make ethanol more sustainable than gas generally, but that's not the end of the problem. Using corn to make ethanol means less corn is left to feed animals and people,which drives up the cost of food. That result leads to turning the fallow land - including, in some cases, rain forest in places such as Brazil - into farmland, which in turn gives off lots of carbon dioxide ( CO2 ) into the air. Finally, over many years, the energy benefit from burning ethanol would make up for the forest loss. But by then, climate change would have progressed so far that it might not help.‎ ‎  You cannot really declare any practice" sustainable" until you have done a complete life-cycle analysis of its environmental(环境的) costs. Even then,technology and public policy keep developing,and that development can lead to unforeseen and undesired results. The admirable goal of living sustainably requires plenty of thought on an ongoing basis.‎ ‎  [语篇解读] 文章通过用玉米转化成乙醇为例子来讨论可持续燃料的问题,来说明一些做法还缺少实际发展利用的基础。‎ ‎  1. What might directly cause the loss of the forest according to the text ?‎ ‎  A. The growing demand for energy to make ethanol.‎ ‎  B. The increasing carbon dioxide in the air.‎ ‎  C. The greater need for farmland.‎ ‎  D. The big change in weather.‎ ‎  [解析] 事实细节题。从第二段"...including,in some cases,rain forest in places such as Brazil-into farmland."可知这些雨林要被变成农田,故选择C。‎ ‎  [答案] C ‎  2. The underlined word "it" in the second paragraph refers to"________".‎ ‎  A. the energy benefit ‎  B. the forest loss ‎  C. climate change ‎  D. burning ethanol ‎  [解析] 猜测词义题。从第二段"...the energy benefit from burning ethanol would make up for the forest loss."可知,上文中的the energy benefit就是下文it所指代的内容,故选择A。‎ ‎  [答案] A ‎  3. The author thinks that replacing gas with corn ethanol is________.‎ ‎  A. impractical B. acceptable ‎  C. admirable D. useless ‎  [解析] 推理判断题。impractical不切实际的;acceptable可以接受的;admirable受尊敬的;useless无用的。从文中最后一句"The admirable goal of living sustainable requires plenty of thought on an ongoing basis."可知,开发这种可持续性燃料还是不太实际,故选择A。‎ ‎  [答案] A ‎  4. What does the author mainly discuss in the text?‎ ‎  A. Technology.‎ ‎  B. Sustainability.‎ ‎  C. Ethanol energy.‎ ‎  D. Environmental protection.‎ ‎  [解析] 主旨大意题。technology技术;sustainability可持续性;ethanol energy乙醇能源;environmental protection环境保护。从文中论述可知作者将乙醇作为例子,以此来谈论可持续问题,故选择B。‎ ‎(第94篇)‎ ‎  I suddenly heard an elephant crying as though frightened. Looking down ,I immediately recognized that something was wrong, and ran down to the edge of the near bank. There I saw Ma Shwe with her three-month-old calf struggling in the fast-rising water, and it was a life-and-death struggle. Her calf was floating and screaming with fear. Ma Shwe was as near to the far bank as she could get, holding her whole body against the rushing water, and keeping the calf pressed against her huge body. Every now and then the rushing water would sweep the calf away.‎ ‎  There was a sudden rise in the water and the calf was washed clean over the mother's body and was gone. Ma Shwe turned quickly to reach it and pressed the calf with her head and trunk(象鼻) against the rocky bank. Then with a huge effort, she picked it up in her trunk and tried until she was able to place it on a narrow shelf of rock.‎ ‎  Just at this moment, she fell back into the river. If she were carried down,it would be certain death. I knew, as well as she did, that there was one spot (地点) where she could get up the bank, but it was on the other side from where she had put her calf.‎ ‎  While I was wondering what I could do next, I heard the sound of a mother's love. Ma Shwe had crossed the river and got up the bank and was making her way back as fast as she could, roaring(吼叫) all the time, but to her calf it was music.‎ ‎  [语篇解读] 本文讲述了一只大象的母爱。正是这伟大的母爱使得大象一次次冒着生命的危险在洪水中救小象。‎ ‎  1. The moment the author got down to the river bank he saw________.‎ ‎  A. the calf was about to fall into the river ‎  B. Ma Shwe was placing the calf on the rock ‎  C. the calf was washed away by the rising water ‎  D. Ma Shwe was holding the calf against the rushing water ‎  [解析] 事实细节题。根据第一段第三句There I saw Ma Shwe with her three-month-old calf struggling in the fast-rising water...可知D项正确。‎ ‎  [答案] D ‎  2. How did Ma Shwe manage to save her calf from the fast-flowing water?‎ ‎  A. By putting it on a safe spot.‎ ‎  B. By pressing it against her body.‎ ‎  C. By taking it away with her.‎ ‎  D. By carrying it on her back.‎ ‎  [解析] 事实细节题。根据第二段最后一句Then with a huge effort,she picked it up in her trunk and tried until she was able to place it on a narrow shelf of rock.可知母象把小象放在了一个安全地点。‎ ‎  [答案] A ‎  3. How did the calf feel about the mother elephant's roaring?‎ ‎  A. It was a great comfort.‎ ‎  B. It was a sign of danger.‎ ‎  C. It was a call for help.‎ ‎  D. It was a musical note.‎ ‎  [解析] 推理判断题。根据最后一段最后一句...roaring all the time,but to her calf it was music.可推出选A。‎ ‎  [答案] A ‎  4. What can be the best title for the text?‎ ‎  A. A Mother's Love ‎  B. A Brave Act ‎  C. A Deadly River ‎  D. A Matter of Life and Death ‎  [解析] 主旨大意题。根据全文以及最后一段第一句While I was wondering what I could do next,I heard the sound of a mother's love.可知:是伟大的母爱使得大象一次一次地冒着生命的危险救小象。‎ ‎  [答案] A ‎(第95篇)‎ ‎  A few years ago I had an " aha!" moment regarding handwriting.‎ ‎  I had in my hand a sheet of paper with handwritten instructions on it for some sort of editorial task. It occurred at first that I did not recognize the handwriting, and then I realized whose it must be. I finally became aware of the fact that I had been working with this colleague for at least a year, maybe two, and yet I did not recognize her handwriting at that point.‎ ‎  It was a very important event in the computerization of life - a sign that the informal, friendly communication of people working together in an office had changed from notes in pen ‎ to instant messages and emails. There was a time when our workdays were filled with little letters, and we recognized one another's handwriting the way we knew voices or faces.‎ ‎  As a child visiting my father's office, I was pleased to recognize,in little notes on the desks of his staff,the same handwriting I would see at home in the notes he would leave on the fridge - except that those notes were signed "dad" instead of" RFW".‎ ‎  All this has been on my mind because of the talk about The Rise and Fall of Handwriting, a book by Florey. She shows in her book a deep concern about the fall of handwriting and the failure of schools to teach children to write well, but many others argue that people in a digital age can't be expected to learn to hold a pen.‎ ‎  I don't buy it.‎ ‎  I don't want to see anyone cut off from the expressive, personal associations that a pen still promotes better than a digital keyboard does. For many a biographer, part of really getting to know their subjects is learning to read their handwriting.‎ ‎  What some people advocate is teaching one of the many attractive handwritings based on the handwriting of 16thcentury Italy. That may sound impossibly grand-as if they want kids to learn to draw by copying classical paintings. However, they have worked in many school systems.‎ ‎  [语篇解读] 由于一次无法辨认同事字迹的经历,作者意识到了电脑技术对传统的亲笔书写方式的冲击,不过作者希望书法能够得到保存。‎ ‎  1. Why was the author surprised at not recognizing his colleague's handwriting?‎ ‎  A. He had worked with his colleague long enough.‎ ‎  B. His colleague's handwriting was so beautiful.‎ ‎  C. His colleague's handwriting was so terrible.‎ ‎  D. He still had a lot of work to do.‎ ‎  [解析] 事实细节题。从第二段"I had been working with this colleague for at least a year..."可知,使作者吃惊的原因是他与一位同事共事一年之久竟认不出他的笔迹。‎ ‎  [答案] A ‎  2. People working together in an office used to________.‎ ‎  A. talk more about handwriting ‎  B. take more notes on workdays ‎  C. know better one another's handwriting ‎  D. communicate better with one another ‎  [解析] 事实细节题。从第三段"There was a time...we recognized one another's handwriting the way we knew voices and faces."可知,过去的同事都互相熟知对方,认得各自的笔迹。‎ ‎  [答案] C ‎  3. The author's father wrote notes in pen________.‎ ‎  A. to both his family and his staff ‎  B. to his family in small letters ‎  C. to his family on the fridge ‎  D. to his staff on the desk ‎  [解析] 事实细节题。从第四段"...the same handwriting I would see..."可知作者的父亲曾写便条给员工以及家人。‎ ‎  [答案] A ‎  4.According to the author,handwritten notes________.‎ ‎  A. are harder to teach in schools ‎  B. attract more attention ‎  C. are used only between friends ‎  D. carry more message ‎  [解析] 推理判断题。从倒数第二段"For many a biographer,part of really getting to know their subjects is learning to read their handwriting."可知:对于一些撰写传记的作家来说,真正了解自己的写作对象,部分要依赖于阅读他们(被写传记者)的亲手笔迹。所以我们可以推断笔迹承载了很多的个人信息。‎ ‎  [答案] D ‎  5.We can learn from the passage that the author________.‎ ‎  A. thinks it impossible to teach handwriting ‎  B. does not want to lose handwriting ‎  C. puts the blame on the computer ‎  D. does not agree with Florey ‎  [解析] 推理判断题。从倒数第三段"I don't buy it."和倒数第二段"I don't want to see anyone cut off from...personal association that a pen still promotes better..."可知作者不希望亲手笔迹消亡。‎ ‎(第96篇)‎ ‎  When people hear a president speak, they seldom think about others helping to shape the presentation (报告). Today, however,presidents depend on writers such as J. Terry Edmonds to help them communicate (交流) effectively. Edmonds is the first African American ever to work as a full-time speechwriter for a U. S. president;he is also the first African American to serve as director of speechwriting for the White House. His is an all-American story of success.‎ ‎  Edmonds grew up in Baltimore, Maryland; his father drove a truck,and his mother worked as a waitress. A great reader, Edmonds showed a gift for writing at his high school,Baltimore City College. After graduating in 1967, Edmonds went on to Morgan State University.‎ ‎  Edmonds began his career in business, with jobs in public relations and communications. He joined the world of politics as news secretary for his congressman (国会议员) from Baltimore. During Bill Clinton's presidency, he wrote speeches for Health and Human Services Secretary Donna Shalala and worked in a number of jobs in the White House and in governmental departments. President Clinton then appointed(任命) him to the office of director ‎ of speechwriting. Following the 2000 elections, Edmonds returned to Morgan State University as the school's special assistant to the president for 2001-2002.‎ ‎  [语篇解读] 文章主要介绍美国总统演讲稿撰写人J.Terry Edmonds的成长经历。‎ ‎  1. Which of the following statements is TRUE according to the text?‎ ‎  A. Edmonds proved himself to be good at writing at high school.‎ ‎  B. Edmonds graduated from Morgan State University in 1967.‎ ‎  C. Edmonds was the first full-time speechwriter.‎ ‎  D. Edmonds served the White House after 2000.‎ ‎  [解析] 事实细节题。根据第二段第二句...Edmonds showed a gift for writing at his high school,Baltimore City College.可知Edmonds上中学时就表现出了写作天赋,故选A。‎ ‎  [答案] A ‎  2. Edmonds entered the world of politics first as________.‎ ‎  A. news secretary for a congressman ‎  B. a speechwriter for President Clinton ‎  C. news secretary in the White House ‎  D. a speechwriter for Secretary Donna Shalala ‎  [解析] 事实细节题。根据第三段第二句He joined the world of politics as news secretary for his congressman(国会议员)from Baltimore.可知答案为A。‎ ‎  [答案] A ‎  3. The text is most likely to be found in a book about________.‎ ‎  A. popular science B. historical events ‎  C. successful people D. political systems ‎  [解析] 推理判断题。本文主要是介绍Edmonds的成功经历,故本题选C。‎ ‎  [答案] C ‎(第97篇)‎ ‎  Outside her shabby cottage, old Mrs. Tailor was hanging out laundry on a wire line, unaware that some children lay hidden in the leaves of a nearby tree watching her every move. They were determined to find out if she really was a witch.‎ ‎  They watched as she took a broomstick to clean the dirt from her stone steps. But, much to their disappointment, she did not mount the broomstick and take flight. Suddenly,the old lady's work was interrupted by the cackling of her hen-a signal that an egg had been laid in the warm nest on top of the haystack.‎ ‎  The old broomstick was put aside as she hobbled off towards the haystack followed by Sooty, a black cat she had rescued from a fox trap some time back. With only three legs, it was hard for Sooty to keep up with the old lady. The cat provided proof-the children were sure that only a witch would own a black cat with three legs.‎ ‎  There ,standing on a wooden box, was Mrs. Tailor, stretching out to gather her precious egg. Taking the egg in one of her hands, she began to climb down when, without warning, the box broke and the old lady fell.‎ ‎  "We have to go and help her, "whispered Amy.‎ ‎  "What if it is a trick?"replied Ben.‎ ‎  "Don't be silly, Ben. If she were a witch, she would have turned us into frogs already,"reasoned Meg. "Come on Amy, let's go. "The girls climbed down the tree and ran all the way to the haystack.‎ ‎  Approaching carefully, they could see a wound on the old lady's face. She had knocked her head on a stone and her ankle was definitely broken. "Go and get Dad," Amy yelled to her brother. "Tell him about the accident."‎ ‎  The boys did not need another excuse to leave. They ran as fast as they could for help, hoping that Mrs. Tailor would not wake and turn the girls into frogs.‎ ‎  [语篇解读] 孩子们对住在破旧房舍里的、用扫帚柄扫台阶的、后面跟着一只黑色的三条腿的猫的Old Mrs.Tailor是不是巫师很感兴趣,于是躲在附近的树上偷偷观察,结果如何呢?本文的寓意又是什么呢?‎ ‎  1. Why were the children hiding in the tree?‎ ‎  A. They wanted to watch Mrs. Tailor do her housework closely.‎ ‎  B. They were playing a hide-and-seek game.‎ ‎  C. They wanted to find out if the rumors about Mrs. Tailor were true.‎ ‎  D. They were pretending to be spies.‎ ‎  [解析] 事实细节题。由首段尾句They were determined to find out if she really was a witch.可知孩子们想看看人们对老太太是巫师的传言是否是真的。‎ ‎  [答案] C ‎  2. Mrs. Tailor stopped sweeping when________.‎ ‎  A. her front steps were clean ‎  B. she noticed the children in the tree ‎  C. she was ready to take a flight ‎  D. she heard the hen cackling ‎  [解析] 事实细节题。由第二段倒数最后一句可知答案为D。‎ ‎  [答案] D ‎  3. Ben did not rush to help Mrs. Tailor because________.‎ ‎  A. he thought that she could be tricking them ‎  B. he knew that they should not have been in the tree ‎  C. he did not see the old lady fall down ‎  D. he was afraid of the three-legged cat ‎  [解析] 推理判断题。从文中描述的内容可知,Ben认为Mrs. Tailor是一个巫师,他认为可能是Mrs. Tailor在使诡计,所以在两个女孩子前去救助时,他依然没有奔向前去帮助Mrs. Tailor。‎ ‎  [答案] A ‎  4.Which of these old sayings best suits the story's lesson for us?‎ ‎  A. Make hay while the sun shines.‎ ‎  B. Never judge a book by its cover.‎ ‎  C. People in glasshouses should not throw stones.‎ ‎  D. A bird in the hand is worth two in the bush.‎ ‎  [解析] 推理判断题。通读全文可以推知,作者想告诉我们"不要以貌取人或仅从事物的表象来判断事物"。‎ ‎  [答案] B ‎(第98篇)‎ ‎  Andrew Ritchie, inventor of the Brompton folding bicycle, once said that the perfect portable bike would be" like a magic carpet...You could fold it up and put it into your pocket or handbag". Then he paused:" But you'll always be limited by the size of the wheels. And so far no one has invented a folding wheel."‎ ‎  It was a rare-indeed unique-occasion when I was able to put Ritchie right. A 19th-century inventor,William Henry James Grout,did in fact design a folding wheel. His bike, predictably named the Grout Portable, had a frame that split into two and a larger wheel that could be separated into four pieces. All the bits fitted into Grout's Wonderful Bag,a leather case.‎ ‎  Grout's aim: to solve the problems of carrying a bike on a train.Now doesn't that sound familiar? Grout intended to find a way of making a bike small enough for train travel: his bike was a huge beast. And importantly, the design of early bicycles gave him an advantage:in Grout's day ,tyres were solid,which made the business of splitting a wheel into four separate parts relatively simple. You couldn't do the same with a wheel fitted with a one-piece inflated(充气的) tyre.‎ ‎  So,in a 21st-century context, is the idea of the folding wheel dead? It is not. A British design engineer, Duncan Fitzsimons, has developed a wheel that can be squashed into something like a slender ellipse(椭圆). Throughout,the tyre remains inflated.‎ ‎  Will the young Fitzsimons's folding wheel make it into production? I haven't the foggiest idea. But his inventiveness shows two things. First, people have been saying for more than a century that bike design has reached its limit, except for gradual advances. It's as silly a concept now as it was 100-years-ago:there's plenty still to go for. Second, it is in the field of folding bikes that we are seeing the most interesting inventions. You can buy a folding bike for less than £1,000 that can be knocked down so small that it can be carried on a plane - minus wheels, of course-as hand baggage.‎ ‎  Folding wheels would make all manner of things possible. Have we yet got the magic carpet of Andrew Ritchie's imagination? No. But it's progress.‎ ‎  [语篇解读] 一位发明家想要发明折叠自行车的话,最大的困难是如何折叠轮子。其实19世纪的一位发明家在当年已经做到了,令人欣喜的是现在又有人想出了折叠轮子的办法。我们都在期待着。‎ ‎  1. We can infer from Paragraph 1 that thc Brompton folding bike________.‎ ‎  A. was portable ‎  B. had a folding wheel ‎  C. could be put in a pocket ‎  D. looked like a magic carpet ‎  [解析] 推理判断题。这种自行车的特点是便于携带,而不是呈地毯状或为的是能放进口袋,故不能选C或D。‎ ‎  [答案] A ‎  2. We can learn from the text that the wheels of the Grout Portable________.‎ ‎  A. were difficult to separate ‎  B. could be split into 6 pieces ‎  C. were fitted with solid tyres ‎  D. were hard to carry on a train ‎  [解析] 推理判断题。从文章第三段"And importantly...in Grout's day,tyres were solid..."可得知答案。他的折叠方法只适合当时的固体轮胎。‎ ‎  [答案] C ‎  3. We can learn from the text that Fitzsimons's invention________.‎ ‎  A. kept the tyre as a whole piece ‎  B. was made into production soon ‎  C. left little room for improvement ‎  D. changed our views on bag design ‎  [解析] 推理判断题。从文章第四段"squashed into...like a slender ellipse"可得知答案。‎ ‎  [答案] A ‎  4. Which of the following would be the best title for the text?‎ ‎  A. Three folding bike inventors ‎  B. The making of a folding bike ‎  C. Progress in folding bike design ‎  D. Ways of separating a bike wheel ‎  [解析] 主旨大意题。全文讲述了不同时期的三位发明家对折叠自行车的探索。故选C。‎ ‎  [答案] C ‎(第99篇)‎ ‎  We once had a poster competition in our fifth grade art class.‎ ‎  "You could win prizes," our teacher told us as she wrote the poster information on the blackboard. She passed out sheets of construction paper while continuing," The first prize is ten dollars. You just have to make sure that the words on the blackboard appear somewhere on your poster."‎ ‎  We studied the board critically. Some of us looked with one eye and held up certain colors against the blackboard, rocking the sheets to the right or left while we conjured up our designs. Others twisted their hair around their fingers or chewed their erasers while deep in thought. We had plans for that ten-dollar grand prize, each and every one of us. I'm going to spend mine on candies, one hopeful would announce, while another practiced looking serious, wise and rich.‎ ‎  Everyone in the class made a poster. Some of us used parts of those fancy paper napkins, while others used nothing but colored construction paper. Some of us used big designs, and some of us preferred to gather our art tidily down in one corner of our poster and let the space draw the viewer's attention to it. Some of us would wander past the good students' desks and then return to our own projects with a growing sense of hopelessness. It was yet another grown-up trick of the sort they seemed especially fond of, making all of us believe we had a fair chance, and then always-always-rewarding the same old winners.‎ ‎  I believe I drew a sailboat, but I can't say that with any certainty. I made it. I admired it. I determined it to be the very best of all of the posters I had seen,and then I turned it in.‎ ‎  Minutes passed.‎ ‎  No one came along to give me the grand prize, and then someone distracted me, and I probably never would have thought about that poster again.‎ ‎  I was still sitting at my desk, thinking, What poster? when the teacher gave me an envelope with a ten-dollar bill in it and everyone in the class applauded for me.‎ ‎  [语篇解读] 本文描述了在五年级美术课的一次海报设计竞赛中,作者出乎意料地取得了第一名的经历。‎ ‎  1. What was the teacher's requirement for the poster?‎ ‎  A. It must appear in time.‎ ‎  B. It must be done in class.‎ ‎  C. It must be done on a construction sheet.‎ ‎  D. It must include the words on the blackboard.‎ ‎  [解析] 事实细节题。从第二段"...make sure that the words on the blackboard appear somewhere on your poster."可知,老师的要求是黑板上的内容需在海报中出现。‎ ‎  [答案] D ‎  2.The underlined phrase in Paragraph 3 most probably means ________.‎ ‎  A. formed an idea for ‎  B. made an outline for ‎  C. made some space for ‎  D. chose some colors for ‎  [解析] 猜测词义题。第三段中conjure up意为"想起;使......呈现于脑际"。从对称的结构"Some...while we conjured up our designs."与下文的"Others...while deep in thought."可以猜测出该词词义。‎ ‎  [答案] A ‎  3. After the teacher's words, all the students in the class________.‎ ‎  A. looked very serious ‎  B. thought they would be rich ‎  C. began to think about their designs ‎  D. began to play games ‎  [解析] 事实细节题。第三段生动、形象地描写了同学们思考时的各种动作。‎ ‎  [答案] C ‎  4. After seeing the good students' designs, some students________.‎ ‎  A. loved their own designs more ‎  B. thought they had a fair chance ‎  C. put their own designs in a corner ‎  D. thought they would not win the prize ‎  [解析] 事实细节题。从第四段"...wander past the good students'desks and then return to our own projects with a growing sense of hopelessness."可知:看到优秀学生作品后,另外一些学生对自己这次的参赛作品信心更加不足了。‎ ‎  [答案] D ‎  5. We can infer from the passage that the author________.‎ ‎  A. enjoyed grownup tricks very much ‎  B. loved poster competitions very much ‎  C. felt surprised to win the competition ‎  D. became wise and rich after the competition ‎  [解析] 推理判断题。最后三段描写了作者焦急地等待比赛结果的心情,所以对于不自信的作者来说,这次比赛的结果让他很吃惊。‎ ‎  [答案] C ‎(第100篇)‎ ‎  Few laws are so effective that you can see results just days after they take effect. But in the nine days since the federal cigarette tax more than doubled-to $1.01 per pack-smokers have jammed telephone "quit lines" across the country seeking to kick the habit.‎ ‎  This is not a surprise to public health advocates. They've studied the effect of state tax increases for years, finding that smokers, especially teens, are price sensitive. Nor is it a shock to the industry, which fiercely fights every tax increase.‎ ‎  The only wonder is that so many states insist on closing their ears to the message. Tobacco taxes improve public health, they raise money and most particularly, the deter people from taking up the habit as teens, which is when nearly all smokers are addicted. Yet the rate of taxation varies widely.‎ ‎  In Manhattan, for instance, which has the highest tax in the nation,a pack of Marlboro Light Kings cost $10.06 at one drugstore Wednesday. In Charleston, S. C. , where the 7centapack tax is the lowest in the nation, the price was $4.78.‎ ‎  The influence is obvious.‎ ‎  In New York, high school smoking hit a new low in the latest surveys - 13. 8%, far below the national average. By comparison,26% of high school students smoke in Kentucky. Other low tax states have similarly depressing teen smoking records.‎ ‎  Hal Rogers, Representative from Kentucky, like those who are against high tobacco taxes, argues that the burden of the tax falls on low-income Americans "who choose to smoke."‎ ‎  That's true. But there is more reason in keeping future generations of low-income workers from getting hooked in the first place. As for today's adults, if the new tax drives them to quit, they will have more to spend on their families, cut their risk of cancer and heart disease and feel better.‎ ‎  [语篇解读] 烟草税增加后,取得了立竿见影的效果--许多烟民纷纷戒烟。然而有些地方却对此充耳不闻,中学生吸烟的比例居高不下。本文让我们意识到:从长远看,通过增加烟草税而使人们戒烟是泽被千家万户的大事。‎ ‎  1. The text is mainly about________.‎ ‎  A. the price of cigarettes ‎  B. the rate of teen smoking ‎  C. the effect of tobacco tax increase ‎  D. the differences in tobacco tax rate ‎  [解析] 主旨大意题。本文主要谈论的是烟草税增加后取得的明显效果以及给人们带来的益处。A、B、D均为文章的某一方面。‎ ‎  [答案] C ‎  2. What does the author think is a surprise?‎ ‎  A. Teen smokers are price sensitive.‎ ‎  B. Some states still keep the tobacco tax low.‎ ‎  C. Tobacco taxes improve public health.‎ ‎  D. Tobacco industry fiercely fights the tax rise.‎ ‎  [解析] 事实细节题。由第三段第一句可知,唯一让人吃惊的是,有些州对此充耳不闻,也就是继续保持低烟草税。‎ ‎  [答案] B ‎  3.The underlined word "deter" in Paragraph 3 most probably means________.‎ ‎  A. discourage B. remove ‎  C. benefit D. free ‎  [解析] 猜测词义题。由语境可知,烟草税会改善公众的健康,筹集资金,特别是能让青少年放弃吸烟的习惯。‎ ‎  [答案] A ‎  4. Rogers' attitude towards the low-income smokers might be that of________.‎ ‎  A. tolerance B. unconcern ‎  C. doubt D. sympathy ‎  [解析] 推理判断题。Roger认为高烟草税是把税收负担加到了低收入的烟民身上,故应该持同情态度。‎ ‎  [答案] D ‎  5. What can we learn from the last paragraph?‎ ‎  A. The new tax will be beneficial in the long run.‎ ‎  B. Low-income Americans are more likely to fall ill.‎ ‎  C. Future generations will be hooked on smoking.‎ ‎  D. Adults will depend more on their families.‎ ‎  [解析] 推理判断题。由全文最后一句可知,增加烟草税促使烟民戒烟,可让他们有更多的时间与家人团聚,减少癌症和心脏病的发病率,从长远看,是一件有益的事情。‎ ‎  [答案] A ‎(第101篇)‎ ‎  How Room Designs Affect Our Work and Feelings ‎  Architects have long had the feeling that the places we live in can affect our thoughts, feelings and behaviors. But now scientists are giving this feeling an empirical (经验的,实证的) basis. They are discovering how to design spaces that promote creativity, keep people focused, and lead to relaxation.‎ ‎  Researches show that aspects of the physical environment can influence creativity. In 2007, Joan Meyers-Levy at the University of Minnesota,reported that the height of a room's ceiling affects how people think. Her research indicates that higher ceilings encourage people to think more freely, which may lead them to make more abstract connections. Low ceilings, on the other hand, may inspire a more detailed outlook.‎ ‎  In addition to ceiling height, the view afforded by a building may influence an occupant's ability to concentrate. Nancy Wells and her colleagues at Cornell University found in their study that kids who experienced the greatest increase in greenness as a result of a family move made the most gains on a standard test of attention.‎ ‎  Using nature to improve focus of attention ought to pay off academically, and it seems to according to a study led by C. Kenneth Tanner, head of the School Design & Planning Laboratory at the University of Georgia. Tanner and his team found that students in classrooms ‎ with unblocked views of at least 50 feet outside the window had higher scores on tests of vocabulary, language arts and maths than did students whose classrooms primarily overlooked roads and parking lots.‎ ‎  Recent study on room lighting design suggests that dim(暗淡的) light helps people to loosen up. If that is true generally, keeping the light low during dinner or at parties could increase relaxation. Researchers of Harvard Medical School also discovered that furniture with rounded edges could help visitors relax.‎ ‎  So far scientists have focused mainly on public buildings. "We have a very limited number of studies, so we're_almost_looking_at_the_problem_through_a_straw_(吸管)," architect David Allison says. "How do you take answers to very specific questions and make broad, generalized use of them? That's what we're all struggling with."‎ ‎  [语篇解读] 建筑师认为房子的设计可影响居住者的思想、情绪、行为,对此科学家进行了理论上的论证。‎ ‎  1. What does Joan Meyers-Levy focus on in her research?‎ ‎  A. Light. B. Ceilings.‎ ‎  C. Windows. D. Furniture.‎ ‎  [解析] 事实细节题。由第二段"...the height of a room's ceiling affects how people think."可知选B。‎ ‎  [答案] B ‎  2. The passage tells us that________.‎ ‎  A. the shape of furniture may affect people's feelings ‎  B. lower ceilings may help improve students' creativity ‎  C. children in a dim classroom may improve their grades ‎  D. students in rooms with unblocked views may feel relaxed ‎  [解析] 事实细节题。B、C均属于与事实相反的选项;D项属于移花接木式的错误,由文章倒数第二段最后一句"furniture with rounded edges could help visitors relax"可知。‎ ‎  [答案] A ‎  3.The underlined sentence in the last paragraph probably means that________.‎ ‎  A. the problem is not approached step by step ‎  B. the researches so far have faults in themselves ‎  C. the problem is too difficult for researchers to detect ‎  D. research in this area is not enough to make generalized patterns ‎  [解析] 猜测词义题。根据"We have a very limited number of studies."及"How do you take answers to very specific questions and make broad,generalized use of hem?"可知,对这个地区的研究不足以制定出大众化的模式。‎ ‎  [答案] D ‎  4. Which of the following shows the organization of the passage?‎ ‎  ‎ ‎  CP: Central Point P: Point Sp: Sub-point (次要点)‎ ‎  C: Conclusion ‎  [解析] 推理判断题。根据文章结构先总述房子的设计影响人的创造性、专注性以及休闲性,再分述这三方面,并对第二方面加以分点阐述,最后得出结论:经初步研究结论成立,但我们的研究远远不足,还将继续努力。‎ ‎  [答案] C ‎(第102篇)‎ ‎  Societies all over the world name places in similar ways. Quite often there is no official naming ceremony but places tend to be called names as points of reference by people. Then an organized body steps in and gives the place a name. Frequently it happens that a place has two names:One is named by the people and the other by the government. As in many areas, old habits die-hard,_ and the place continues to be called by its unofficial name long after the meaning is lost.‎ ‎  Many roads and places in Singapore(新加坡) are named in order that the pioneers will be remembered by future generations. Thus we have names such as Stamford Road and Raffles Place. This is in keeping with traditions in many countries-in both the West and the East.‎ ‎  Another way of naming places is naming them after other places. Perhaps they were named to promote friendships between the two places or it could be that the people who used to live there were originally from the places that the roads were named after. The mystery is clearer when we see some of the roads named in former British bases. If you step into Selector Airbase you will see Piccadilly Circus obviously named by some homesick Royal Air Force personnel.‎ ‎  Some places were named after the activities that used to go on at those places. Bras Basah Road is an interesting example. "Bras Basah" means " wet rice" in Malay (马来语). Now why would anyone want to name a road "Wet Rice Road"?The reason is simple. During the pioneering days, wet rice was laid out to dry along this road.‎ ‎  A few roads in Singapore are named by their shapes. There is "Circular Road" for one. Other roads may have part of their names to describe their shapes,like "Paya Lebar Crescent". This road is called a crescent(月牙)because it begins on the main road,makes a crescent and comes back to join the main road again.‎ ‎  [语篇解读] 本文议论世界上不同地区的人们以类似的方式为地点命名,如:以先驱者的名字命名;以其他地点的名字命名;以活动的名称命名;以形状命名等。‎ ‎  1. We learn from Paragraph 1 that________.‎ ‎  A. the government is usually the first to name a place ‎  B. many places tend to have more than one name ‎  C. a ceremony will be held when a place is named ‎  D. people prefer the place names given by the government ‎  [解析] 推理判断题。从第一段第四句Frequently it happens that a place has two names:One is named...推论可知。‎ ‎  [答案] B ‎  2. What does the underlined phrase " die hard" in Paragraph 1 probably mean?‎ ‎  A. Change suddenly.‎ ‎  B. Change significantly.‎ ‎  C. Disappear mysteriously.‎ ‎  D. Disappear very slowly.‎ ‎  [解析] 猜测词义题。从die hard后句...and the place continues to be called by its unofficial name long after the meaning is lost.的意思可判断die hard的词义应为:消失得很慢。‎ ‎  [答案] D ‎  3. Which of the following places is named after a person?‎ ‎  A. Raffles Place. B. Selector Airbase.‎ ‎  C. Piccadilly Circus. D. Paya Lebar Crescent.‎ ‎  [解析] 事实细节题。根据第二段的第一句(特别是pioneers"先驱者"一词)和第二句的内容得知:Raffles Place应是指先驱者的名字。‎ ‎  [答案] A ‎  4. Bras Basah Road is named________.‎ ‎  A. after a person B. after a place ‎  C. after an activity D. by its shape ‎  [解析] 事实细节题。根据第四段第一句和第二句内容很明确地判断:Bras Basah是活动名称。‎ ‎  [答案] C ‎  5. What can be inferred from the passage?‎ ‎  A. Some place names in Singapore are the same as in Britain.‎ ‎  B. Some places in Singapore are named for military purposes.‎ ‎  C. The way Singaporeans name their places is unique.‎ ‎  D. Young Singaporeans have forgotten the pioneers.‎ ‎  [解析] 推理判断题。从第三段最后一句可知。Piccadilly Circus皮卡迪利广场(即伦敦戏院及娱乐中心)。‎ ‎  [答案] A ‎(第103篇)‎ ‎  Successful people in international business understand the cultures of other countries and learn to change their practices in different cultures. They understand the importance of avoiding business decisions based on misconceptions-mistaken ideas.‎ ‎  One cause of misconceptions is ethnocentrism, the belief that one's own culture's way of doing things is better than the way of other cultures. It's ethnocentrism that leads to failure ‎ in international business. To avoid ethnocentrism, it's necessary to study the different elements (组成部分) of culture:language, values and attitudes, and customs and manners.‎ ‎  Language ‎  A knowledge of the local language can help international business people in four ways. First, people can communicate directly.Second, people are usually more open in their communication with someone who speaks their language. Third, an understanding of the language allows people to infer meanings that are not said directly.Finally, knowing the language helps people to understand the culture better.‎ ‎  Values and Attitudes ‎  Values are people's basic beliefs about the difference between right and wrong, good and bad. An attitude is a way of thinking or acting. Values and attitudes influence international business. For example,many people in the United States believe that chocolate from Switzerland is better than chocolate from other countries, and they buy a lot of it.‎ ‎  Customs and Manners ‎  Customs are common social practices. Manners are ways of acting that the society believes are polite. For example, in the United States, it is the custom to have salad (色拉) before the main course at dinner, not after. It's not acceptable to talk with food in mouth at table. Failure to understand the customs and manners of other countries will bring difficulty selling their products. Orange juice as a breakfast drink of an American company in France doesn't sell well because the French don't usually drink juice with breakfast.‎ ‎  [语篇解读] 成功的国际商人之所以能取得成功,一个重要原因是他们能够理解不同的民族文化,并能够根据不同的文化而改变自己的行为。针对如何避免每个人心中由来已久的"民族优越感",文章就态度和价值观及风俗习惯等几方面提出了建议。‎ ‎  1.A knowledge of the local language allows international business people________.‎ ‎  A. to be more open with their customers ‎  B. to communicate without outside help ‎  C. to express their thoughts indirectly ‎  D. to have a better idea of their own culture ‎  [解析] 事实细节题。由第三段中的First,people can communicate directly.可知本题选B。‎ ‎  [答案] B ‎  2.The act of many people buying chocolate of Switzerland shows the role of________.‎ ‎  A. manners        B. values ‎  C. attitudes D. customs ‎  [解析] 推理判断题。由Values and Attitudes这一部分An attitude is a way of thinking or acting.可知答案为C。‎ ‎  [答案] C ‎  3.What would be the best title for the text?‎ ‎  A. Misconceptions in Business ‎  B. Basic Beliefs in Business ‎  C. International Business Culture ‎  D. Successful International Business ‎  [解析] 主旨大意题。第二段是文章的主题段,由本段可知本题选C。‎ ‎  [答案] C ‎  4.The author's purpose of writing this article is to tell people________.‎ ‎  A. how to take a right attitude in business ‎  B. how to avoid misunderstandings in business ‎  C. how to use a local language in business ‎  D. how to act politely and properly in business ‎  [解析] 推理判断题。由第二段可知作者的写作意图是针对如何避免每个人心中由来已久的"民族优越感"提出建议,故本题选B。‎ ‎  [答案] B ‎(第104篇)‎ ‎  "Get your hands off me , I have been stolen," the laptop,a portable computer, shouted. That is a new solution to laptop computer theft: a program that lets owners give their property a voice when it has been taken.‎ ‎  The program allows users to display alerts on the missing computer' s screen and even to set a spoken message. Tracking software for stolen laptops has been on the market for some time, but this is thought to be the first that allows owners to give the thief a piece of their mind.‎ ‎  Owners must report their laptop missing by visiting a website, which sends a message to the model: a red and yellow "lost or stolen" sign appears on its screen when it is started.Under the latest version(版本) of the software, users can also send a spoken message.‎ ‎  The message can be set to reappear every 30 seconds, no matter how many times the thief closes it. "One customer sent a message saying,'You are being tracked. I am right at your door' ," said Carrie Hafeman, chief executive of the company which produces the program, Retriever.‎ ‎  In the latest version, people can add a spoken message.For example, the laptop' s speakers will say: "Help, this laptop is reported lost or stolen. If you are not my owner, report me now."‎ ‎  The Retriever software package, which costs $ 29.95 but has a free trial period, has the functions of many security software programs.Owners can remotely switch to an alternative password if they fear that the thief has also got hold of the access details.‎ ‎  If a thief accesses the internet with the stolen laptop, Retriever will collect information on the internet service provider in use, so that the police can be alerted to its location.‎ ‎  Thousands of laptops are stolen every year from homes and offices, but with the use of laptops increasing, the number stolen while their owners are out and about has been rising sharply.‎ ‎  Other security software allows users to erase data remotely or lock down the computer.‎ ‎  [语篇解读] 随着社会的发展,使用手提电脑的人越来越多,而盗窃手提电脑的行为也变得猖獗。本文介绍了一种新版本的电脑软件,电脑拥有者能够在程序中输入声音,一旦电脑被盗,只要被打开,它就会有规律地提醒盗贼,电脑不是他的,或者帮助主人发泄愤怒。‎ ‎  5.The expression "to give the thief a piece of their mind" can be understood as"________".‎ ‎  A. to give the thief an alert mind ‎  B. to express the owners' anger to the thief ‎  C. to remind the thief of his conscience ‎  D. to make the thief give up his mind ‎  [解析] 细节推理题。根据文章第一段中的"Get your hands off me,I have been stolen"和下文第四段中的"You are being tracked.I am right at your door."可推测出被盗者是很生气的,他通过电脑软件设计出的言语来表达自己的愤怒。所以答案选B。‎ ‎  [答案] B ‎  6.Different from other security software, Retriever can________.‎ ‎  A. record the stealing process ‎  B. help recognize the lost laptop ‎  C. lock down the computer remotely ‎  D. send a spoken message ‎  [解析] 细节理解题。根据文章第五段第一句话"In the latest version,people call add a spoken message."可知D项正确。A项没有提及;B项的recognize表述错误;C项不是这种新的安全程序所特有的,故答案为D。‎ ‎  [答案] D ‎  7.One function of the program is that it allows the owner to________at a distance.‎ ‎  A. change some access details for switching on the laptop ‎  B. turn on the laptop by using the original password ‎  C. operate the laptop by means of an alternative password ‎  D. erase the information kept in the stolen laptop ‎  [解析] 细节理解题。根据文章第六段的最后一句话"Owners can remotely switch to an alternative password if they fear that the thief has also got hold of the access details."可知A项正确。‎ ‎  [答案] A ‎  8.Which of the following can best summarize the main idea of the passage?‎ ‎  A. With no Retriever, thousands of laptops are stolen every year.‎ ‎  B. A new software provides a means to reduce laptop theft.‎ ‎  C. Retriever has helped to find thieves and lost computers.‎ ‎  D. A new program offers a communication platform with the thief.‎ ‎  [解析] 主旨大意题。这篇文章主要介绍了一种新的电脑软件,它可以提供多种安全程序,使得电脑偷盗者无处藏身,从而减少电脑被盗。A项太绝对了;C项只是文章的一个小的方面,并不是主题;D项表述错误,不符合文章的意思。‎ ‎  [答案] B ‎(第105篇)‎ ‎  ‎ ‎  Timetable Mondays to Fridays OxforadLondon Paddington Oxford  London Paddington  Oxford   London Paddington ‎09∶05    10∶01      11∶15     12∶14‎ ‎09∶48 10∶42 11∶45 12∶44‎ ‎10∶05 11∶04 12∶00 12∶56‎ ‎10∶35 11∶34 12∶15 13∶14‎ ‎  Membership(会员)Card ‎  ‎ Menu ‎◇ French Slam(r)$4.29‎ French toast, two eggs any style,two pieces of bacon and two sausage links.‎ ‎◇Sandwich with Salad or Soup$4.35‎ Chicken breast on bread.Served with your choice of garden salad,Caesar salad,vegetable beef or soup of the day.‎ ‎◇The Classic Hamburger$4.99‎ Over 1/3 pound. Topped with tomato, red onions and cheese.‎ ‎◇ChickenFried Steak$5.09‎ A southern style! Goldenfried and covered with French cheese.‎ ‎◇The Super Bird(r)$5.49‎ Thin flat pieces of chicken breast with Swiss cheese, bacon and tomato on bread.‎ ‎◇Chinese Chicken Salad$5.99‎ Chicken breast, mushrooms, green peppers, and onions. Topped with tomatoes and fried noodles. Served with bread.‎ ‎  [语篇解读] 文章介绍了四则广告。‎ ‎  1.Which of the following is true of the membership card?‎ ‎  A. Its number is 10865 305305.‎ ‎  B. It belongs to Mr. E. M. Driscoll.‎ ‎  C. It is valid through the year of 2010.‎ ‎  D. It gets the owner a discount when used.‎ ‎  [解析] 细节判断题。由第二则广告中的"When booking always ask for your Friends Discount"可得出答案。‎ ‎  [答案] D ‎  2.If one wants to attend a business lunch in London at 12∶00, the latest train that he should take at Oxford leaves at________.‎ ‎  A. 11∶45   B. 11∶15‎ ‎  C. 10∶35 D. 10∶05‎ ‎  [解析] 细节理解题。根据题干所给的条件,12点在伦敦参加商业午餐(宴),那么在12点之前就要到达伦敦,在第一则广告中可知,10点35分由牛津发出的这趟车符合要求。‎ ‎  [答案] C ‎  3.If you would like to have some vegetable beef, what may be your choice?‎ ‎  A. French Slam(r).‎ ‎  B. ChickenFried Steak.‎ ‎  C. The Super Bird(r).‎ ‎  D. Sandwich with Salad or Soup.‎ ‎  [解析] 细节理解题。由第四则广告中"Sandwich with Salad or Soup"的内容可得出答案。‎ ‎  [答案] D ‎  4.The chart shows that from 2005 to 2008,________.‎ ‎  A. the percentage of the Spanish families with a computer rose 35 points ‎  B. the percentage of the White families with a computer remained unchanged ‎  C. the number of the Black families with a computer was on the decrease ‎  D. the number of the Asian families with a computer showed the sharpest increase ‎  [解析] 细节理解题。由第三则广告中的柱状图可知,D项是正确的。‎ ‎  [答案] D ‎(第106篇)‎ ‎  New archaeological discoveries suggest that trade between Europe and Asia along the Silk Road probably began in some form many centuries earlier than once thought. The findings, coupled with a widening range of scientific and historical research, could add a fascinating new page to the epic of the Silk Road.‎ ‎  The latest and most surprising discovery is pieces of silk found in the hair of an Egyptian mummy from about 1,000 BC ,long before regular traffic on the Silk Road and at least one thousand years before silk was previously thought to be used in Egypt. Other research may extend human activity along this route back even further, perhaps a million years to the migration of human ancestors into eastern Asia.‎ ‎  The official origin of EastWest commerce along the road is usually placed in the late 2nd century BC when an agent of the Chinese Emperor Wudi returned from a dangerous secret mission (使命)across the western desert into the remote high country of Central Asia. The agent, Zhang Qian, travelled as far as Afghanistan and brought back knowledge of even more distant lands such as Persia,Syria and a place known as Lijien, perhaps Rome. Historians have called this one of the most important journeys in ancient times. His journey opened the way for what have been thought to be the first indirect contacts between the ancient world's two superpowers, China and Rome. Chinese silk, first traded to central Asian tribes for war horses and to the Parthians of old Persia in exchange for acrobats and ostrich eggs ,was soon finding its way through a network of merchants to the luxury markets of Rome.‎ ‎  But the new discoveries show that Chinese silk was apparently present in the West long before the Han emperor started organized trade over the Silk Road. The research could change thinking about the early history of world trade and provide insights into the mystery of just how and when Europe and the Mediterranean lands first became aware of the glorious culture at the other end of Eurasia.‎ ‎  [语篇解读] 最新的考古学研究表明,古代欧亚大陆之间的主要贸易通道--丝绸之路,其起源可能比我们之前认为的要早许多世纪。‎ ‎  5.The word"coupled" in the first paragraph could best be replaced by________.‎ ‎  A. produced B. continued ‎  C. doubled D. combined ‎  [解析] 猜测词义题。由第一段第二句的句意"这些发现,综合广泛的、科学的、历史的研究可以增添丝绸之路历史新纪元迷人的一页"可以推知答案为D。‎ ‎  [答案] D ‎  6.The silk thread found in the hair of an Egyptian mummy suggests that________.‎ ‎  A. Egyptians had probably travelled to China to buy silk ‎  B. trade along the Silk Road began earlier than once thought ‎  C. historical research often achieves fascinating results ‎  D. new light can now be thrown on ancient trading practices ‎  [解析] 推理判断题。由末段首句可以推断出在汉武帝之前就有了丝绸之路的贸易,故B项正确。A项不合常识和事实;C项句意本身没有错误但太宽泛;D项这一事实可以阐明古代贸易实践,不合文意,故排除。‎ ‎  [答案] B ‎  7.Until recently most historians believed that trade along the Silk Road________.‎ ‎  A. originated in the 2nd century BC ‎  B. extended human migration into eastern Asia ‎  C. began a million years ago ‎  D. primarily benefited the Egyptians ‎  [解析] 事实细节题。由第三段首句可知答案为A。‎ ‎  [答案] A ‎  8.Historians have always considered Zhang Qian's mission important because they believe________.‎ ‎  A. he brought back knowledge of Rome to the emperor ‎  B. he discovered the Silk Road ‎  C. he helped establish EastWest trade ‎  D. he travelled as far as Afghanistan ‎  [解析] 推理判断题。由第三段内容可知C正确。‎ ‎  [答案] C ‎  [长难句解读] The research could change thinking about the early history of world trade and provide insights into the mystery of just how and when Europe and the Mediterranean lands first became aware of the glorious culture at the other end of Eurasia.这项研究可能改变人们关于世界贸易早期历史的看法,进一步洞察了对欧洲和地中海地区何时及怎样知道在欧亚大陆的另一端存在着辉煌的文化的奥秘。‎ ‎(第107篇)‎ ‎  RichardSolo 1800 Rechargeable Battery ‎  In just minutes a day,plug in and charge your iPhone quickly!‎ ‎  Just plug RichardSolo 1800 into your iPhone once or twice a day,for fifteen minutes, and keep your iPhone charged up. At your desk, or at dinner, plug RichardSolo into iPhone to instantly transfer charge. No more battery worries. RichardSolo will charge iPhone to full 1.5 times, and it is good for 3 5 years of recharges.‎ ‎  Use the iPhone while charging it. Even charge the RichardSolo 1800 and iPhone together at the same time. Take only one charger when traveling and wake up in the morning with the RichardSolo and the iPhone charged.‎ ‎  RichardSolo 1800 is largest in its class and holds its charge for months. Works with almost all iPhone cases.‎ ‎  Your satisfaction is guaranteed, with our 30day return privilege. If you're not satisfied for any reason, we'll email you a prepaid return label.‎ ‎  Actual customer comments:‎ ‎  To have your company exhibit such good service is unbelievably refreshing.-P.S.‎ ‎  This is what I call great customer support. I wish more companies would figure this out these days. Thank you so much. -D. C.‎ ‎  You have provided me one of the best services I have ever seen on any online/telephone shopping.-T.K.‎ ‎  You must have the fastest processing and shipping in the industry!! -M. C.‎ ‎  This is the best customer service experience I have had in a long time.-L.L.‎ ‎  I've read online about your amazing customer service, and I must say I'm now a true believer. -B. L.‎ ‎  [语篇解读] 我们对电池充电都习以为常,但这里介绍了一种新款充电型电池。它既能被充电又可以为其他电池充电,顾客给予这种电池以高度评价,尤其是其销售服务。‎ ‎  1.How long does it take the battery to charge up an iPhone?‎ ‎  A. 15 minutes. B. 30 minutes.‎ ‎  C. 1.5 hours. D. 3 hours.‎ ‎  [解析] 事实细节题。从第二段第一句可得出答案。‎ ‎  [答案] A ‎  2.What is special about the battery?‎ ‎  A. It is built in an iPhone.‎ ‎  B. It is the smallest of its kind.‎ ‎  C. It can also be used as a charger.‎ ‎  D. It keeps power for about 30 days.‎ ‎  [解析] 推理判断题。从第二段第四句"RichardSolo will charge iPhone to full 1.5 times..."可知该电池同时又是充电器,这是它与众不同之处。‎ ‎  [答案] C ‎  3.Who mentions the transporting of the battery?‎ ‎  A.P.S. B.B.L.‎ ‎  C. M.C. D.T.K.‎ ‎  [解析] 推理判断题。从顾客评论的第四条中"You must have the fastest processing and shipping in the industry!!"可得出答案。该句中shipping是"运输"的意思。‎ ‎  [答案] C ‎  4. The customer comments on the battery are mainly about its________.‎ ‎  A. quality B. service ‎  C. function D. shopping ‎  [解析] 推理判断题。六条顾客评论中有三条(1、5、6条)中提到了它的优质服务。‎ ‎  [答案] B ‎(第108篇)‎ ‎  It is true that good writers rewrite and rewrite and then rewrite some more. But in order to work up the desire to rewrite,it is important to learn to like what you write at the early stage.‎ ‎  I am surprised at the number of famous writers I know who say that they so dislike reading their own writing later that they even hate to look over the publishers' opinions. One reason we may dislike reading our own work is that we're often disappointed that the rich ideas in our minds seem very thin and plain when first written down. Jerry Fodor and Steven Pinker suggest that this fact may be a result of how our minds work.‎ ‎  Different from popular belief, we do not usually think in the words and sentences of ordinary language but in symbols for ideas (known as "mentalese" ) , and writing our ideas down is an act of translation from that symbolic language.But while mentalese contains our thoughts in the form of a complex tapestry (织锦), writing can only be composed one thread at a time. Therefore it should not be surprising that our first attempt at expressing ideas should look so simple. It is only by repeatedly rewriting that we produce new threads and connect them to get closer to the ideas formed in our minds.‎ ‎  When people write as if some strict critics (批评家) are looking over their shoulder, they are so worried about what this critic might say that they get stuck before they even start. Pierre Elbow makes an excellent suggestion to deal with this problem.When writing we should have two different minds. At the first stage, we should see every idea, as well as the words we use to express it, as wonderful and worth putting down. It is only during rewrites that we should examine what we excitedly wrote in the first stage and check for weaknesses.‎ ‎  [语篇解读] 作者感到奇怪的是,很多著名的作家不喜欢看自己写的作品。分析原因,其中之一便是,写的时候,头脑里面的构思非常不错,可是一旦写出来以后,却往往发现与本意有很大的不同。‎ ‎  5.What do we learn from the text about those famous writers?‎ ‎  A. They often regret writing poor works.‎ ‎  B. Some of them write surprisingly much.‎ ‎  C. Many of them hate reading their own works.‎ ‎  D. They are happy to review the publishers' opinions.‎ ‎  [解析] 细节理解题。由第二段第一句话可知答案为C。‎ ‎  [答案] C ‎  6.What do people generally believe about the way human minds work?‎ ‎  A. People think in words and sentences.‎ ‎  B. Human ideas are translated into symbols.‎ ‎  C. People think by connecting threads of ideas.‎ ‎  D. Human thoughts are expressed through pictures.‎ ‎  [解析] 细节理解题。由第三段第一句话可知答案为A。‎ ‎  [答案] A ‎  7.What can we conclude from the text?‎ ‎  A. Most people believe we think in symbols.‎ ‎  B. Loving our own writing is scientifically reasonable.‎ ‎  C. The writers and critics can never reach an agreement.‎ ‎  D. Thinking and writing are different stages of mind at work.‎ ‎  [解析] 推理判断题。综观全文可知,思考和写作是大脑工作的不同阶段。‎ ‎  [答案] D ‎(第109篇)‎ ‎  I was in a rush as always, but this time it was for an important date I just couldn' t be late for! I found myself at a checkout counter behind an elderly woman seemingly in no hurry as she paid for her groceries. A PhD student with not a lot of money, I had hurried into the store to pick up some flowers. I was in a huge rush, thinking of my upcoming evening. I did not want to be late for this date.‎ ‎  We were in Boston, a place not always known for small conversation between strangers. The woman stopped unloading her basket and looked up at me. She smiled. It was a nice smile -warm and reassuring - and I returned her_gift_by smiling back.‎ ‎  "Must be a special lady, whoever it is that will be getting those beautiful flowers," she said.‎ ‎  "Yes, she' s special," I said, and then to my embarrassment, the words kept coming out. "It' s only our second date,but somehow I am just having the feeling she's'the one' ,"Jokingly, I added, "The only problem is that I can' t figure out why she' d want to date a guy like me."‎ ‎  "Well, I think she's very lucky to have a boyfriend who brings her such lovely flowers and who is obviously in love with her," the woman said. "My husband used to bring me flowers every week - even when times were tough and we didn' t have much money. Those were incredible days; he was very romantic and - of course - I miss him since he' s passed away."‎ ‎  I paid for my flowers as she was gathering up her groceries. There was no doubt in my mind as I walked up to her. I touched her on the shoulder and said, "You were right, you know. These flowers are indeed for a very special lady." I handed her the flowers and thanked her for such a nice conversation.‎ ‎  It took her a moment to realize that I was giving her the flowers I had just purchased."You have a wonderful evening,"I said. I left her with a big smile and my heart warmed as I saw her smelling the beautiful flowers.‎ ‎  I remember being slightly late for my date that night and telling my girlfriend the above story. A couple of years later,when I finally worked up the courage to ask her to marry me,she told me that this story had helped to seal it for her - that was the night that I won her heart.‎ ‎  [语篇解读] 作者赶着去和女朋友约会,在他买花的途中,偶遇一个"微笑"的老妇人,他们进行了一次"真诚"的谈话......‎ ‎  1.Why was the writer in a hurry that day?‎ ‎  A. He was to meet his girlfriend.‎ ‎  B. He had to go back to school soon.‎ ‎  C. He was delayed by an elderly lady.‎ ‎  D. He had to pick up some groceries.‎ ‎  [解析] 事实细节题。从全文特别是第一段可知,我们不难发现作者很匆忙的原因是"急着买花,要去赴约(约会女朋友)",答案应该选A。‎ ‎  [答案] A ‎  2.What does the underlined phrase"her gift" (Paragraph 2) refer to?‎ ‎  A. Her words.       ‎ ‎  B. Her smile.‎ ‎  C. Her flowers.‎ ‎  D. Her politeness.‎ ‎  [解析] 词义理解题。从文章第二段中的"She smiled.It was a nice smile-warm and reassuring...",可知此处的"her gift",指代上文的smile,所以答案选B。‎ ‎  [答案] B ‎  3.Why did the writer give his flowers to the elderly lady?‎ ‎  A. She told him a nice story.‎ ‎  B. She allowed him to pay first.‎ ‎  C. She gave him encouragement.‎ ‎  D. She liked flowers very much.‎ ‎  [解析] 逻辑推理题。从作者和老妇人的谈话可知,老妇人讲述的关于她的丈夫对她的关爱的故事深深地感动了作者,同时也鼓励了作者:送花给女朋友是非常值得做的,而且对他女朋友来说,能收到他送的花是很幸运的。‎ ‎  [答案] C ‎  4.What is the message conveyed in the story?‎ ‎  A. Flowers are important for a date.‎ ‎  B. Small talk is helpful.‎ ‎  C. Love and kindness are rewarding.‎ ‎  D. Elderly people deserve respecting.‎ ‎  [解析] 逻辑推理题。从全文可知,这个故事告诉我们"爱和善心会给人们带来幸福",所以C项与本文要传递的信息一致。‎ ‎  [答案] C ‎(第110篇)‎ ‎  We already know the fastest, least expensive way to slow climate change: Use less energy. With a little effort, and not much money, most of us could reduce our energy diets by 25 percent or more - doing the Earth a favor while also helping our wallets.‎ ‎  Not long ago, my wife, PJ, and I tried a new diet - not to lose a little weight but to answer an annoying question about climate change. Scientists have reported recently that the world is heating up even faster than predicted only a few years ago,and that the consequences could be severe if we don' t keep reducing emissions (排放) of carbon dioxide (CO2) and other greenhouse gases that are trapping heat in our atmosphere.‎ ‎  We decided to try an experiment.For one month we recorded our personal emissions of CO2. We wanted to see how much we could cut back, so we went on a strict diet. The average US household(家庭) produces about 150 pounds of CO2 a day by doing common place things like turning on airconditioning or driving cars. That' s more than twice the European average and almost five times the global average, mostly because Americans drive more and have bigger houses. But how much should we try to reduce?‎ ‎  For an answer, I checked with Tim Flannery, author of The Weather Makers: How Man Is Changing the Climate and What It Means for Life on Earth.in his book, he had challenged readers to make deep cuts in personal emissions to keep the world from reaching extremely important tipping_points,such as the melting (融化) of the ice sheets in Greenland or West Antarctica. "To stay below that point, we need to reduce CO2 emissions by 80 percent," he said.‎ ‎  Good advice, I thought.I' d opened our bedroom windows to let in the wind. We' d gotten so used to keeping our airconditioning going around the clock.I' d almost forgotten the windows even opened.We should not let this happen again. It' s time for us to change our habits if necessary.‎ ‎  [语篇解读] 我们已经知道,减缓气候变化最快、最经济的办法就是少用能源。举手之劳,大部分人就能把所耗费的能源减少25%以上,并能减少CO2的排放量--一方面爱护地球,另一方面对自己的钱包也有帮助。那我们何乐而不为呢?‎ ‎  5.Why did the author and his wife try a new diet?‎ ‎  A. To take special kinds of food.‎ ‎  B. To respond to climate change.‎ ‎  C. To lose weight.‎ ‎  D. To improve their health.‎ ‎  [解析] 细节理解题。依据文章第二段中的"not to lose a little weight but to answer an annoying question about climate change"可知,作者这么做的目的是回答一个令人烦恼的气候变化问题。‎ ‎  [答案] B ‎  6.The underlined words "tipping points" most probably refer to "________".‎ ‎  A. freezing points B. burning points ‎  C. melting points D. boiling points ‎  [解析] 猜测词义题。在他的书中,他要读者大幅减少个人温室气体排放量,以免世界来到"引爆的临界点",下面给出的例子"such as the melting(融化)of the ice sheets in Greenland or West Antarctica",可以判断出C项正确。‎ ‎  [答案] C ‎  7.It can be inferred from the passage that________.‎ ‎  A. it is necessary to keep the airconditioning on all the time ‎  B. it seems possible for every household to cut emissions of CO2‎ ‎  C. the average US household produces about 3,000 pounds of CO2 a month ‎  D. the average European household produces about 1,000 pounds of CO2 a month ‎  [解析] 推理判断题。由第一段和最后一段可知大多数人可以减少能源消耗与温室气体的排放量。故选B。‎ ‎  [答案] B ‎  8.Which of the following would be the best title for this passage?‎ ‎  A. Saving Energy Starts at Home ‎  B. Changing Our Habits Begins at Work ‎  C. Changing Climate Sounds Reasonable ‎  D. Reducing Emissions of CO2 Proves Difficult ‎  [解析] 主旨大意题。文章作者通过讲述自己一家人切实有效地减少温室气体排放量的行动,向我们证明了:节约能源从家庭做起,从我做起。故A项正确。‎ ‎  [答案] A ‎(第111篇)‎ ‎  It's only 4 hours flying time from Sydney, but a world away.What better place to rest than a country where the only place people hurry is on the football field and things are done in"Fiji time"?‎ ‎  Viti Levu - Great Fiji - is the largest island. Here you'll find the capital Suva and the international airport at Nadi. Vatoa, on the other hand,is a tiny island in the farthest part of Fiji. Then there are 331 other islands,many of them with places to stay.‎ ‎  With less than a million people living on islands, you'll never feel crowded. And with a climate (气候)that changes only for five degrees between seasons, there's never a bad time to come.‎ ‎  From cities to villages, from mountains to beaches, from water sports to wooden artworks, Fiji can give you more adventures and special experiences than you could find almost anywhere in the world.‎ ‎  Whenever you come, wherever you go, you're sure to see some unforgettable events. From war dances to religious (宗教的)songs.From market days to religious days. It's not just staged for tourists;it's still a part of everyday life in Fiji. And any one of us can enjoy Fiji's spirit by being part of the traditional(传统的) sharing of yaqona, a drink made from the root of a Fiji plant.‎ ‎  So why not join us for the experience of a lifetime?‎ ‎  [语篇解读] 飞行4个小时,到Fiji来看看吧,你绝对会不虚此行,能尽情享受到大自然的馈赠。‎ ‎  1.Where is the international airport of Fiji?‎ ‎  A. In Suva.‎ ‎  B. In Sydney.‎ ‎  C. On the island of Vatoa.‎ ‎  D. On the island of Viti Levu.‎ ‎  [解析] 事实细节题。第二段第一句话和第二句告诉我们:Fiji国际机场在Viti Levu岛上。‎ ‎  [答案] D ‎  2.What does the text tell us about Fijian people?‎ ‎  A. They invented"Fiji time"for visitors.‎ ‎  B. They stick to a traditional way of life.‎ ‎  C. They like to travel from place to place.‎ ‎  D. They love taking adventures abroad.‎ ‎  [解析] 推理判断题。信息依据在倒数第二段。‎ ‎  [答案] B ‎  3.One of the things that make Fiji a tourist attraction is________.‎ ‎  A. its comfortable hotels ‎  B. its good weather all year round ‎  C. its exciting football matches ‎  D. its religious beliefs ‎  [解析] 事实细节题。第三段第二句告诉我们:这里气候全年差距不大,什么时候来都可以。‎ ‎  [答案] B ‎  4.Where can we most probably read this text?‎ ‎  A. In a personal diary.‎ ‎  B. In a science report.‎ ‎  C. In a travel magazine.‎ ‎  D. In a geography textbook.‎ ‎  [解析] 推理判断题。本文向我们介绍了一个旅游胜地,所以本文很有可能来自a travel magazine。‎ ‎  [答案] C ‎(第112篇)‎ ‎  When students and parents are asked to rate subjects according to their importance, the arts are unavoidably at the bottom of the list.Music is nice,people seem to say, but not important. Too often it is viewed as mere entertainment, but certainly not an education priority (优先). This view is shortsighted. In fact, music education is beneficial and important for all students.‎ ‎  Music tells us who we are. Because music is an expression of the beings who create it, it reflects their thinking and values, as well as the social environment it came from. Rock music represents a lifestyle just as surely as does a Schubert song. The jazz influence that George Gershwin and other musicians introduced into their music is obviously American because it came from American musical traditions. Music expresses our character and values. It gives us identity as a society.‎ ‎  Music provides a kind of perception (感知) that cannot be acquired any other way. Science can explain how the sun rises and sets. The arts explore the emotive (情感的) meaning of the same phenomenon. We need every possible way to discover and respond to our world for one simple but powerful reason:No one way can get it all.‎ ‎  The arts are forms of thought as powerful in what they communicate as mathematical and scientific symbols. They are ways we human beings"talk" to each other. They are the language of civilization through which we express our fears, our curiosities, our hungers, our discoveries, our hopes. The arts are ways we give form to our ideas and imagination so that they can be shared with others. When we do not give children access to an important way of expressing themselves such as music,we take away from them the meanings that music expresses.‎ ‎  Science and technology do not tell us what it means to be human. The arts do. Music is an important way we express human suffering, celebration, the meaning and value of peace and love.‎ ‎  So music education is far more necessary than people seem to realize.‎ ‎  [语篇解读] 音乐课在学校课程设置中被学生家长认为可有可无,无足轻重;但是音乐有着丰富的内涵,是学校教育必不可少的组成部分,请重视音乐课吧。‎ ‎  5.According to Paragraph 1, students________.‎ ‎  A. regard music as a way of entertainment ‎  B. disagree with their parents on education ‎  C. view music as an overlooked subject ‎  D. prefer the arts to science ‎  [解析] 事实细节题。根据第一段第三句"Too often it is viewed as mere entertainment..."可知。‎ ‎  [答案] A ‎  6.In Paragraph 2,the author uses jazz as an example to________.‎ ‎  A. compare it with rock music ‎  B. show music identifies a society ‎  C. introduce American musical traditions ‎  D. prove music influences people's lifestyles ‎  [解析] 推理判断题。根据第二段The jazz influence...is obviously American because it came from American musical traditions.以及第二段最后一句It gives us identity as a society.可知,音乐可以识别一个社会。‎ ‎  [答案] B ‎  7.According to the passage ,the arts and science________.‎ ‎  A. approach the world from different angles ‎  B. explore different phenomena of the world ‎  C. express people's feelings in different ways ‎  D. explain what it means to be human differently ‎  [解析] 推理判断题。由第四段第二句"They are ways we human beings 'talk'to each other."可知。‎ ‎  [答案] A ‎  8.What is the main idea of the passage?‎ ‎  A. Music education deserves more attention.‎ ‎  B. Music should be of top education priority.‎ ‎  C. Music is an effective communication tool.‎ ‎  D. Music education makes students more imaginative.‎ ‎  [解析] 主旨大意题。文章阐述了音乐的内涵、音乐的功能、音乐的重要性,所以音乐课是需要我们特别重视的。‎ ‎  [答案] A ‎(第113篇)‎ ‎  Camping wild is a wonderful way to experience the natural world and, at its best, it makes little environmental influence. But with increasing numbers of people wanting to escape into the wilderness, it is becoming more and more important to camp unobtrusively (不引人注目地) and leave no mark.‎ ‎  Wild camping is not permitted in many places, particularly in crowded lowland Britain. Wherever you are, find out about organizations responsible for managing wild spaces, and contact them to find out their policy on camping and shelter building. For example, it is fine to camp wild in remote parts of Scotland, but in England you must ask the landowner's permission, except in national parks.‎ ‎  Camping is about having relaxation, sleeping outdoors, experiencing bad weather, and making do without modern conveniences. A busy,fullyequipped campsite(野营地) seems to go against this, so seek out smaller, more remote places with easy access to open spaces and perhaps beaches. Better still, find a campsite with no road access:walking in makes a real adventure.‎ ‎  Finding the right spot to camp is the first step to guaranteeing a good night's sleep. Choose a campsite with privacy and minimum influence on others and the environment. Try to use an area where people have obviously camped before rather than creating a new spot.When camping in woodland, avoid standing dead trees, which may fall on a windy night. Avoid animal runs and caves, and possible homes of biting insects. Make sure you have most protection on the windward side. If you make a fire, do so downwind of your shelter.Always consider what influence you might have on the natural world.Avoid damaging plants. A good campsite found, not made-changing it should be unnecessary.‎ ‎  [语篇解读] 野营是人们感受自然的绝佳方式,既感受自然又不破坏自然,但我们需要知道该做什么和不该做什么事情。‎ ‎  1.You needn't ask for permission when camping in________.‎ ‎  A. national parks in England ‎  B. most parts of Scotland ‎  C. crowded lowland Britain ‎  D. most parts of England ‎  [解析] 事实细节题。由第二段...,but in England you must ask the landowner's permission,except in national parks.可知A项正确。‎ ‎  [答案] A ‎  2.The author thinks that a good campsite is one________.‎ ‎  A. with easy access ‎  B. used previously ‎  C. with modern conveniences ‎  D. far away from beaches ‎  [解析] 推理判断题。由第三段最后一句"Better still,find a campsite with no road access..."排除A项;由第三段首句"Camping is about having relaxation,sleeping outdoors,experiencing bad weather,and making do without modern conveniences."排除C项;由"...seek out smaller,more remote places with easy access to open spaces and perhaps beaches."排除D项;由第四段第三句可知B项正确。‎ ‎  [答案] B ‎  3.The last paragraph mainly deals with________.‎ ‎  A. protecting animals ‎  B. building a campfire ‎  C. camping in woodland ‎  D. finding a campsite with privacy ‎  [解析] 事实细节题。由"When camping in woodland,avoid standing dead trees..."可知。‎ ‎  [答案] C ‎  4.The passage is mainly about________.‎ ‎  A. the protection of campsites ‎  B. the importance of wild camping ‎  C. the human influence on campsites ‎  D. the dos and don'ts of wild camping ‎  [解析] 主旨大意题。文章谈及了遵守野营规则、如何选择野营地点以及野营注意事项。‎ ‎  [答案] D ‎(第114篇)‎ ‎  ‎ ‎  Western New Bridge Library Announcement ‎  2008-3-10‎ Shortened Library Hours for Spring Break Library Hours have been shortened to 7 hours a day (9∶00 a. m4∶00 p. m. ) for Spring Break from March 24 to March 30.‎ Coming Events ‎·On Monday,March 24,at 10∶30 a. m. ,Scott Sutton,a children's writer,will tell stories to kids over seven. Sutton's attractive style will surely inspire everyone present!‎ ‎·At 1∶00 p. m. ,on March 26,the Georgetown Musicians will present an Irish Folk Concert, which will be entertaining for the entire family. Come for the music and stay to check out some relevant books for the rest of the week!‎ ‎·On Thursday, March 27, at 2∶00 p. m., the annual Children's Gathering will take place in Room 201 ,the secondfloor. Pick up an invitation in the Children's Room and return your RSVP(回复) to reserve your seat at the table by 3∶00 p. m. on Tuesday,March 25. Only children are allowed in the Gathering.‎ ‎·At 10∶30 a. m. on Friday, March 28, Enzo Monfre of the hit kids'science show,ENZOology,will bring Fossils Live! Surely Enzo will take the audience back in time, deep beneath the surface of the earth, to uncover the mysteries of killer dinosaurs, and more.Enzo recently appeared on the Ellen DeGeneres Showcome and see him at the library!‎ Please note:In case of emergency,please call the Help Desk at 9263736 and follow the procedures outlined on the voice message. The calldown service is staffed 24 hours a day,7 days a week for emergencies. The Help Desk supplies service to you all the year round!For questions about all these, please contact hld@wnbl.org.‎ Come for the great Fun;Stay for the relevant Books !‎ ‎  5. To attend the annual Children's Gathering, one has to________.‎ ‎  A. buy a ticket ‎  B. apply in advance ‎  C. make a reservation ‎  D. contact the calldown service ‎  [解析] 细节理解题。在第三项活动Children's Gathering中提到reserve your seat"预订座位",其他选项文中没有。‎ ‎  [答案] C ‎  6. According to the passage, Enzo Monfre will________.‎ ‎  A. show the children around a zoo ‎  B. tell stories to children over seven ‎  C. be present at the science show in person ‎  D. lead the children to the Ellen DeGeneres Show ‎  [解析] 细节理解题。A项"领孩子们参观动物园"文章没提。B项是第一项活动的内容。"Enzo recently appeared on the Ellen DeGeneres Show..."中的appeared可以看出C项正确,D项中的lead一词不对。‎ ‎  [答案] C ‎  7. The Help Desk in this library supplies service________.‎ ‎  A. only during the daytime ‎  B. in case of emergency ‎  C. till the end of the Spring Break ‎  D. after 22∶00 p. m. every day ‎  [解析] 细节理解题。从"...24 hours a day,7 days a week..."可以看出A、D两项不对。从"The Help Desk supplies service to you all the year round!"可以看出C项错误。从"In case of emergency,please call the Help Desk..."可以看出B项是正确的。‎ ‎  [答案] B ‎  8. We can learn from the passage that children can________.‎ ‎  A. attend all the activities with their parents ‎  B. borrow some relevant books for the activities ‎  C. participate in the activities from 8∶00 a. m. to 4∶00 p. m.‎ ‎  D. choose only one of the activities according to their interest ‎  [解析] 细节理解题。第三项活动只有儿童参加,所以A项不对。从"Stay for the relevant Books!"可以看出B项正确。各个活动都有具体时间,并不是从上午9点到下午4点随时参加,所以C项不对。文中没有限定只能参加其中一项活动,所以D项不对。‎ ‎  [答案] B ‎(第115篇)‎ ‎  Sunday is more like Monday than it used to be. Places of business that used to keep daytime" business hours" are now open late into the night. And on the Internet, the hour of the day and the day of the week have become irrelevant(不相关的). A half century ago in the United States, most people experienced strong and precise dividing lines between days of rest and days of work,school time and summer time. Today the boundaries still exist, but they seem not clear.‎ ‎  The law in almost all states used to require stores to close on Sunday; in most, it no longer does. It used to keep the schools open in all seasons except summer; in most, it still does. And whether the work week should strengthen its legal limits,or whether it should become more" flexible", is often debated. How should we, as a society,organize our time? Should we go even further in relaxing the boundaries of time until we live in a world in which every minute is much like every other?‎ ‎  These are not easy questions even to ask. Part of the difficulty is that we rarely recognize the"law of time" even when we meet it face to face. We know as children that we ‎ have to attend school a certain number of hours, a certain number of days, a certain number of years-but unless we meet the truant officer(学监), we may well think that we should go to school due to social custom and parents' demand rather than to the law. As adults we are familiar with"extra pay for overtime working," but less familiar with the fact that what constitutes(构成) "overtime" is a matter of legal definition. When we turn the clock forward to start daylightsaving time, have we ever thought to ourselves:" Here is the law in action"? As we shall see, there is a lot of law that has great influence on how we organize and use time: compulsory education law, overtime law, and daylightsaving law-as well as law about Sunday closing, holidays, being late to work ,time zones, and so on. Once we begin to look for it, we will have no trouble finding a law of time to examine and assess.‎ ‎  [语篇解读] 文章通过开头句(Sunday is more like Monday than it used to be.和过去相比,现在星期天倒更像星期一。)表明了我们的生活由时间规律来支配。‎ ‎  1.By saying "Sunday is more like Monday that it used to be",the writer means that________.‎ ‎  A. work time is equal to rest time ‎  B. many people have a day off on Monday ‎  C. it is hard for people to decide when to rest ‎  D. the line between work time and rest time is unclear ‎  [解析] 推理判断题。第一段首句Sunday is more like Monday than it used to be.是提纲挈领式的句子,后面紧接着举例论证了这一观点尤其是根据第一段最后一句可知我们上班时间和休息时间这两个界限的模糊性。从而推断本题选择D。‎ ‎  [答案] D ‎  2.The author raises the questions in Paragraph 2 to introduce the fact that people________.‎ ‎  A. fail to make full use of their time ‎  B. enjoy working overtime for extra pay ‎  C. are unaware of the law of time ‎  D. welcome flexible working hours ‎  [解析] 推理判断题。第二段主要论述时间规律问题,比如过去要求所有的商场星期天关门休业;过去除了夏季外,要求所有的学校所有季节全部开放,在很大程度上,今天商场不是关门休息,而是继续营业,学校继续上课,从而说明了人们没有意识到时间的自然规律问题。因此答案选择C。‎ ‎  [答案] C ‎  3.According to the passage, most children tend to believe that they go to school because they________.‎ ‎  A. need to acquire knowledge ‎  B. have to obey their parents ‎  C. need to find companions ‎  D. have to observe the law ‎  [解析] 事实细节题。根据第三段We know as children that we have to attend school a certain number of hours,a certain number of days,a certain number of years-but unless we meet the truant officer,we may well think we should go to school due to social custom and parents' demand rather than the law.这句信息句分析得知,这里大部分的孩子认为他们必须上学的原因应该是服从父母而非遵守法律。故选择B。‎ ‎  [答案] B ‎  4.What is the main idea of the passage?‎ ‎  A. Our life is governed by the law of time.‎ ‎  B. How to organize time is not worth debating.‎ ‎  C. New ways of using time change our society.‎ ‎  D. Our time schedule is decided by social customs.‎ ‎  [解析] 主旨大意题。从头到尾,文章论述的是时间掌控我们的生活。最后一句就是点睛之笔。‎ ‎  [答案] A ‎(第116篇)‎ ‎  Last week my youngest son and I visited my father at his new home in Tucson, Arizona. He moved there a few years ago,and I was eager to see his new place and meet his friends.‎ ‎  My earliest memories of my father are of a tall,handsome,successful man devoted to his work and family,but uncomfortable with his children. As a child I loved him;as a school girl and young adult (成年人) I feared him and felt bitter about him. He seemed unhappy with me unless I got straight A's and unhappy with my boyfriends if their fathers were not as "successful" as he was. Whenever I went out with him on weekends, I used to struggle to think up things to say, feeling on guard.‎ ‎  On the first day of my visit, we went out with one of my father's friends for lunch at an outdoor cafe. We walked along that afternoon,did some shopping, ate on the street table, and laughed over my son's funny facial expressions. Gone was my father's critical (挑剔的) air and strict rules. Who was this person I knew as my father,who seemed so friendly and interesting to be around? What had held him back before?‎ ‎  The next day my dad pulled out his childhood pictures and told me quite a few stories about his own childhood. Although our times together became easier over the years, I never felt closer to him at that moment. After so many years, I'm at last seeing another side of my father. And in so doing, I'm delighted with my_new_friend. My dad, in his new home in Arizona, is back to me from where he was.‎ ‎  5. Why did the author feel bitter about her father as a young adult?‎ ‎  A. He was silent most of the time.‎ ‎  B. He was too proud of himself.‎ ‎  C. He did not love his children.‎ ‎  D. He expected too much of her.‎ ‎  [解析] 细节理解题。根据文章第二段中的"He seemed unhappy with me unless I got straight A's and unhappy with my boyfriends if their fathers were not as 'successful'as he was. "可知,作者小时候怀恨爸爸是因为爸爸对她要求很严厉、很苛刻。‎ ‎  [答案] D ‎  6. When the author went out with her father on weekends, she would feel________.‎ ‎  A. nervous  B. sorry ‎  C. tired D. safe ‎  [解析] 推理判断题。根据第二段中"Whenever I went out with him on weekends,I used to struggle to think up things to say,feeling on guard."可知每次和父亲一起外出度周末,她都非常紧张。on guard为固定短语,意为"警戒"。‎ ‎  [答案] A ‎  7. What does the author think of her father after her visit to Tucson?‎ ‎  A. More critical.‎ ‎  B. More talkative.‎ ‎  C. Gentle and friendly.‎ ‎  D. Strict and hardworking.‎ ‎  [解析] 推理判断题。根据文章第三段和第四段的描述可知,作者的爸爸其实也很温和、友善。从第三段结尾的几个反问句可以知道。第四段中作者感到和爸爸成了朋友。‎ ‎  [答案] C ‎  8. The underlined words "my new friend" in the last paragraph refer to________.‎ ‎  A. the author's son ‎  B. the author's father ‎  C. the friend of the author's father ‎  D. the café owner ‎  [解析] 推理判断题。文中的new friend指的是作者的爸爸。作者以前一直以为爸爸是个严厉、苛刻的人,和他相处时,总是感到惧怕和紧张,但是在到了Tucson,和爸爸接触之后才看到爸爸友善的一面,所以作者感到她有了一个新朋友。‎ ‎(第117篇)‎ ‎  Celebrity(名人) has become one of the most important representatives of popular culture. Fans used to be crazy about specific film,but now the public tends to base its consumption (消费)on the interest of celebrity attached to any given product. Besides,fashion magazines have almost abandoned the practice of putting models on the cover because they don't sell nearly as well as famous faces. As a result.celebrities have realized their unbelievably powerful market potential, moving from advertising for others'products to developing their own.‎ ‎  Celebrity clothing lines aren't a completely new phenomenon,but in the past they were typically aimed at the ordinary consumers,and limited to a few TV actresses. Today they're ‎ started by firstclass stars whose products enjoy equal fame with some world top brands. The most successful startups have been those by celebrities with specific personal style. As celebrities become more and more experienced at the market,they expand their production scale rapidly, covering almost all the products of daily life.‎ ‎  However,for every success story, there's a related warning tale of a celebrity who overvalued his consumer appeal. No matter how famous the product's origin is, if it fails to impress consumers with its own qualities it begins to resemble an exercise in selfpromotional marketing. And once the initial (最初的) attention dies down, consumer interest might fade, loyalty (忠诚) returning to triedandtrue labels.‎ ‎  Today, celebrities face even more severe embarrassment. The popcultural circle might be bigger than ever, but its rate of turnover has speeded up as well. Each_misstep_threatens_to_reduce_a_celebrity's_shelf_life,_and the same newspaper or magazine that once brought him fame has no problem picking him to pieces when the opportunity appears. Still,the ego's (自我的)potential for expansion is limitless. Having already achieved great wealth and public recognition,many celebrities see fashion as the next frontier to be conquered. As the saying goes, success and failure always go hand in hand. Their success as designers might last only a short time, but fashionlike celebrityhas always been temporary.‎ ‎  5. Fashion magazines today________.‎ ‎  A. seldom put models on the cover ‎  B. no longer put models on the cover ‎  C. need not worry about celebrities'market potential ‎  D. judge the market potential of every celebrity correctly ‎  [解析] 细节理解题。由第一段"fashion magazines have almost abandoned the practice of putting models on the cover"可知答案。‎ ‎  [答案] A ‎  6. A change in the consumer market can be found today that________.‎ ‎  A. price rather than brand name is more concerned ‎  B. producers prefer models to celebrities for advertisements ‎  C. producers prefer TV actresses to film stars for advertisements ‎  D. quality rather than the outside of products is more concerned ‎  [解析] 推理判断题。文章第三段论述名人如果高估了自己的形象价值而忽视了质量问题,人们还是最终会失去兴趣而信任质量过硬的品牌。‎ ‎  [答案] D ‎  7. The underlined sentence in Paragraph 4 indicates that any wrong step will possibly________.‎ ‎  A. decrease the popularity of a celebrity and the sales of his products ‎  B. damage the image of a celebrity in the eyes of the general public ‎  C. cut short the artistic career of a celebrity in show business ‎  D. influence the price of a celebrity's products ‎  [解析] 推理判断题。联系此句上下文:流行文化圈越来越大,但失败的风险也会越来越大,依靠名人效应的品牌一旦失足,影响的不只是名人自己的形象还有企业的效益。A项最全面,其他选项片面或偏离中心。‎ ‎  [答案] A ‎  8. The passage is mainly about________.‎ ‎  A. celebrity and personal style ‎  B. celebrity and market potential ‎  C. celebrity and fashion design ‎  D. celebrity and clothing industry ‎  [解析] 主旨大意题。通过对文章的分析可以看出,文章讲的是名人和市场潜能的关系,而不是和个人风格、流行设计、服装工业的关系。‎ ‎  [答案] B ‎(第118篇)‎ ‎  Having a husband means an extra seven hours of housework each week for women, according to a new study. For men, getting married saves an hour of housework a week. "It's a wellknown_pattern," said lead researcher Frank Stafford at University of Michigan's Institute for Social Research. "Men usually work more outside the home, while women take on more of the housework."‎ ‎  He points out that differences among households (家庭) exist.But in general, marriage means more housework for women and less for men. "And the situation gets worse for women when they have children. "Stafford said.‎ ‎  Overall,times are changing in the American home. In 1976,women busied themselves with 26 weekly hours of sweepinganddusting work, compared with 17 hours in 2005. Men are taking on more housework, more than doubling their housework hours from six in 1976 to 13 in 2005.‎ ‎  Single women in their 20s and 30s did the least housework, about 12 weekly hours,while married women in their 60s and 70s did the most-about 21 hours a week.‎ ‎  Men showed a somewhat different pattern ,with older men picking up the broom more often than younger men. Single men worked the hardest around the house, more than that of all other age groups of married men.‎ ‎  Having children increases housework even further. With more than three children, for example, wives took on more of the extra work,clocking about 28 hours a week compared with husbands' 10 hours.‎ ‎  [语篇解读] 过去,对于女人来说,结婚意味着每周多做7个小时的家务;对于男人来说,结婚意味着每周少做1小时的家务。而如今,情况却发生了变化。‎ ‎  1.According to the"wellknown pattern"in Paragraph 1, a married man________.‎ ‎  A. takes on heavier work ‎  B. does more housework ‎  C. is the main breadwinner ‎  D. is the master of the house ‎  [解析] 猜测词义题。第一段最后一句告诉我们:男人通常在外挣钱养家,而女人则更多负责家务。‎ ‎  [答案] C ‎  2.How many hours of housework did men do every week in the 1970s?‎ ‎  A. About 28.  B. About 26.‎ ‎  C. About 13. D. About 6.‎ ‎  [解析] 事实细节题。第三段最后一句告诉我们:男人在20世纪70年代每周做家务6小时。‎ ‎  [答案] D ‎  3.What kind of man is doing most housework according to the text?‎ ‎  A. An unmarried man.‎ ‎  B. An older married man.‎ ‎  C. A younger married man.‎ ‎  D. A married man with children.‎ ‎  [解析] 事实细节题。第五段最后一句告诉我们:单身男子做家务最多。‎ ‎  [答案] A ‎  4.What can we conclude from Stafford's research?‎ ‎  A. Marriage gives men more freedom.‎ ‎  B. Marriage has effects on job choices.‎ ‎  C. Housework sharing changes over time.‎ ‎  D. Having children means doubled housework.‎ ‎  [解析] 主旨大意题。本文主要告诉我们:男女分担家务随着时间的推移发生了变化。‎ ‎  [答案] C ‎(第119篇)‎ ‎  You're sitting on the train home and the person opposite you yawns (打哈欠). Suddenly, you're yawning with him,though you're not tired.‎ ‎  This phenomenon confused scientists for years until a recent study found that people tend to sympathize with fellow humans. Supporting this claim was the discovery that those children who were unable to form normal emotional ties with others did not experience contagious (有感染力的) yawning,which showed that humans communicate regularly without words.‎ ‎  Hugo Critchley,a neuroscientist, has conducted an experiment recently, which will prove that happiness and sadness can spread like the common cold. According to Critchley,our mind ‎ and body are in constant exchange about how we're feeling."Emotions are closely linked with states of internal(内部的) responses," he explained. "There are also more visible changes in our gestures and facial expressions. When we're in a group,these signals can spread to another person. For example,there's the obvious tendency to smile when smiled at and there are less obvious changes that reflect emotions of surprise, anger or sadness such as a change in our heart rate and blood pressure."‎ ‎  Hugo Critchley further explained,"Our bodies synchronise and when we like the other person,we even copy his behavior. Next time you chat with a friend, take note of how you're sitting-it's pretty likely that you will be the same. Scientists believe it's our way of telling each other that we're partners. Through body language, humans give each other very subtle(微妙的) but clear signals that show emotions."‎ ‎  So,what lessons can we learn from this? "Spend time with happy people-otherwise your health could suffer," said Critchley. "When we're sad, our body goes into fight or flight mode. But when we're happy,our body works normally and we feel relaxed and positive. So we look bright,our skin glows,we feel healthy and it affects everyone around us."‎ ‎  5. According to Hugo Critchley,________.‎ ‎  A. emotions are as visible as facial expressions ‎  B. we yawn more frequently when we have a cold ‎  C. emotions are connected with states of internal responses ‎  D. the change of blood pressure is not linked with the change of emotions ‎  [解析] 细节判断题。从第三段"Emotions are closely linked with states of internal responses"可以看出C项是正确。从第三段"There are also more visible changes in our gestures and facial expressions."可知A项不对。从第三段最后一句话可见D项错误。‎ ‎  [答案] C ‎  6. The underlined word "synchronise" in Paragraph 4 means"________".‎ ‎  A. move slowly ‎  B. change rapidly ‎  C. relax temporarily ‎  D. respond accordingly ‎  [解析] 词义猜测题。由synchronise后的"...when we like the other person,we even copy his behavior."可以看出答案:当我们喜欢对方时,我们就模仿他的行为。‎ ‎  [答案] D ‎  7. From the passage we can learn________.‎ ‎  A. sadness is as contagious as happiness ‎  B. anger is less contagious than friendliness ‎  C. surprise is more contagious than smile ‎  D. surprise is the most contagious among emotions ‎  [解析] 推理判断题。这四个选项是考查学生对第三段最后一句话的理解"For example,there's the obvious tendency to smile when smiled at and there are less obvious changes that reflect emotions of surprise,anger or sadness such as a change in our heart rate and blood pressure."当人们对我们微笑时,我们显然有微笑的倾向;那些反映惊奇、生气或悲伤的情绪却不太容易受影响而变化。‎ ‎  [答案] B ‎  8. Which of the following statements is true according to the passage?‎ ‎  A. Emotions have delicate influence on fellow humans.‎ ‎  B. Children like copying the actions of fellow humans.‎ ‎  C. Scientists are still confused about contagious yawning.‎ ‎  D. People tend to communicate more with body language.‎ ‎  [解析] 判断正误题。整篇文章都在论述情绪会对同伴有微妙的影响:第一段举例说明,第二段和第三段用理论说明,第四段讲我们喜欢一个人就会模仿他,因此A项正确。‎ ‎  [答案] A ‎(第120篇)‎ ‎  A recent study, while showing a generally positive attitude toward science, also suggests a widespread worry that it may be"running out of control". This idea is dangerous.‎ ‎  Science can be a force for evil as well as for good. Its applications can be channeled either way, depending on our decisions. The decisions we make, personally or collectively,will determine the outcomes of science. But here is a real danger. Science is advancing so fast and is so strongly influenced by businesses that we are likely to believe whatever decisions we come to will make little difference.And, rather than fighting for the best possible policies, we may step back and do nothing.‎ ‎  Some people go even further. They say that despite the moral and legal objections(反对) ,whatever is scientifically possible will be done-somewhere, sometime. They believe that science will get out of control in the end. This belief is dangerous too, because it fuels a sense of hopelessness and discourages them from making efforts to build a safer world.‎ ‎  In our interconnected world, the lack of agreement in and out of the world of science can lead to the failure to control the use of science. Without a common understanding, the challenges of"controlling" science in this century will be really tough. Take human cloning for example. Despite the general agreement among scientists on its possible huge impact(影响)on traditional moral values,some countries still go ahead with the research and development of its related techniques. The outcomes are hard to predict.‎ ‎  Therefore,discussions on how science is applied should be extended far beyond scientific societies. Only through the united efforts of people with hope, can we be fully safe against the misuse of science and can science best serve mankind in the future.‎ ‎  [语篇解读] 最近的一次调查表明了人们对于科学及其如何运用于人类世界的不同态度。尽管大多数人都持积极的态度,但有人认为科学发展及其应用最终可能危害人类。只有通过大家的努力,才能让科学最好地服务于人类。‎ ‎  1.What can we conclude from the recent study?‎ ‎  A. People think highly of science.‎ ‎  B. People hold mixed opinions about science.‎ ‎  C. Science is getting dangerously out of control.‎ ‎  D. Science is used for both good and bad purposes.‎ ‎  [解析] 推理判断题。第一段已点明了全文的主要内容:尽管人们对科学一般都是积极的态度,但最近调查表明还存在着广泛的担忧。‎ ‎  [答案] B ‎  2.According to the passage, what will happen if we hold that science is getting beyond control?‎ ‎  A. The development of science will hopelessly slow down.‎ ‎  B. Businesses will have even greater influence on science.‎ ‎  C. The public will lose faith in bringing about a bright future.‎ ‎  D. People will work more actively to put science under control.‎ ‎  [解析] 事实细节题。从第三段最后两句可知答案为C。‎ ‎  [答案] C ‎  3.The discussion should reach beyond scientific societies because________.‎ ‎  A. scientists have failed to predict the outcomes ‎  B. the ties between different areas need strengthening ‎  C. united efforts are necessary for the development of science ‎  D. people need to work together to prevent the bad use of science ‎  [解析] 推理判断题。最后一段最后一句句意:只有通过充满希望的人们团结努力,我们才能安全地与错误地使用科学的现象作斗争,才能让科学最好地服务于人类。‎ ‎  [答案] D ‎  4.What is the main idea of the passage?‎ ‎  A. Science and its applications bring us many dangers.‎ ‎  B. The development of science mostly lies in people's attitudes.‎ ‎  C. Mankind can largely take control of science with their efforts.‎ ‎  D. The future of science will be influenced by the dangerous ideas.‎ ‎  [解析] 主旨大意题。第一段表明了全文的主要内容。‎ ‎  [答案] B ‎  [长难句解读] Science is advancing so fast and is so strongly influenced by businesses that we are likely to believe whatever decisions we come to will make little ‎ difference.科学进步如此之快,受商业影响如此之深,让我们有可能认为我们无论做出何种决定都没有什么作用。本句是so...that 的句式结构。that引导的结果状语从句中含有whatever引导的宾语从句。‎ ‎  Despite the general agreement among scientists on its possible huge impact on traditional moral values,some countries still go ahead with the research and development of its related techniques.尽管科学家们普遍认同人类克隆可能会给传统的道德价值观念带来巨大的影响,但有些国家仍然进行与此相关的技术研究及开发。agreement on...就......达成共识;impact on...对......有影响。‎
查看更多

相关文章

您可能关注的文档